You are on page 1of 146

lOMoARcPSD|38673565

LPT - Reviewer

Life and Works of Rizal (Bulacan Agricultural State College)

Scan to open on Studocu

Studocu is not sponsored or endorsed by any college or university


Downloaded by Noime Burtanog (noimeburtanog@gmail.com)
lOMoARcPSD|38673565

LET REVIEWER

General Education | Vocabulary Words to Remember


 AUDIENCE – a number of people listening to a lecture
 ALTRUIST – one, who considers the happiness and well-being of others first
 ATHEIST – a person who does not believe in God
 ANTHROPOLOGIST – one, who studies the evolution of mankind
 AUTOCRACY – government by one person
 AUTOBIOGRAPHY – the life history of a person written by himself
 AMPUTATE – to cut off a part of a person's body which is infected
 ARSENAL – a place for ammunition and weapons
 ARCHIVES – a place where government or public records are kept
 AMATEUR – a man who does a thing for pleasure and not as a profession
 ARISTOCRACY – government by the nobles
 AQUATIC – animals/plants which live in water
 AMPHIBIAN – animals which live both on land and sea
 AMBIDEXTER – one, who can use either hand with ease
 ALIMONY – allowance paid to wife on legal separation
 ANTHOLOGY – a collection of poems
 ABDICATION – voluntary giving up of throne in favor of someone
 ARBITRATOR – a person, appointed by two parties to solve a dispute
 ASTRONOMER – a person, who studies stars, planets and other heavenly bodies
 ASTROLOGER – a person who studies the influence of heavenly bodies on human
beings
 AXIOM – a statement which is accepted as true without proof
 AGENDA – a list of headings of the business to be transacted at a meeting
 ANARCHIST – one, who is out to destroy all governance, law and order
 ALMANAC – an annual calendar with positions of stars
 BANKRUPT – a person who is unable to pay his debts
 BIGAMY – the practice of having two wives or husbands at a time
 BIBLIOPHILE – a lover and collector of books
 BOUQUET – a collection of flowers
 BUREAUCRACY – government by the officials
 BELLIGERENT – a person, nation that is involved in war
 BIENNIAL – an event which happens once in two years
 BLASPHEMY – the act of speaking disrespectfully about sacred things
 CRECHE – a nursery where children are cared for while their parents are at work
 COSMOPOLITAN – a person who regards whole world as his country
 CHAUFFEUR – one, who is employed to drive a motor car
 CURATOR – a person in charge of a museum
 CARNIVOROUS – one, who lives on flesh
 CANNIBAL – one, who feeds on human flesh
 CONTEMPORARIES – belonging to or living at the same time
 CLOAK ROOM – a place for luggage at railway station
 CYNOSURE – center of attraction
 CONNOISSEUR – a critical judge of any art and craft
 CRUSADE – a religious war
 CHOREOGRAPHER – one, who teaches dancing
 CACOGRAPHIST – a person, who is bad in spellings
 CALLIGRAPHIST – a person, who writes beautiful handwriting
 CYNIC – one, who sneers at the aims and beliefs of his fellow men
 CONVALESCENT – one, who is recovering health
 CAVALRY – soldiers, who fight on horse back
 CARDIOLOGIST – a person, who is specialist in heart diseases
 CARTOGRAPHER – one, who draws maps
 DORMITORY – the sleeping rooms with several beds especially in a college or
institution
 DRAWN – a game that results neither in victory nor in defeat
 ELEGY – a poem of lamentation
 EPITAPH – words which are inscribed on the grave or the tomb in the memory of the
buried

Downloaded by Noime Burtanog (noimeburtanog@gmail.com)


lOMoARcPSD|38673565

 EPHEMERAL – lasting one day


 EFFEMINATE – a person who is womanish
 EMIGRANT – a person who leaves his own country and goes to live in another
 EDIBLE – fit to be eaten
 EGOISM – practice of talking too much about oneself
 ENCYCLOPEDIA – a book that contains information on various subjects
 EPICURE – one, who is devoted to the pleasure of eating and drinking
 FLORIST – one, who deals-in flowers
 FASTIDIOUS – one, who is very -selective in one's taste
 FANATIC OR BIGOT – one, who is filled with excessive and mistaken enthusiasm in
religious matters
 FATAL – causing death
 FATALIST – one, who believes in fate
 FACSIMILE – an exact copy of handwriting, printing etc.
 FAUNA – the animals of a certain region
 FLORA – the plants of a particular region
 FRATRICIDE – murder of brother
 FUGITIVE – one, who runs away from justice or the law
 FRAGILE – easily broken
 FEMINIST – one, who works for the welfare of the women
 GRANARY – a place for grains
 GENOCIDE – murder of race
 GREGARIOUS – animals which live in flocks
 HANGAR – a place for landing aeroplanes
 HIVE – a place for bees
 HORTICULTURE – the art of cultivating and managing gardens
 HOMICIDE – murder of man
 HEARSE – a vehicle which is used to carry a dead body
 HEDONIST – one, who believes that pleasure is the chief good (sensual)
 HORIZON – a line at which the earth and the sky seem to meet
 HONORARY – holding office without any remuneration
 HERETIC – one, who acts against religion
 HERBIVOROUS – one, who lives on herbs
 INSOLVENT/BANKRUPT – a person who is unable to pay his debts
 INAUDIBLE – a sound that cannot be heard
 INACCESSIBLE – that cannot be easily approached
 INCORRIGIBLE – incapable of being corrected
 IRREPARABLE – incapable of being repaired
 ILLEGIBLE – incapable of being read
 INEVITABLE – incapable of being avoided
 IMPRACTICABLE – incapable of being practiced
 IMMIGRANT – a person who comes to one country from another in order to settle there
 INVINCIBLE – undefeatable
 INDELIBLE – that cannot be erased
 INCOGNITO – travelling under another name than one's own
 INDEFATIGABLE – one, who does not tire easily
 INFALLIBLE – one, who is free from all mistakes and failures
 INVIGILATOR – one, who supervises in the examination hall
 ITINERANT – one, who journeys from place to place
 INFIRMARY – a home or room used for ill or injured people
 INFANTICIDE – murder of an infant
 INFANTRY – soldiers, who fight on foot
 INFLAMMABLE – liable to catch fire easily
 INTERREGNUM – a period of interval between two reigns or governments
 KENNEL – a place for dogs
 LUNATIC ASYLUM – a home for lunatics
 LEXICOGRAPHER – one, who compiles a dictionary
 LOQUACIOUS – one, who talks continuously
 LINGUIST – one, who is skilled in foreign languages
 LAPIDIST – one, who cuts precious stones
 MISANTHROPE – a hater of mankind
 MISOGAMIST – one, who hates marriage

Downloaded by Noime Burtanog (noimeburtanog@gmail.com)


lOMoARcPSD|38673565

 MORTUARY – a place, where dead bodies are kept for post mortem
 MERCENERY – working only for the sake of money
 MATRICIDE – murder of mother
 MARTYR – one, who dies for a noble cause
 MAIDEN SPEECH – the first speech delivered by a person
 MINT – a place where coins are made
 MISOGYNIST – a hater of womankind
 MORGUE – a place, where dead bodies are kept for identification
 MAMMALS – animals which give milk
 MONOGAMY – the practice of marrying one at a time
 MISSIONARY – a person, who is sent to propagate religion
 NUMISMATICS – the study of coins
 NAMESAKE – a person having same name as another
 NOSTALGIA – a strong desire to return home, home sickness
 NOVICE OR TYRO – one, new to anything, inexperienced
 NARCOTIC – a medicine for producing sleep
 OPTIMIST – a person who looks at the brighter side of things
 ORPHAN – one, who has lost parents
 OMNIPRESENT – one, who is present everywhere
 OMNIPOTENT – one, who is all powerful
 OMNISCIENT – one, who knows everything
 OMNIVOROUS – one, who eats everything.
 OBITUARY – an account in the newspaper of the funeral of the one deceased
 ORPHANAGE – a home for orphans
 OBSTETRICIAN – one, who is skilled in midwifery
 OSTLER – one, who looks after horses at an inn
 OPAQUE – that which cannot be seen through
 PESSIMIST – a person who looks at the darker side of things
 POTABLE – fit to drink
 POST MORTEM – an examination of dead body
 PHILANTHROPIST – a lover of mankind
 PATRICIDE – murder of father
 PHILATELIST – one, who collects stamps
 PLAGIARISM – literary theft or passing off an author's original work as one's own
 POLYGAMY – the practice of marrying more than one wife at a time
 POLYANDRY – the practice of marrying more than one husband at a time
 PHILOGYNIST – a lover of womankind
 PLEBISCITE – (a decision made by) votes of all qualified citizens
 PHILANDERER – one, who amuses himself by love making
 PHILISTINE – one who does not care for art and literature
 PLUTOCRACY – government by the rich
 PSEUDONYM – an imaginary name assumed by an author for disguise
 POSTHUMOUS – a child born after the death of his father or the book published after
the death of the writer
 PANACEA – a remedy for all diseases
 PEDIATRICIAN – a person, who is specialist in child diseases
 PLATITUDE – ordinary remarks often repeated
 PEDANT – one, who makes a vain display of his knowledge
 POLYGLOT – one, who speaks many languages
 PALEOGRAPHY – the study of ancient writing
 POSSE – a number of policemen called to quell a riot
 PAROLE – pledge given by a prisoner for temporary release, not to escape
 PEDESTRIAN – one, who goes on foot
 PORTABLE – that can be carried easily
 QUARANTINE – an act of separation from other persons to avoid infection
 RHETORIC – the art of elegant speech or writing
 REGICIDE – murder of King or Queen
 SACRILEGE – violating or profaning religious things/places
 SCULPTOR – one, who cuts in stones
 SUICIDE – murder of oneself
 STABLE – a place for horses
 SOMNAMBULIST – a person, who walks in sleep

Downloaded by Noime Burtanog (noimeburtanog@gmail.com)


lOMoARcPSD|38673565

 SOMNILOQUIST – a person, who talks in sleep


 SOUVENIR – a thing kept as a reminder of a person, place or event
 SWAN SONG – the last work (literary) of a writer
 SOT, TOPER – one, who is a habitual drunkard
 SINECURE – a job with high salary but little responsibility
 STOIC – a person, who is indifferent to pleasure and pain and has control over his
passions
 SANATORIUM – a place for the sick to recover health
 SORORICIDE – murder of sister
 TRIENNIAL – an event which happens once in three years
 TRUANT – a person/student who absents himself from class or duty without permission
 TEETOTALLER – one, who does not take any intoxicating drink
 TRANSPARENT – that which can be seen through
 THEOCRACY – government by religious principles
 UXORIOUS – one extremely fond of one's wife
 UTOPIA – an imaginary perfect social and political system
 UXORICIDE – murder of wife
 VERBATIM – repetition of speech or writing word for word
 VOLUNTEER – one, who offers one's services
 VIRGIN – a woman who has no sexual experience
 VERSATILE – expert in different things
 VETERAN – one, who has a long experience of any occupation
 VENIAL – a fault that may be forgiven
 WARDROBE – a place for clothes Best of luck

LAWS IN EDUCATION
 PRC BR 435 – Code of Ethics for Professional Teachers
 PD 1006 – Decree Professionalizing Teachers
 RA NO. 1425 – inclusion of the works of Jose Rizal
 RA NO. 4670 – “Magna Carta for Public School Teacher”
 RA 7722 – CHED
 RA 7796 – “TESDA Act of 1994”
 RA 7836 – Phil. Teachers Professionalization Act of 1994
 RA 9155 – BEGA (Basic Educ.) or DepEd Law
 RA 9293 – Teachers Professionalization Act
 RA 10533 – K-12 Law
 ACT NO. 2706 – “Private School Law”
 COMMONWEALTH ACT NO. 578 – “persons in authority”
 KAUTUSANG PANGKAGAWARAN BLG 7 - PILIPINO National Language
 PROKLAMA BLG 12 - Linggo ng Wika (Balagtas, Mr29-Ap4)
 PROKLAMA BLG. 186 – Linggo ng Wika (Quezon, Ag13-19)
 PROKLAMA BLG. 1041 – Buwan ng Wika (Ramos)
 PHIL. CONSTITUTION ACT 14 – ESTACS
 RA 1079 – no limit of Civil Service eligibility
 RA 6655 – “Free Public Secondary Educ. Act of 1988”
 RA 6728 – “Act Providing Government Assistance to Students and Teachers in Private
Education
 RA 7277 – Magna Carta for PWD
 RA 7610 – Anti-Child Abuse Law (Amendment: RA 9231)
 RA 7743 – establishment of public libraries
 RA 7877 – “Anti Sexual Harassment Act of 1995”
 RA 7880 – “Fair and Equitable Access to Education Act”
 RA 8049 – Anti-Hazing Law
 RA 8187 – Paternity Act
 RA 10627 – Anti-Bullying
 SB 1987 ART. 14 SEK. 6-9 – FILIPINO (National Language)

Prepositions of TIME AT, ON, and IN.


 AT- usually use in exact time like at 7pm, at 3 o’clock, etc.
 ON - is used with days and dates like on January 2, on Monday, on Tuesday, on the 4th of
April
 IN - is used with months and years like in April, in June, in 1990, in 2017, etc.

Downloaded by Noime Burtanog (noimeburtanog@gmail.com)


lOMoARcPSD|38673565

Prepositions of LOCATION, AT, ON, and IN.


 AT- usually use in specific point like at the center, at the corner, and in addresses usually
if there is a house number like “I live at #3 Roxas Street”, etc...
 ON - is used with surfaces like on the wall, on the floor, on the table, on the ground, and
in addresses usually if it’s only the name of the street, avenue, road like “I Live on Roxas
Street, I live on Roxas Avenue”.
 IN - is used when the thing is inside the box like in the structure, like in the kitchen, in
the room, in the hall and with addresses IN is use with cities, places like in Seoul, in the
Philippines, in Mindanao, etc.

FILIPINO

MGA TEORYA NG PINAGMULAN NG WIKA


 BOW-WOW –kalikasan at hayop
 DING-DONG – bagay
 POOH-POOH – masidhing damdamin
 YOHEHO – pwersang pisikal
MGA URI NG PANGUNGUSAP
WALANG PAKSA:
 EKSISTENSYAL – mayroong isa o higit pang tao
Halimbawa: Mayroon daw puno sa bakuran.
 MODAL – nais/pwede/maari (Gusto ko matulog.)
 PANLIPUNAN – pagbati, pagbigay galang atbp.
 SAGOT LAMANG – “Talaga?”, “Oo”
 SAMBITLA – masidhing damdamin (Aray!)
 TEMPORAL – panandaliang kalagayan o panahon
KAYARIAN:
 PAYAK – iisang kaisipan
 TAMBALAN – dalawang sugnay na ‘di makapag-iisa
 HUGNAYAN – madalas nagsisimula sa kung, dahil sa
 LANGKAPAN – mahabang pangungusap
MGA URI NG KWENTO
 PABULA (fable) – hayop
 PARABULA (parable) – Bibliya
 ANEKDOTA (anecdote) – tunay na buhay
 MITOLOHIYA (myth) – diyos at diyosa (pinagmulan)
ASPEKTO NG PANDIWA (Verb)
 PERPEKTIBO – tumakbo
 IMPERPEKTIBO – tumatakbo
 KONTEMPLATIBO – tatakbo
KAANTASAN NG PANG-URI (Adjective)
 LANTAY – walang pinaghahambingan
 PAHAMBING – inihahalintulad
 PASUKDOL – nangingibabaw (H: pinakamataas)
MGA URI NG TULA
 PATULA (Moro-moro)
 PASALAYSAY (Epiko, Awit, Korido)
MGA AWITING BAYAN
 DALIT/HIMNO – pagsamba sa anito o pang-relihiyon
 DIONA – kasal
 DUNG-AW – patay (pagdadalamhati)
 KALUSAN – paggawa
 KUMINTANG – tagumpay (pandigma)
 KUNDIMAN – pag-ibig
 OYAYI – pagpapatulog ng bata
 SOLIRANIN – pagsasagwan
 TALINDAW – pamamangka
PAGBABAGONG MORPONEMIKO
 ASIMILASYON – Parsyal (pangsukli), Ganap (panukli)
 MAY ANGKOP – wikain mo – “kamo”
 MAYSUDLONG/PAGDARAGDAG NG PONEMA – muntik – muntikan, pagmuntikan,
pagmuntikanan

Downloaded by Noime Burtanog (noimeburtanog@gmail.com)


lOMoARcPSD|38673565

 METATESIS – linipad – nilipad


 PAGKAKALTAS NG PONEMO – takipan – takpan
 PAGLILIPAT-DIIN – laRUan (playground) - laruAN (toy)
 PAGPAPALIT NG PONEMA – madapat – marapat
MGA URI NG PANGHALIP/PRONOUNS
 PANAO/PERSONAL PRONOUN – ako/I etc.
 PAMATLIG/DEMONSTRATIVE PRONOUN – ito/this etc.
 PANAKLAW/INDEFINITE P. - isa, all, anyone etc.
 PATULAD – ganito, ganyan atbp.
 PANANONG/INTERROGATIVE P. – sino, when etc.
 PAMANGGIT/RELATIVE P. – daw, umano, which, who
MGA AKDANG NA MAY IMPUWENSYA SA MUNDO
 AKLAT NG MGA ARAW – China (by Confucius)
 AKLAT NG MGA PATAY – Egypt cults & myths (by Osiris)
 AWIT NI ROLANDO – France (by Doce Pares, Roncesvalles)
 BIBLIYA – Palestino at Greece
 CANTEBURY TALES – America (by Chaucer)
 DIVINE COMEDIA – Italy (by Dante)
 EL CID COMPEADOR – katangian at history ng Spain
 ILIAD o ODYSSEY – Myths of Greece made by Homer.
 ISANG LIBO’T ISANG GABI – Ugali sa Arabia at Persia
 KORAN – Arabia (Muslim Bible) MAHABRATA – India
 UNCLE TOM’S CABIN – about slaves that becomes the basis of democracy. (by Harriet
Beecher Stowe of U.S.)
MGA URI NG PANITIKAN
1. TULUYAN – binubuo ng mga pangungusap
 NOBELA – binubuo ng mga kabanata
 DULA – pagtatanghal sa entablado
A. MGA DULANG PANLIBANGAN:
 TIBAG – Sta. Elena
 LAGAY – Pilarenos ng Sorsogon
 PANUNULUYAN – pagtatanghal bago mag-alas dose (12PM) ng gabi ng
kapaskuhan
 PANUBOL – parangal sa may kaarawan
 KARILYO – ala-puppet show
 KURIDO – katapangan, kabayanihan, kababalaghan, pananampalataya
 SARSUELA – musical tungkol sa pag-ibig, paghihiganti atbp. masisidhing
damdamin
B. ALAMAT – pinagmulan
C. ANEKDOTA – ugali, may mabuting aral
2. PATULA - may sukat,pantig,tugma,taludtod,saknong
A. TULANG PASALAYSAY - mahahalagang tago o pangyayari sa buhay.
a. EPIKO – kabayanihan sa kababalaghan
 BIDASARI, PARANG SABIR – Moro
 BIAG NI LAM ANG – Iloko
 MARAGTAS, HARAYA, LAGDA AT HARI SA BUKID – Bisaya
 KUMINTANG – Tagalog
 DAGOY AT SUDSUD – Tagbanua
 TATUANG - Bagobo
b. AWIT o KORIDO - kaharian
c. TULA NG DAMDAMIN o LIRIKO – own feeling
MGA TULANG LIRIKO:
 AWITING BAYAN – kalungkutan
 ELEHIYA – yumao
 DALIT – pagpupuri sa Diyos
 PASTORAL – buhay sa bukid
 ODA – papuri
A. TULANG DULA O PANGTANGHALAN
a. KOMEDYA
b. MELODRAMA – musical
c. TRAHEDYA – death of main character

Downloaded by Noime Burtanog (noimeburtanog@gmail.com)


lOMoARcPSD|38673565

d. PARSA – mga pangyayaring nakakatawa


e. SAYNETE – karaniwang pag-uugali ng tao/ pook
B. TULANG PATNIGAN
a. KARAGATAN – alamat ng singsing ng prinsesa na naihulog niya sa dagat sa
hangaring mapangasawa ang kasintahang mahirap.
b. DUPLO – paligsahan ng husay sa pagtula
c. BALAGTASAN – pumalit sa Duplo

FIGURES OF SPEECH/TAYUTAY
PAG-UUGNAY O PAGHAHAMBING:
 SIMILE/PAGTUTULAD – mayroong pangatnig
 METAPHOR/PAGWAWANGIS – walang pangatnig
 ALUSYON – iba’t ibang aspekto ng buhay ng tao
 METONYMY/PAGPAPALIT – TAWAG
 SYNECDOCHE/SINEKDOKE – pagbanggit ng isa upang tukuyin ang kabuuan
– Hal: Dalawang bibig ang umaasa kay Romeo.
PAGLALARAWAN:
 HYPERBOLE/ PAGMAMALABIS o EKSAHERASYON
 APOSTROPHE/PAGTAWAG – pakikipag-usap sa hindi buhay o malayong tao.
– Hal: Ulan, tumigil ka na.
 EXCLAMATION/PAGDARAMDAM – strong feeling.
 PARADOX/PARADOKS – “malayo ma’y malapit pa rin”
 OXYMORON/PAGTATAMBIS – paradox w/ extra words
PAGSASALIN NG KATANGIAN:
 PERSONIFICATION/PAGSASATAO
 PAGSASATUNOG
 ONOMATOPOEIA/PANGHIHIMIG – tunog ang paksa
 ALLITERATION/PAG-UULIT – repetition of 1st letter in the 1st word.
– Ex: Dinggin mo ang Diyos na Dinadakila
 REPITASYON – repetition of phrase.
– Ex: Tama! Tama!
IBA PANG TAYUTAY NA GAMIT SA TULA:
 ALITERASYON – unang titik o pantig ay pare-pareho
 ANADIPLOSIS – paggamit ng salita sa unahan at hulihan
 EPIPORA – pag-uulit ng salita sa hulihan
 PAG-UYAM – sarcasm
 LITOTES – pagtanggi o pagkukunwari.
 TALUDTOD – linya sa tula

BAHAGI NG PANANALITA
MGA URI NG PANGATNIG
 PANINSAY – Ito ay ginagamit sa pangungusap na ang dalawang isipan ay
nagkakasalungatan.
– Halimbawa: Namatay si Mang Isko ngunit ang kanyang prinsipyo ay mananatiling
buhay.
 PANANHI – Ito ay ginagamit upang makatugon sa mga tanong na bakit o upang
maipakilala ang mga kadahilanan ng isang pangyayari at ng anumang iniisip o niloloob.
– Halimbawa: Ang kanyang prinsipyo ay mananatiling buhay sapagkat nariyan si Dong
na magpapatuloy ng kanyang naudlot na gawain.
 PAMUKOD – Ito ay ginagamit upang ihiwalay, itangi, o itakwil ang isa sa ilang bagay o
isipan.
– Halimbawa: Maging ang mga kasamahan niya’y nagpupuyos ang kalooban.
 PANLINAW – ito ay ginagamit upang dagdagan o susugan ang kalinawan ng mga nasabi
na.
– Halimbawa: Sumisigaw ang kanyang pusoat humihingi ng katarungan.
 PANAPOS – Nagsasaad ito ng wakas ng pagsasalita.
– Halimbawa: At sa wakas naibigay rin ang kanilang sahod.
 PANUBALI – Nagsasaad ito ng pagkukurong di-ganap at nangangailangan ng ibang diwa
o pangungusap upang mabuo ang kahulugan.
– Halimbawa: Sakaling hindi ibigay, magpapatuloy ang welga.
PANG-UKOL – (preposition) ginagamit kung para kanino o para saan ang kilos.

Downloaded by Noime Burtanog (noimeburtanog@gmail.com)


lOMoARcPSD|38673565

PANG-ANGKOP – (ligature) bahagi ng pananalita na ginagamit para maging maganda


pakinggan ang pagkakasabi ng pangungusap.
– Halimbawa: na, ng, g. magandang bata.

URI NG PANG-ABAY
 PANG-ABAY NA PAMANAHON – nagsasaad kung kailan naganap o magaganap ang
kilos.
Halimbawa:
Kailangan mo bang pumasok nang araw-araw?
Tuwing pasko ay nagtitipon silang mag-anak.
 PANG-ABAY NA PANLUNAN – tumutukoy sa pook na pinangyarihan. Karaniwang
ginagamit ang pariralang sa, kina/kay
Halimbawa:
Maraming masasarap na ulam ang itinitinda sa kantina.
Nagpaluto ako kina aling Ingga ng masarap ng keyk para sa iyong kaarawan.
 PANG-ABAY NA PAMARAAN – naglalarawan kung paano naganap, nagaganap, o
magaganap ang kilos
Halimbawa:
Kinamayan niya ako nang mahigpit.
Bakit siya umalis na umiiyak?
 PANG-ABAY NA PANG-AGAM – nagbabadya ng di-katiyakan sa pagganap sa kilos ng
pandiwa.
Halimbawa:
marahil, siguro, tila, baka, wari, atb.
Halimbawa:
Marami na marahil ang nakabalita tungkol sa desisyon ng Sandiganbayan.
Higit sigurong marami ang dadalo ngayon sa Ateneo Home Coming kaysa
nakaraang taon.
Tila patuloy na ang pag-unlad ng turismo sa Pilipinas.
 PANG-ABAY NA PANANG-AYON – nagsasaad ng pagsang-ayon. Gumagamit ng Oo,
opo, tunay, sadya, talaga, atb.
Halimbawa:
Oo,asahan mo ang aking tulong.
Talagang mabilis ang pag-unlad ng bayan.
 PANG-ABAY NA PANANGGI – nag-sasaad ng pagtanggi, tulad ng hindi/di at ayaw.
Halimbawa:
Hindi pa lubusang nagagamot ang kanser
Ngunit marami pa rin ang ayaw tumigil sa paninigarilyo.
 PANG-ABAY NA PANGGAANO O PAMPANUKAT – nagsasaad ng timbang o sukat.
Sumasagot sa tanong na gaano o magkano.
Halimbawa:
Tumaba ako nang limang libra.
Tumagal nang isang oras ang operasyon.
 PANG-ABAY NA PAMITAGAN – nagsasad ng paggalang.
Halimbawa:
Kailan po kayo uuwi?
Opo, aakyat na po ako
 INGKLITIK O PANINGIT – mga katagang laging sumusunod sa unang salita ng
kayariang kinabibilangan. Mayroong 16 na ingklitik sa Filipino
– ba, daw/raw, pala, man, kasi, din/rin, tuloy, muna, kaya, naman, nga, pa, na, yata,
lamang/lang, sana,
 PANG-ABAY NA KUNDISYUNAL – nagsasaad ng kundisyon para maganap ang kilos
na isinasaad ng pandiwa.
– Pinangungunahan ng kung, kapag o pag at pagka
 PANG-ABAY NA KAWSATIBO – tawag sa pang-abay na nagsasaad ng dahilan sa
pagganap ng kilos ng pandiwa
– Binubuo ng parirala o sugnay na pinangungunahan ng dahil sa
 PANG-ABAY NA BENEPAKTIBO – tawag sa mga pang-abay na nagsasaad ng
benepisyo para sa isang tao dahil sa pagkaganap sa kilos ng pandiwa o ng layunin ng
pandiwa.
 PANG-ABAY NA PANGKAUKULAN - pinangungunahan ng tungkol, hinggil o ukol

Downloaded by Noime Burtanog (noimeburtanog@gmail.com)


lOMoARcPSD|38673565

UNFAMILIAR PARTS OF THE SPEECH


 PREPOSITIONS – on, under, off, by, in near, for, to, since
 CONJUNCTIONS (PANGATNIG) – para/for, at/and, nor, or, pero/but, yet, so, ni, ngunit
 INTERJECTION – with exclamation mark

PROPER SEQUENCE OF WORDS IN A SENTENCE


1. ARTICLES – a, an, the
2. OPINION
3. SIZE
4. AGE
5. SHAPE
6. COLOR
7. MATERIAL
8. PURPOSE

CLASSIFICATIONS OF POEM
 BALLAD – narrative, less folk tale/legend, to be sung
 BLACK VERSE – with meter but no rhyme
 DRAMATIC MONOLOGUE – written in form of speech for individual character.
 ELEGY –death of individual
 EPIC – tells a story about heroic figure
 EULOGY – message for the dead
 FREE VERSE (vers libre) – without meter but with rhyme
 HAIKU – Japanese poem about nature. 5, 7, 5 (3 lines and 17 syllables) 9. IDYLL (Idyl)
– peaceful, idealized country scene
 LYRICS - thoughts and feelings
 NARRATIVE – tells story
 ODE -typically serious/meditative nature, type of Lyric
 PASTORAL –rural life in peaceful & romanticized way
 SONNET – Lyric poem consists of 14 lines
 TANKA – Japanese poem: 5 lines, 31 syllables

ENGLISH
 HENLY – “I am the master of my fate; I am the captain of my soul” poem of INVICTUS
 KEATS – “A thing of beauty is a joy forever"
 MARK TWAIN – American greatest humorist
 EPIC – a long narrative poem dealing with persons of heroic proportion & actions of
great significance
 EDGAR ALLAN POE – considered the father of the modern American short story
 HAIKU – Japanese poem w/ 17 syllables
 ANTHOLOGY – collection of literary pieces
 SONNET – 14 iambic pentameter lines
 MAHABHARATA – longest epic
 FOLKTALES – stories that reflect people's beliefs & are handed from generation to
generation
 FABLES – these are not tales making use of animals as characters
 MARCELO H. DEL PILAR – his pen name “Dolores manapat”
 QUEZON – “like the molave” his source of inspiration
 ELEGY – a poem lamenting the dead
 SOLILOQUY – a speech by a person who reveals his thoughts
 MANUEL ARGUILLA – author of “how my brother Leon brought home a wife”
 JOSE RIZAL – he wrote the famous letter “to the women of Malolos”
 URBANA AT FELIZA – a kind of literary piece w/c moralizes & was written in letter
between 2 sisters dwelling in the city& the other in the province
 WASHINGTON – author of “the legend of sleppy hollow”
 RHODORA – “if eyes are made for seeing, the beauty is its own excuse for being” – is
taken from the poem
 THE ILLIAD OF HOMER – A great epic poem whose plot centers around the anger &
wrath of Achilles against Agamemnon, a great Leader
 LEONARDO DA VINCI – famous work “Monalisa”
 JUAN LUNA – famous painting “Spolarium”
 MICHAEL ANGELO – created “the statue of David”

Downloaded by Noime Burtanog (noimeburtanog@gmail.com)


lOMoARcPSD|38673565

PROFESSIONAL EDUCATION

SUBCATEGORIES OF TEACHER MOVEMENT/MOVEMENT MANAGEMENT


ISM’s IN EDUCATION
 BEHAVIORISM – change
 ESSENTIALISM – basic
 EXISTENTIALISM – choice
 HUMANISM – build
 IDEALISM – enough in mind
 PERRENIALISM – constant
 PRAGMATISM - practice (T&E) PROGRESSIVISM – improve
 REALISM – enough to see
 UTILITARIANISM - best
 SOCIAL RECONSTRUCTIVISM – benefit of all

AIMS OF ERAS
 PRE-SPANISH – survival and conformity
 SPANISH – Christianity
 AMERICAN – democratic ideals and way of life
 COMMONWEALTH – moral character, efficiency
 JAPANESE – progress

PROF. ED PROPONENTS
 B.F. SKINNER – Operant Conditioning
 BANDURA – Modeling
 BANDURA & WALLACE – Social Learning
 CARL JUNG – Psychological
 CONFICIUS – Education for all, Golden Rule
 EDWARD THORNDIKE – Connectionism
 ERICK ERIKSON – Psychosocial
 IVAN PAVLOV – Classical Conditioning
 JEAN PIAGET – Cognitive
 FROEBEL - Father of Kindergarten
 PEZTALLOZI – realia, Froebel’s protégé
 JEROME BRUNER – Instrumental Conceptualism
 JOHN DEWEY – learning by doing
 JOHN LOCKE – Tabula Rasa (blank sheet)
 KOHLERS – Insight Learning
 LAURENCE KOHLBERG – Moral Development
 LEV VGOTSKY – Social Cognitivist, Scaffolding
 SIGMUND FREUD – Psychosexual
 WILLIAM SHELDON – Physiological

PRINCIPLES
 HEDONISM – pleasure principle
 DOUBLE EFFECT – sacrifice for the good or bad
 FORMAL COOPERATION – cooperation with will
 LESSER EVIL – choice of the less one from two bad things
 MATERIAL COOPERATION – cooperation without will

FREUD’S PSYCHOSEXUAL/PSYCHOANALYTICAL THEORY


 ORAL (0-1 yrs. old) – Infant
 ANAL (1-3 yrs. old) – Toddler
 PHALLIC – Preschool
 LATENCY – School Age
 GENITAL – Adolescence
OEDIPUS – son to mom
ELECTRA – daughter to dad

BRUNER’S THREE MODES OF REPRESENTATION

10

Downloaded by Noime Burtanog (noimeburtanog@gmail.com)


lOMoARcPSD|38673565

 ENACTIVE (0-1 yrs. old) – action-based information


 ICONIC (1-6 yrs. old) – image-based information
 SYMBOLIC (7+) – code/symbols such as language

TAXONOMY OF OBJECTIVES
A. COGNITIVE:
BLOOM (LOTS) ANDERSON (HOTS)
 Knowledge
 Comprehension
 Application
 Analysis
 Synthesis
 Evaluation or Remembering
 Understanding
 Applying
 Analyzing
 Evaluating
 Creating
B. AFFECTIVE:
 Receiving
 Responding
 Valuing
 Organizing
 Characterization
C. PSYCHOMOTOR:
 SIMPSON HARROW
 Perception
 Set
 Guided Response
 Mechanism
 Complex Overt Response
 Adaptation
 Origination o Reflex movement
 Fundamental Movement
 Physical Movement
 Perceptual Abilities
 Skilled Movements
 Non-discursive communication
 DALES CONE OF EXPERIENCE
 Read
 Hear
 Picture
 Video
 Exhibit
 Demonstration
 Collaborative Work
 Simulation
 Real thing

ERIKSON’S PSYCHOSOCIAL TASKS


 TRUST VS. MISTRUST (0-12 months)
 AUTONOMY VS. SHAME/DOUBT (1-3 years old)
 INITIATIVE VS. GUILT (3-6 years old)
 INDUSTRY VS. INFERIORITY (6-12 years old)
 INDENTITY VS. ROLE CONFUSION (12-18 years old)
 INTIMACY VS. ISOLATION (early 20s-early 40s
 GENERATIVITY VS. STAGNATION (40s-mid 60s)
 INTEGRITY VS. DESPAIR (mid 60s-death)

PIAGET’S COGNITIVE DEVELOPMENT THEORY


 SENSORY – senses
 PRE-OPERATIONAL - imagination

11

Downloaded by Noime Burtanog (noimeburtanog@gmail.com)


lOMoARcPSD|38673565

 CONCRETE
 FORMAL

GENERATIONS OF COMPUTER
 VACUUM TUBES (1940-1956)
 TRANSISTORS (1956-1963)
 INTEGRATED CIRCUITS (1964-1971)
 MICROPROCESSORS (1971-present)
 ARTIFICIAL INTELLIGENCE (present-future)

MISTAKEN GOALS
 ATTENTION SEEKER – “teacher, notice me”
 REVENGE – “teacher, I am hurt”
 POWER-SEEKING – “teacher, may I help?”
 INADEQUACY – “teacher, don’t give up on me”
 WITHDRAWAL – “teacher, please help me”

KOHLBERG’S THEORY OF MORAL DEVELOPMENT


 LEVEL 1: PRE-CONVENTIONAL MORALITY
 Stage 1: Obedience and Punishment Orientation
 Stage 2: Individualism and Exchange
 LEVEL 2: CONVENTIONAL MORALITY
 Stage 3: Good Interpersonal Relationships
 Stage 4: Maintaining the Social Order
 LEVEL 3: POST-CONVENTIONAL MORALITY
 Stage 5: Social Contract and Individual Rights
 Stage 6: Universal Principles

CENTRAL TENDENCY -Central (middle location) Tendency


 MEAN – Average
 MODE – most occurring
 RANGE – highest score minus lowest score
 LOW SD–Homogenous, scores near to mean (almost same)
 HIGH SD – Heterogenous, scores far to mean (scattered)
 DECILE – 10 groups (D1…D10) QUARTILE – 4 groups (Q1…Q4)
 SUSPENSION – time
 REVOKATION – condition

DIFFICULTY INDEX
 0-0.20 VERY DIFFICULT
 0.21-0.40 DIFFICULT
 0.41-0.60 MODERATELY DIFFICULT
 0.61-0.80 EASY
 0.81-1.00 VERY EASY

POSITIVELY SKEWED (LEFT FOOT) – low scores, mean greater than mode
NEGATIVELY SKEWED (RIGHT FOOT) – high scores, mean is lower than mode
HORN/HALO EFFECT – overcoming other trait, either bad/good

GENERAL EDUCATION

FILIPINO/ENGLISH:

FILIPINO POETS AND PEN NAMES


 BENVENIDO SANTOS – American Culture Writings
 DANIEL DEFOE – “Robinson Crusoen” (novel)
 EDILBERTO TIEMPO – made “Cry Slaughter” that has been translated many times
 ERNEST HEMINGWAY – Ring Lardner Jr.
 JOSE GARCIA VILLA – “Comma Poet”, Dove G. Lion
 MIGUEL DE CERVANTES/ESCALANDE – Masterpiece is “Don Quixote” that is most
influential

12

Downloaded by Noime Burtanog (noimeburtanog@gmail.com)


lOMoARcPSD|38673565

 NICK JOAQUIN – Quijano de Manila, “Spanish Culture Writing”


 PAZ MARQUEZ BENITEZ – made “Dead Stars” that is the 1st modern English short
story
 SAMUEL LANGHORNE CLEMENS – Mark Twain
 “Adventures of Tom Sawyer”
 “Adventures of Huckleberry Finn” (best novel) SINTAKS/SINTAKSIS/-
PALAUGNAYAN
 sangay ng barirala na tumatalakay sa masistemang pagkaka-ayus-ayos ng mga salita
sa pagbuo ng mga parirala at pangungusap

PEN NAMES OF FILIPINO PROTAGONISTS


 ANDRES BONIFACIO – May Pag-asa, Agapito, Bagumbayan
 ANTONIO LUNA – Taga-Ilog
 EMILIO AGUINALDO - Magdalo
 EMILIO JACINTO – Di Masilaw, Tingkian
 GRACIANO LOPEQ JAENA – Diego Laura
 JOSE MA. PANGANIBAN ¬– JoMaPa
 JOSE RIZAL – Dimas alang, Laon laan
 JUAN LUNA – Buan
 MARCELO DEL PILAR – Plaridel, Dolores Manapat, Piping Dilat
 MARIANO PONCE – Tikbalang, Naning (Satanas), Kalipulako

PHILIPPINE CONSTITUTIONS
 BIAK NA BATO – pact, thought of 1st Republic.
 Spaniards paid P200 000
 MALOLOS CONSTITUTION – Apolinario Mabini
 rights of soldiers
 no Visayas yet in right of territories
 1935 CONSTITUTION – adapted from American Const.
 1943 CONSITUTION – Jose P. Laurel
 Japan invades but gave freedom for Phil. to rule.
 1937 CONSTITUTION – Ferdinand Marcos
 Martial Law – 60days max
 Nat’l Territory forced Kalayaan grp. of Islands & Saba
 1987 CONSTITUTION - 18 articles
 past chairwoman: Cecilla Muñoz Palma (Feb 2, 1987)
 Bill of Rights are for the criminals
 JUS SANGUINI – blood
 JUS SOLI – place

TYPES OF GOVERNMENT
 �������� – a government whose authority is based on citizens' votes, which are
represented by elected or nominated officials chosen in free elections.
 ��������� – democracy means the rule of the people. The term today refers to a
political system in which the people or their elected representatives govern themselves.
 ��������� – a government where priests rule in the name of God or by officials
who are regarded as divinely guided, or consistent with the principles of a particular
religion.
 ��������� – a government-controlled by absolute power, and in the hands of a
single person with minimal restraints on the decisions and lack of any mechanisms of
popular control.
 ����������� – a government where scientists and technical experts are in
control of the state, and where rulers are selected on the basis of their knowledge/skill
rather than wealth/power.
 ������� – it is a way of ruling that advocates total control of the people and seeks to
promote ancestral and cultural values and eradicate foreign influences.
 ������� – is a state of absence of law, a state of lawlessness and disorder (usually
resulting from a failure of government.
 �������� – a government where supreme authority is vested in a single and
usually hereditary figure, such as a king, and whose powers can vary from absolute to
none at all.

13

Downloaded by Noime Burtanog (noimeburtanog@gmail.com)


lOMoARcPSD|38673565

��������� – rule of the few. It is a form of power structure in which power


effectively rests with a faction of persons or families.
 ���������� – government ruled by the rich or power provided by wealth, often
used to describe a wealthy class ruling a government, often from behind the scenes.
 ������� – government or authority of an absolute ruler; arbitrary exercise of power
over subjects not requisite for the purposes of government/approved by law and justice.
 ������������ – a totalitarian system is one in which a single political authority
regulates total control over the state, that is centralized and dictatorial.
 ���������� – a political organization characterized by a union of small states,
groups, or parties, which are self-governed in internal affairs and are united under a
central government.
 ��������� – communism is a system of government in which the state owns and
operates industry on behalf of the people.
 ����� – it refers to a group or coalition that takes control of the state after
overthrowing a government. Usually, this is done by military groups.
 ������������ – a form of government where the power rests entirely on one
person or a group of persons. This rule could be acquired by inheritance or force and is
usually oppressive.
SOME TYPES OF GOVERNMENTS
 COMMUNIST – classless society, state plans and controls economy
 PARLIAMENTARY – majority of people voted
 REPUBLICAN – power comes from people

PHILIPPINE PRESIDENTS
(AgQueLaOsRoQuiMagGarMaMarAquiRaEsArAquiDut)

ACTS
 ASSOCIATION OF SE ASIAN NATIONS (ASEAN)
– Myanmar, Cambodia, Laos, Brunei, Philippines, Singapore, Thailand, Indonesia,
Vietnam, Malaysia
 BELL TRADE (PHILIPPINE TRADE ACT) – bet. Phil. & U.S.
 KYOTO PROTOCOL (UNNCC) – fight global warming decreasing greenhouse gases
 RIO DE JANEIRO CONVENTION – environment and sustainable development
 TEJEROS CONVENTION – election, Bonifacio elected as Director Imperior
 TREATY ON GENERAL RELATIONS – recognition of U.S. to Philippine freedom
 UNDERWORLD-SIMMONS ACT – full free foreign trade
 PAYNE ALDRICH ACT – partial free foreign trade

MISSIONARIES AND EXPEDITIONS


 AUGUSTINIAN – most intelligent
 FRANCISCAN – sends medical aids
 JESUITS
 DOMINICANS – richest
 RECOLECTS – most killed schools
MARTYR PRIESTS
 BURGOS – youngest, mastermind of secularization
 GOMEZ – Oldest, likes “sabong” and hid there
 ZAMORA
HOMO HABILIS – man of steel (bighead, uses muscle)
HOMO ERECTUS – man who discovered fire & clothes
HOMO SAPIENS – thinking man (can produce materials)

EARTH’S SPHERES
 ATMOSPHERE – gaseous sphere protection from meteors
Divided into five:
 Troposphere
 Stratosphere
 Mesosphere
 Thermosphere
 Exosphere
 HYDROSPHERE – water
 LITHOSPHERE – oceanic and continental crust

14

Downloaded by Noime Burtanog (noimeburtanog@gmail.com)


lOMoARcPSD|38673565

 BIOSPHERE – all life forms in Earth


 CRYOSPHERE – ice
 ANTHROSPHERE – ancestors

PLANETS AND THEIR SEQUENCE


 SUN – 99.86% of Solar System
 believed was formed 4.6 billion years ago
 Responsible for weather and climate
 MERCURY – named after Roman God
 no satellite and atmosphere
 discovered by Mariner Ten
 VENUS – Goddess of Love and Beauty (Mariner 2)
 Perfect sphere, sister planet of Earth
 EARTH
 MARS – God of War, red planet (Mariner 9)
 JUPITER – Gas Giant, fastest rotating planet (10hrs less)
 has Great Red Spot: huge storm for 350yrs
 SATURN – God of Agriculture (chunks of rocks)
 made mostly of hydrogen
 URANUS – Frederick William Herscel
 Sky & Ice Giant, 3rd largest planet

SCIENCE PROPONENTS
 CAROLUS LINNAEUS – Father of Taxonomy
 ROBERT HOOKE – termed “cells” (cellulae)
 ANTON VAN LEUWENHOEK – 1st person to observe microscopic organisms (animal
cule)
 ROBERT BROWN – discovered Nucleus
 MATTHIAS SCHIEDEN (Botanist) & THEODORE (Zoologist) – found all plants
consist of cells
 RUDOLF VIRCHOW – proposed cells come fr. existing cells

PROPONENTS AND THEIR THEORIES


 WILHELM WUNDT – father of modern psychology
 SIGMUND FREUD – father of psychoanalysis, and psychosexual theory
 JOHANN HEINRICH – father of education and pedagogy
 IVAN PAVLOV – classical conditioning
 BURRHUS F. SKINNER – operant conditioning.
 DAVID AUSUBEL – meaningful learning
 JEROME BRUNER – discovery learning, spiral curriculum
 ALBERT BANDURA – social cognitive learning theory.
 EDWARD LEE THORNDIKE – law of readiness and exercises
 KURT LEVIN – life space content.
 KOHLER – problem solving by insight, insightful learning
 URIE BROFENBRENNER – ecological theory
 SANDRA BEM – gender schema theory
 HOWARD GARDNER – theory of multiple intelligence
 ELLIOT TURRIEL – Social domain theory
 LAWRENCE KOHLBERG – moral development theory
 ROBERT STERNBERG – triathlon theory intelligence
 ERIK ERIKSON – psychosocial development theory
 MA. MONTESSORI – transfer of learning, kindergarten preparation of children.
 EDWARD PAUL TORRANCE – creative problem solving
 CHOMSKY – linguistic acquisition theory
 JEAN PIAGET – cognitive learning theory
 JOHN WATSON – behavioral theory
 EDWARD TOLMAN – purpose behaviorism
 BERNARD WEINER – attribution theory
 DANIEL GOLEMAN – emotional intelligence.
 TITCHENER – structuralism psychology
 ROBERT GAGNE – sequence of instruction
 ABRAHAM MASLOW – hierarchy of needs, motivation theory

15

Downloaded by Noime Burtanog (noimeburtanog@gmail.com)


lOMoARcPSD|38673565

 BENJAMIN BLOOM – bloom's cognitive taxonomy


 DAVID KRATHWOHL – affective domain
 LEV VYGOTSKY – socio-cultural theory of cognitive devt, linguistic theory,
Scaffolding
 JOHN LOCKE – tabularasa, empiricism
 CHARLES COOLEY – looking glass self-theory
 JOHN FLAVEL – metacognition
 ARNOLD GESELL – maturation theory
 JOHN DEWEY – Learning by doing
 DAVID FROEBEL – Father of kindergarten
 AUGUSTE COMTE – Father of Sociology.
 JOHN AMOS COMENCIUS – Father of modern education.

PHILIPPINE HISTORY

 The first book written in the Philippines was DOCTRINA CRISTIANA.


 The Father of Ilocano Literature is PEDRO BUKANEG.
 The Father of Tagalog Poetry is FRANCISCO BALTAZAR.
 Lola Basyang is the pen name of SEVERINO REYES.
 The first and longest running comics series in the Philippines is KENKOY (Liwayway
Magasin,1929)
 The Father of Pampango Literature who wrote “There is no God” is JUAN
CRISOSTOMO SOTO.
 The oldest existing newspaper in the Philippines since the 1900 is MANILA BULLETIN.
 The Father of Modern Tagalog Poetry is ALEJANDRO ABADILLA.
 The work of Bonifacio which tells the history of the Philippines ANG DAPAT
MABATID NG MGA TAGALOG.
 He wrote the popular fable The Monkey and the Turtle – JOSE RIZAL
 This is known as Andres Bonifacio’s Ten Commandments of the Katipunan – THE
DECALOGUE.
 Rizal’s model for Pilosopong Tasyo was PACIANO RIZAL.
 The following characters created by Rizal reflect his own personality except SIMOUN
(El Filibusterismo)
 The line “whoever knows not how to love his native tongue is worse than any beast or
even smelly fish” TO MY FELLOW CHILDREN
 Rizal’s pen name – DIMASALANG, LAONG-LAAN
 Juan Luna’s pen name is TAGA-ILOG.
 The first Filipino alphabet was called ALIBATA/BAYBAYIN
 The first Filipino alphabet consisted of 15 LETTERS
 This is a song about love – TALINDAW
 Awit ng mga taong hindi naimbetahan sa kainan (COLADO)
 He was known for his `Memoria Fotografica` - JOSE MA. PANGANIBAN
 He is known as the `poet of the workers or laborers` - AMADO HERNANDEZ
 Ilocano balagtasan is called BUKANEGAN
 Visayan epic about good manners and right conduct – MARAGTAS
 The father of Filipino newspaper is PASCUAL POBLETE
 Lupang Tinubuan is considered to be the best story written during Japanese Period. The
author is NARCISO REYES
 The original title of Ibong Adarna was CORIDO AT BUHAY NA PINAGDAANAN NG
TATLONG PRINSIPENG ANAC NG HARING FERNANDO AT REYNA
VALERIANA SA CAHARIANG BERBANIA
 First Filipino bread – PANDEREGLA
 The Great Plebian: ANDRES BONIFACIO
 The Father of the Katipunan: ANDRES BONIFACIO
 Hero of the Tirad Pass Battle: GREGORIO DEL PILAR
 President of the First Philippine Republic: GENERAL EMILIO AGUINALDO
 Brains of the Philippine Revolution: APOLINARIO MABINI
 Martyred Priests in 1872: GOMBURZA
 Brains of the Katipunan: EMILIO JACINTO
 Co-founder of La Independencia: GENERAL ANTONIO LUNA
 Mother of Balintawak: MELCHORA AQUINO

16

Downloaded by Noime Burtanog (noimeburtanog@gmail.com)


lOMoARcPSD|38673565

 Greatest Filipino Orator of the Propaganda Movement: GRACIANO LOPEZ- JAENA


 First Filipino Cannon-maker: PANDAR PIRA
 Managing Editor of La Solidaridad: MARIANO PONCE
 Lakambini of Katipunan: GREGORIA DE JESUS
 Poet of the Revolution: FERNANDO MA. GUERRERO
 Outstanding Diplomat of the First Philippine Republic: FELIPE AGONCILL
 First University of the Philippines President: RAFAEL PALMA
 Greatest Filipino Painter: JUAN LUNA
 Greatest Journalist of the Propaganda Movement: MARCELO H. DEL PILAR
 First Filipino Poetess: LEONA FLORENTINO
 Peace of the Revolution: PEDRO PATERNO
 Founder of Philippine Socialism: ISABELO
 Delos Reyes Viborra: ARTEMIO RICARTE
 Author of the Spanish lyrics of the Philippine National Anthem: JOSE PALMA
 Chief of Tondo: LAKANDOLA
 The Last Rajah of Manila: RAJAH SOLIMAN
 Fiancée of Jose Rizal: LEONOR RIVERA
 Maker of the First Filipino Flag: MARCELA AGONCILLO
 Co-founder of Katipunan: GALICANO APACIBLE
 Leader of the Ilocano Revolt: DIEGO SILANG
 First Filipino Hero: LAPU-LAPU
 Leader of the Longest Revolt in Bohol: FRANCISCO DAGOHOY
 The Man of Many Talents: EPIFANIO DELOS SANTOS
 Prince of Tagalog Poets: FRANCISCO BALTAZAR
 Visayan Joan of Arc: TERESA MAGBANUA
 Mother of Biak-na-Bato: TRINIDAD TECSON
 Wife of Artemio Ricarte: AGUEDA
 Esteban Leader of the Tarlac Revolt: GEN. FRANCISCO MAKABULOS
 Composer of the Philippine National Anthem: JULIAN FELIPE
 Spaniards born in the Philippines: INSULARES
 Leader of Magdalo: BALDOMERO AGUINALDO
 Leader of Magdiwang: MARIANO ALVAREZ
 Founder of La Liga Filipina: JOSE RIZAL
 Painter of the Spolarium: JUAN LUNA
 FERDINAND MAGELLAN – named the Philippines “Islas de San Lazaro’
 ROY LOPEZ DE VILLALOBOS – named the Philippines “Las Islas Filipinas”
 MIGUEL LOPEZ DE LEGAZPI - known as El Adelantado and first governor of the
Philippines from 1565 until his death
 JOSÉ RIZAL - the national hero of the Philippines
 ISABELO DE LOS REYES – founder of Philippine socialism
 CLARO M. RECTO – Great Dissenter
 PEDRO ABAD SANTOS – founded the socialist party of Philippines in 1929
 JOMALUL KIRAM II – last sultan of Sulu
 JULIO NAKPIL – husband of Gregoria de Jesus after Andres Bonifacio
 FELIPE AGONCILLO – first Filipino diplomat
 EMILIO JACINTO – wrote the “Kartilla” and brain of the Katipunan.
 GOV. GEN. EULOGIO DESPUJOL – Spanish gov. gen. who ordered the deportation of
Rizal to Dapitan
 APOLINARIO DE LA CRUZ – known as “Hermano Pule”.
 GREGORIA DE JESUS – first woman member of Katipunan, “Lakambini of Katipunan”
 EMILIO AGUINALDO – created the designs for the Philippine national flag
 SERGEANT FERDINAND LAMADRID – Filipino soldier who led the Cavite Mutiny
of 1872
 APOLINARIO MABINI – chief advisor of Gen. Emilio Aguinaldo, sublime paralytic and
the “Brain of Revolution”
 GREGORIO DEL PILAR – hero of the Battle at the Tirad Pass
 CARMEN PLANAS – first woman councilor of Manila
 RAMON MAGSAYSAY – former president died in a plane crash
 FRANCISCO DAGOHOY – led the longest revolt in the Philippines during the Spanish
times
 GABRIELA SILANG – wife of Diego Silang, continued the revolt against the Spaniards
in the Ilocos region after Diego’s death

17

Downloaded by Noime Burtanog (noimeburtanog@gmail.com)


lOMoARcPSD|38673565

 SIMEON OLA – last general of the Filipino-American revolution to surrender to the


Americans
 IGNACIO VILLAMOR – first Filipino president of the University of the Philippines
 GENERAL ANTONIO LUNA – greatest general of the revolution and editor of La In
dependencia
 MELCHORA AQUINO – known as Tandang Sora. Recognized as the Grand Woman of
Revolution and the Mother of Balintawak.
 GRACIANO LOPEZ-JAENA – founder and first editor of La Solidaridad
 FERNANDO MA. GUERRERO – greatest lyric poet in Spanish.
 JUAN LUNA – A genius of the brush and a patriot of the highest order. He is a brother of
Gen. Antonio Luna. And a creator of creator of the world-famous painting,
SPOLARIUM.
 PEDRO PATERNO – negotiator/mediator of the Pact of Biak-na-Bato and helped prepare
the Malolos constitution.
 LEONA FLORENTINO – first poetess of the Philippines. Her poems were given
international recognition at the Exposicion in Madrid in 1887 and in Paris in 1889

PARTS OF THE CELL

PLANT CELL
 CELL WALL is the outermost rigid covering of the plant cell. It is a salient feature of
plant cell.
 CELL MEMBRANE is the outer lining of the cell.
 CYTOSOL OR CYTOPLASM is the gel-like matrix inside the cell membrane which
constitutes all other cell organelles.
 NUCLEUS is the control center of the cell. It is a membrane bound structure which
contains the hereditary material of the cell - the DNA
 CHLOROPLAST is a plastid with green pigment chlorophyll. It traps light energy and
converts it to chemical energy by the process of photosynthesis.
 MITOCHONDRIA carry out cellular respiration and provides energy to the cells.
 VACUOLES are the temporary storage center of the cell.
 GOLGI BODY is the unit where proteins are sorted and packed.
 RIBOSOMES are structures that assemble proteins.
 ENDOPLASMIC RETICULUM are membrane covered organelles that transport
materials.

ANIMAL CELL
 CELL MEMBRANE
 It is a semi-permeable barrier, allowing only a few molecules to move across it.
 Electron microscopic studies of cell membrane shows the lipid bi-layer model of the
plasma membrane, it also known as the fluid mosaic model.
 The cell membrane is made up of phospholipids which has polar (hydrophilic) heads
and non-polar (hydrophobic) tails.
 CYTOPLASM
 The fluid matrix that fills the cell is the cytoplasm.
 The cellular organelles are suspended in this matrix of the cytoplasm.
 This matrix maintains the pressure of the cell, ensures the cell doesn't shrink or burst.
 NUCLEUS
 The house for most of the cell’s genetic material- the DNA and RNA.
 The nucleus is surrounded by a porous membrane known as the nuclear membrane.
 The RNA moves in/out of the nucleus through these pores.
 Proteins needed by the nucleus enter through the nuclear pores.
 The RNA helps in protein synthesis through transcription process.
 The nucleus controls the activity of the cell and is known as the control center.
 The nucleolus is the dark spot in the nucleus, and it is the location for ribosome
formation.
 RIBOSOMES
 Ribosomes is the site for protein synthesis where the translation of the RNA takes
place.
 As protein synthesis is very important to the cell, ribosomes are found in large
number in all cells.

18

Downloaded by Noime Burtanog (noimeburtanog@gmail.com)


lOMoARcPSD|38673565

 Ribosomes are found freely suspended in the cytoplasm and also are attached to the
endoplasmic reticulum.
 ENDOPLASMIC RETICULUM
 ER is the transport system of the cell. It transports molecules that need certain
changes and also molecules to their destination.
 ER is of two types, rough and smooth.
 ER bound to the ribosomes appear rough and is the rough endoplasmic reticulum;
while the smooth ER do not have the ribosomes.
 LYSOSOMES
 It is the digestive system of the cell.
 They have digestive enzymes helps in breakdown the waste molecules and also help
in detoxification of the cell.
 If the lysosomes were not membrane bound the cell could not have used the
destructive enzymes.
 CENTROSOMES
 It is located near the nucleus of the cell and is known as the 'microtubule organizing
center' of the cell.
 Microtubules are made in the centrosome.
 During mitosis the centrosome aids in dividing of the cell and moving of the
chromosome to the opposite sides of the cell.
 VACUOLES
 They are bound by single membrane and small organelles.
 In many organisms, vacuoles are storage organelles.
 Vesicles are smaller vacuoles which function for transport in/out of the cell.
 GOLGI BODIES
 Golgi bodies are the packaging center of the cell.
 The Golgi bodies modify the molecules from the rough ER by dividing them into
smaller units with membrane known as vesicles.
 They are flattened stacks of membrane-bound sacs.
 MITOCHONDRIA
 Mitochondria is the main energy source of the cell.
 They are called the power house of the cell because energy (ATP) is created here.
 Mitochondria consists of inner and outer membrane.
 It is spherical or rod-shaped organelle.
 It is an organelle which is independent as it has its own hereditary material.
 PEROXISOMES
 Peroxisomes are single membrane bound organelle that contain oxidative enzymes
that are digestive in function.
 They help in digesting long chains of fatty acids and amino acids and help in
synthesis of cholesterol.
 CYTOSKELETON
 It is the network of microtubules and microfilament fibers.
 They give structural support and maintain the shape of the cell.
 CILIA AND FLAGELLA
 Cilia and flagella are structurally identical structures.
 They are different based on the function they perform and their length.
 Cilia are short and are in large number per cell while flagella are longer and are fewer
in number.
 They are organelles of movement.
 The flagellar motion is undulating and wave-like whereas the ciliary movement is
power stroke and recovery stroke.

PHILOSOPHICAL FOUNDATIONS OF EDUCATION


 IDEALISM – Plato (own ideas) nothing exists except in the mind of a man/ what we
want the world to be
 REALISM – Aristotle; Herbart; Comenius; Pestalozzi; Montessori; Hobbes; Bacon;
Locke
(experience) fully mastery of knowledge
 BEHAVIORISM – always guided by standards/by procedure; purpose is to modify the
behavior
 EXISTENTIALISM – Kierkegaard; Sartre; "Man shapes his being as he lives"

19

Downloaded by Noime Burtanog (noimeburtanog@gmail.com)


lOMoARcPSD|38673565

– Focuses on self/individual
 PRAGMATISM/EXPERIMENTALISM – William James; John Dewey - learn from
experiences through interaction to the environment
– Emphasizes the needs and interests of the children
 PERENNIALISM – Robert Hutchins
– focuses on unchanging/universal truths
 ESSENTIALISM – William Bagley
– teaching the basic/essential knowledge, focuses on basic skills and knowledge
 PROGRESSIVISM – Dewey/Pestalozzi (process of development)
– focuses on the whole child and the cultivation of individuality
 CONSTRUCTIVISM – Jean Piaget
– Focused on how humans make meaning in relation to the interaction b/w their
experiences and their ideas.
– Nature of knowledge w/c represents an epistemological stance.
 SOCIAL RECONSTRUCTIONISM – George Counts
– recognized that education was the means of preparing people for creating his new social
order
– highlights social reform as the aim of education
 ACCULTURATION – learning other culture; the passing of customs, beliefs and
tradition through interaction and reading.
 ENCULTURATION – the passing of group's custom, beliefs and traditions from one
generation to the next generation
 CONVERGENT QUESTIONS – are those that typically have one correct answer.
 DIVERGENT QUESTIONS also called open-ended questions are used to encourage
many answers and generate greater participation of students. Higher order thinking skills;
to think more creatively.

FAMOUS SCIENTISTS

ARCHIMEDES OF SYRACUSE (287 - 212 BC)


 Due to the fact that Archimedes was a person with multiple interests, including math,
physics, astronomy and engineering, he managed to become a successful inventor. He
became famous for his innovative thinking and was the one to come up with a lot of
innovative machines, including the ARCHIMEDES SCREW that is still used today to
pump liquids, coal, grain and more. But probably his most famous discovery is the way
of finding the volume of objects that have an irregular shape. It is worth mentioning that
most of his works were meant to help his home city.
LEONARDO DA VINCI (1452 - 1519)
 Leonardo Da Vinci was a great inventor, being fond of engineering, astronomy,
aeronautics, mathematics and more. His drawings include a number of various
inventions, the basics of which more or less can be found in different technologies today.
 Da Vinci is considered to be the “FATHER OF MODERN SCIENCE”. Some of his most
important inventions include the hydraulic machine, the boat and design of a flying
machine.
EVANGELISTA TORRICELLI (1608-1647)
 The famous Italian physicist and mathematician is the author of the BAROMETER
(scientific tool used in the field of meteorology to estimate atmospheric pressure), built in
1643. It would be interesting to note that a number of Italian Navy submarines were
named after the inventor.
FERDINAND VERBIEST (1623 - 1688)
 Verbiest was an astronomer and a mathematician. He was the one to invent the
“WORLD'S FIRST AUTOMOBILE”. The inventor came up with the idea to create an
automobile while visiting China as a missionary. His automobile was powered by steam,
but could not carry humans.
KARL DRAIS (1785 - 1851)
 The German inventor is the author of the LAUFMASCHINE (German for "running
machine"), which was later called the “velocipede” and is believed to be the forerunner of
the bicycle and motorcycle. His invention represented the first means of transport to use
the two-wheeler principle and the starting point of the mechanized personal transport.
LOUIS PASTEUR (1822-1895)
 The famous French chemist and microbiologist was the one who has come up with the
process of PASTEURIZATION, a process that involves heating a food, usually liquid, to

20

Downloaded by Noime Burtanog (noimeburtanog@gmail.com)


lOMoARcPSD|38673565

a certain temperature for a specific length of time, and then cooling it instantly.
Pasteurization is used today to slow the development of microorganisms in food.
LEVI STRAUSS (1829 - 1902)
 Everyone knows that JEANS were invented in the United States, some know that Levi
Strauss was the author of this invention but few know that Strauss was an immigrant
from Germany managed to found in San Francisco, California the world's first firm to
manufacture blue jeans. The company called “Levi Strauss & Co.,” started its operation
back in 1853.
GOTTLIEB DAIMLER (1834 - 1900)
 Daimler is another representative of German intelligentsia, being an engineer, industrial
designer and industrialist. He was the first to develop INTERNAL-COMBUSTION
ENGINE and a pioneer in the field of automobile development.
 Daimler is the author of the “first high-speed petrol engine and the world's first four-
wheel vehicle”.
WILHELM CONRAD RÖNTGEN (1845 - 1923)
 The famous German physicist Röntgen is the one who discovered the X-RAYS (also
known as Röntgen rays).
 This invention allowed the German scientist to win the first Nobel Prize in Physics in
1901.
THOMAS EDISON (1847 - 1931)
 He is the author of a large number of inventions, but the most well-known one is the
ELECTRIC BULB. Among other discoveries of Thomas Edison there are telegraph
devices, phonograph, carbon transmitter, direct current generator, universal electric
motor, and more.
EMILE BERLINER (1851 - 1929)
 The German-born Jewish American scientist became known thanks to his DISC
RECORD GRAMOPHONE (in the United States known as phonograph or record
player). Used for recording and reproducing sounds on a gramophone record, vinyl
record, the device (with certain modifications made once in a while) was popular until
1980s.
ALEXANDER GRAHAM BELL (1857 - 1922)
 During the experiments he carried out with the telegraph, Bell came up with the idea of
the TELEPHONE.
 The inventor of one of the most popular devices today thought that the telephone was
intruding, which is why he did not have one in his workplace.
RUDLF CHRISTIAN KARL DIESEL (1858 - last seen alive 1913)
 Being a mechanical engineer, Rudolf Christian Karl Diesel managed to discover the
DIESEL ENGINE.
 The German inventor was also a well-known thermal engineer, a polyglot, an expert in
arts, and a social theorist
AMALIE AUGUSTE MELITTA BENTZ (1873 - 1950)
 She was born Amalie Auguste Melitta Liebscher. Bentz was a German business lady, who
invented the COFFEE FILTER back in 1908. Today her grandchildren, Thomas and
Stephen, are managing the family business (Melitta Group KG) based in Minden in
eastern North Rhine-Westphalia.
ALBERT EINSTEIN (1879 - 1955)
 One of the greatest scientists of the 20th century is the author of numerous inventions and
theories that transformed a lot of concepts linked to space and time, with the most
important discovery being the theory of relativity. Other discoveries of Einstein include
the PHOTOELECTRIC EFFECT and the EINSTEIN CALCULATOR.
SIR ALEXANDER FLEMING (1881 - 1955)
 During the World War I Fleming worked as an Army Medical Corp.
 He is the inventor of PENICILLIN that prevented a lot of soldiers from being infected.
 The discovery of penicillin managed to significantly boost the evolution of medicine
industry.
FRITZ PFLEUMER (1881 - 1945)
 The German-Austrian engineer is the author of the MAGNETIC TAPE used for
recording sound. Pfleumer decided to grant the right of use to the AEG, a German
manufacturer of electrical equipment. The event took place on December 1, 1932. Based
on Pfleumer's magnetic tape, the German firm created the world's first practical tape
recorder dubbed Magnetophon K1.
FREDERICK BANTING (1891 - 1941)

21

Downloaded by Noime Burtanog (noimeburtanog@gmail.com)


lOMoARcPSD|38673565

 Initially Banting was dedicated to politics but later decided to shift to medicine. In 1916
he completed his MD and during the World War I worked as a doctor. He was very
interested in diabetes and continuously worked on a cure for it. It would be worth noting
that the Banting searcher for cure for diabetes together with Dr. Charles Best. In 1923 the
researcher was awarded with the Nobel Prize for discovering INSULIN.
EDWIN HERBERT LAND (1909 - 1991)
 The co-founder of the Polaroid Corporation was the first who came up with low-cost
filters for polarizing light (useful system of in-camera instant photography). His most
popular invention, POLAROID INSTANT CAMERA, was officially launched in late
1948 and allowed users to take and develop a picture in just under 60 seconds.

ORGAN SYSTEMS OF THE BODY


 INTEGUMENTARY SYSTEM
 Forms the external body covering
 Composed of the skin, sweat glands, oil glands, hair, and nails
 Protects deep tissues from injury and synthesizes vitamin D
 SKELETAL SYSTEM
 Composed of bone, cartilage, and ligaments
 Protects and supports body organs
 Provides the framework for muscles
 Site of blood cell formation
 Stores minerals
 MUSCULAR SYSTEM
 Composed of muscles and tendons
 Allows manipulation of the environment, locomotion, and facial expression
 Maintains posture
 Produces heat
 NERVOUS SYSTEM
 Composed of the brain, spinal column, and nerves
 Is the fast-acting control system of the body
 Responds to stimuli by activating muscles and glands
 CARDIOVASCULAR SYSTEM
 Composed of the heart and blood vessels
 The heart pumps blood
 The blood vessels transport blood throughout the body
 LYMPHATIC SYSTEM
 Composed of red bone marrow, thymus, spleen, lymph nodes, and lymphatic vessels
 Picks up fluid leaked from blood vessels and returns it to blood
 Disposes of debris in the lymphatic stream
 Houses white blood cells involved with immunity
 RESPIRATORY SYSTEM
 Composed of the nasal cavity, pharynx, trachea, bronchi, and lungs
 Keeps blood supplied with oxygen and removes carbon dioxide
 DIGESTIVE SYSTEM
 Composed of the oral cavity, esophagus, stomach, small intestine, large intestine,
rectum, anus, and liver
 Breaks down food into absorbable units that enter the blood
 Eliminates indigestible foodstuffs as feces
 URINARY SYSTEM
 Composed of kidneys, ureters, urinary bladder, and urethra
 Eliminates nitrogenous wastes from the body
 Regulates water, electrolyte, and pH balance of the blood
 MALE REPRODUCTIVE SYSTEM
 Composed of prostate gland, penis, testes, scrotum, and ductus deferens
 Main function is the production of offspring
 Testes produce sperm and male sex hormones
 Ducts and glands deliver sperm to the female reproductive tract
 FEMALE REPRODUCTIVE SYSTEM
 Composed of mammary glands, ovaries, uterine tubes, uterus, and vagina
 Main function is the production of offspring
 Ovaries produce eggs and female sex hormones
 Remaining structures serve as sites for fertilization and development of the fetus

22

Downloaded by Noime Burtanog (noimeburtanog@gmail.com)


lOMoARcPSD|38673565

 Mammary glands produce milk to nourish the newborn

ASTRONOMY NOTES
 Largest circular storm in our solar system is found in JUPITER.
 The rapidly moving stream of charged particles that is being driven away from the sun is
known as SOLAR WIND.
 Biggest known asteroid is CERES.
 One Mercurian year is equal to 88 EARTH DAYS.
 Largest volcano in the solar system is OLYMPUS MONS found on Mars.
 One Jupiter day is equal to 9 HOURS 50 MINUTES.
 Between 1979 and 1998, the farthest planet from the sun was NEPTUNE.
 The SUNSPOT CYCLE is 11 years.
 The HERTZSPRUNG-RUSSEL DIAGRAM directly compares temperature and
luminosity of stars.
 The ANDROMEDA GALAXY is spiral.
 The MILKY WAY GALAXY is 100,000 light years across.
 The smallest planet in our solar system is PLUTO.
 The only planet with a moon almost as big and as heavy as the planet itself is PLUTO.
 NEIL ARMSTRONG landed on the moon on 1969.
 The only planet with a day longer than its year is VENUS.
 Pluto’s moon is called CHARON.
 The area between Saturn’s two rings is called CASSINI’S DIVISION.
 HELIOCENTRIC means around the sun; GEOCENTRIC means around the earth.
 Neptune’s moon TRITON has an ocean made of liquid Nitrogen.
 The first man to classify stars according to their brightness was HIPPARCHUS.
 The nearest star to the sun is PROXIMA CENTAURI, also known as ALPHA
CENTAURI.
 The greatest distance of a planet from the sun is called APHELION (A for Away). The
smallest distance of a planet from the sun is called PERIHELION (P for Proximal or
near).
 The main component of the atmosphere of Mars is CARBON DIOXIDE.
 JUPITER has a mass greater than the combined masses of all the planets.
 The summer solstice usually occurs on JUNE 21.
 The only two planets that do not have their moons or natural satellites are VENUS and
MERCURY.
 The constellation ANDROMEDA is also called the CHAINED MAIDEN.
 TAURUS is the brightest star in Aldebaran.
 A comet’s tail always points away from the sun.
 Spectral line splitting due to the influence of magnetic fields is called ZEEMAN
EFFECT.
 99.8 percent of asteroids have orbits between Mars and Jupiter.
 A RED GIANT has a low effective temperature (3000 to 4000 Kelvin) and a radius of
around 100 times the Sun’s.
 When three celestial bodies are in a line or collinear, it is called a SYZYGY.
 PLUTO has the most eccentric orbit around the sun.
 Beads of light visible around the rim of the moon at the beginning and end of a total solar
eclipse are called BAILY’S BEADS.
 The last planet Voyager II visited is NEPTUNE.
 Only 0.001% of the Sun’s mass has been converted into energy.

BRANCHES OF SCIENCE
 GEOLOGY: the science that deals with the earth's physical structure and substance, its
history, and the processes that act on it.
 METEOROLOGY: the branch of science concerned with the processes and phenomena
of the atmosphere, especially as a means of forecasting the weather.
 PALEONTOLOGY: the branch of science concerned with fossil animals and plants.
 OCEANOGRAPHY: the branch of science that deals with the physical and biological
properties and phenomena of the sea.
 VOLCANOLOGY: the scientific study of volcanoes.
 CHEMISTRY: the branch of science that deals with the identification of the substances
of which matter is composed; the investigation of their properties and the ways in which

23

Downloaded by Noime Burtanog (noimeburtanog@gmail.com)


lOMoARcPSD|38673565

they interact, combine, and change; and the use of these processes to form new
substances.
 PHYSICS: the branch of science concerned with the nature and properties of matter and
energy. The subject matter of physics, distinguished from that of chemistry and biology,
includes mechanics, heat, light and other radiation, sound, electricity, magnetism, and the
structure of atoms.
 THERMODYNAMICS: the branch of physical science that deals with the relations
between heat and other forms of energy (such as mechanical, electrical, or chemical
energy), and, by extension, of the relationships between all forms of energy.
 MECHANICS: the branch of applied mathematics dealing with motion and forces
producing motion.
 ASTRONOMY: the branch of science that deals with celestial objects, space, and the
physical universe as a whole.
 BIOLOGY: the study of living organisms, divided into many specialized fields that cover
their morphology, physiology, anatomy, behavior, origin, and distribution.
 ZOOLOGY: the scientific study of the behavior, structure, physiology, classification, and
distribution of animals.
 BOTANY: also called plant science(s), plant biology or phytology, is the science of plant
life and a branch of biology.
 GENETICS: the study of heredity and the variation of inherited characteristics.
 AGRICULTURE: the science or practice of farming, including cultivation of the soil for
the growing of crops and the rearing of animals to provide food, wool, and other
products.
 FORESTRY: the science or practice of planting, managing, and caring for forests.
 ENGINEERING: the branch of science and technology concerned with the design,
building, and use of engines, machines, and structures.
 METALLURGY: the branch of science and technology concerned with the properties of
metals and their production and purification.
 OPTICS: the scientific study of sight and the behavior of light, or the properties of
transmission and deflection of other forms of radiation.

BRANCHES OF SOCIAL SCIENCE


 ��������� — The study of social life, social change, and the social causes and
consequences of human behavior
 ������������ — the scientific study of humanity, concerned with human
behavior, human biology, cultures, societies, and linguistics, in both the present and past,
including past human species
 ���������� — the scientific study of the human mind and its functions,
especially those affecting behavior in a given context.
 ��������� — the study of places and the relationships between people and their
environments
 ������� — the study of change over time, past events, and it covers all aspects of
human society. Political, social, economic, scientific, technological, medical, cultural,
intellectual, religious, and military developments are all part of history.
 ���������� — the study of the fundamental nature of knowledge, reality, and
existence, especially when considered as an academic discipline.
 ��������� — is a social science concerned with the production, distribution, and
consumption of goods and services. It studies how individuals, businesses, governments,
and nations make choices about how to allocate resources.
 ���������� — the study of statistics such as births, deaths, income, or the
incidence of disease, simply the statistical study of human populations.
 ��������� ������� — the study of governments and political institutions. An
example of political science is the study of democracy. The systematic study of
government and politics.
 ��� — The law is a system of rules that a society or government develops in order to
deal with crime, business agreements, and social relationships. You can also use the law
to refer to the people who work in this system. Law is used to refer to a particular branch
of the law, such as criminal law or company law.
 ����������� — the study of the ancient and recent human past through material
remains. Archaeology analyzes the physical remains of the past in pursuit of a broad and
comprehensive understanding of human culture.

24

Downloaded by Noime Burtanog (noimeburtanog@gmail.com)


lOMoARcPSD|38673565

 ����������� — the study of language. Some linguists are theoretical linguists and
study the theory and ideas behind language, such as historical linguistics (the study of the
history of language, and how it has changed), or the study of how different groups of
people may use language differently (sociolinguistics).
 �������� — the study of religion. It examines the human experience of faith, and
how different people and cultures express it. Theologians examine the many different
religions of the world and their impact on society.
 ���������� — books and other written works, especially those considered to have
creative or artistic merit or lasting value. Books written by Charles Dickens are an
example of literature. Books written on a scientific subject are examples of scientific
literature.

UNFAMILIAR BRANCHES OF BIOLOGY


 ANATOMY – inner organs
 ENTOMOLOGY – insects
 BIOCHEMISTRY – chemical patterns of animals
 ECOLOGY – living things bet. Each other in environment
 EMBRYOLOGY – developmental patterns from zygote to birth
 GENETICS – heredity
 MYCOLOGY – fungi
 HERPETOLOGY – reptiles and amphibians
 HISTOLOGY – plant and animal tissues
 MORPHOLOGY – phenotype (appearance)
 ORNITHOLOGY – birds
 PARASITOLOGY – parasites
 PALEONTOLOGY – fossils of animals and plants
 PHYSIOLOGY – function of tissue, organ & system
 TAXONOMY – classification of living organisms

LABORATORY TOOLS AND SAFETY MEASURES


 BEAKER: hold and heat liquids
 REAGENT BOTTLE: storage, mixing and displaying
 BUNSEN BURNER: heating and exposing items to flame
 BURET: chemical analysis; measure precisely how much liquid is used in titration
 CRUCIBLE: heat small quantities of substances in very high temperatures
 ERLENMEYER FLASK: heat and store liquids; bottom is wider than the top
 EVAPORATING DISH: heat and evaporate liquids
 FLORENCE FLASK: heat substances that need to be heated evenly; mostly used in
distillation experiments
 FUNNEL: transfer liquids into another container to avoid spillage
 MICROSPATULA: transfer small amounts of solid from one container to another
 MORTAR AND PESTLE: crush solids into powders
 PIPET: move small amounts of liquid from one container to another
 RING STAND: used to items being heated; used with clamps or rings
 STIRRING ROD/ GLASS ROD: stir solutions to dissolve substances
 STOPPER/CORK: seal test tubes and flasks
 TEST TUBE BRUSH: easily clean the inside of a test tube
 TEST TUBE HOLDER: hold test tubes while being heated
 TEST TUBE RACK: hold test tubes upright while observing reactions
 THERMOMETER: take temperature
 TONGS: hold flasks, crucibles and evaporating dish when they are hot
 CLAY TRIANGLE: used to hold crucibles when they are being heated
 VOLUMETRIC FLASK: measure one specific volume; used in preparing solutions

SKYPE – is a face-to-face communication


RAFAEL PALMA – first-up president
LEORA FLORENTINO – First Filipino Poetess
PANDAY PINA – First Filipino Canon Maker
PEDRO PATERNO – Peacemaker of the Revolution
ISABELO DELOS REYES – Founder of Philippine Socialism
ESPIFANIO DELOS SANTOS – the Man of many talents
ANFDRES BONIFACIO – the great Plebian

25

Downloaded by Noime Burtanog (noimeburtanog@gmail.com)


lOMoARcPSD|38673565

FELIPE AGONCILLO – outstanding diplomat of the first Philippine Republic


MEMORIA FOTOGRAFICA – Jose Ma. Panganiban

BLACK SHIP – Idiom


PHANTOM OF DELIGHT – Metaphor
TINKLE 3x – Onomatopoeia
ELLIPSIS – tatlong tuldok na sunod – sunod
KUWIT – naghihiwlaay sa mga salita
PANUKOL – pangatnig na nagbubukod
ANTARTICA or SAHARA – largest desert
IMPROMPTU – ang paksa ay ibinibigay bago magsimula ang talumpati
FRAILOCRACIA – term of Marcelo H. Del Pilar to friars
ARENA THEATER – Severino Montano
CARBON DIOXIDE – released by plants at night
SLA THEATER – Roger Nate Crane or Naty Crane Rogers
RECTANGLE – shape of a brick
CILIA – hair-like structure
VILLI – finger-like structure
EPIGLOTTIS – flap-like structure
FLAGELLA – whip-like structure
EPISTOLARY – novels in letters
CALAMBA, LAGUNA – birthplace of Rizal
ALEXANDER FLEMING – penicillin discoverer
FRAY JUAN de PLASENCIA – author of Doctrina Christiana
AIR – light travels fastest
JONES – Law of Bicameralism
REDUCCION – pagbabagong anyo ng bayan
PERMUTATION 9 P9 – 362, 880
ABSOLUTE MEAN – 14.35 or 14.5
BLOOD COMPACT (SANDUGO) – agreement between Legazpi and Datu Sikatuna
HINDUISM – oldest religion in Asia
MATINDING HIRAP SA PAGKABINAT/ MATINDING LAGNAT – ikinamatay ni Donya Pia
LOVE FEELINGS – is more qualitative
GULF – is a sea partly enclosed by land
ZONE – is the surface between tropic of cancer and arctic cancer
COMBINE CHITINS AND GLYCOLS – is a feature of fungi
SERGIO OSMENA – is the speaker of the Philippines Assembly
AQUILINO PIMENTEL – is the father of Local Government Code
ERRONEOUS DATA – is not related to ethics in research
CITY STATES – is a single political unit
TALUMPATI – binibigkas sa harap ng madla
LEECHES – facilitate anticoagulation
PEDRO PELAEZ – led the native secular clergy
RECTANGLE – opposite sides are equal
CONSTITUTIONAL MONARCHY – government of England
DOUGLAS MCARTHUR – is the Field Marshall of the Philippines Army
VETO POWER – is the power to set aside a certain grant
PHILIPPINES lies at the Pacific Ring of Fire
HURRICANE – storm in Atlantic region
MICROFILAMENT – thin structure in cell
METALLOIDS – display properties of both metal and non-metals
BRIGHT GREEN – in a flame test indicate the presence of boron
PHYLOGENETICS – describes the evolutionary sequence of organisms
MICROBIAL CULTURE – allows microorganisms to multiply in medium
The Philippines is much hotter than Canada because the Philippines receives sun rays closer to
vertical position.
I enjoy painting but I would not like doing it all my life.
ABASE – means denoted
AMORPITOUS – means shapeless.
PERSIFLAGE – is to praise glowingly
DEXTERITY – manual skill

26

Downloaded by Noime Burtanog (noimeburtanog@gmail.com)


lOMoARcPSD|38673565

WORD PROCESSING – task oriented software that lets the user create, edit, format, store and
print the graphics in one document
INCOME TAX – is the principal kind of tax that directly affects the Filipino workers
JONES PREFERS FRIES AND SUNDAES – shows that culture is learned in a foreign context
JONES SAY PO AND HO TO ELDERS – shows that culture is learned in a Filipino context
GRAPHICS – illustrations/ maps
Leaves with green color look green in the sunlight because they reflect green light
BACTERIA – example of unicellular organisms
AGRICULTURE – crops/ foods
PRODIGY – OFFSPRING: two words lack correlation
Dr. Jose Rizal’s execution was the greatest blunder committed by Spanish authorities because it
made him a martyr of freedom
KATALINUHAN – basal o di-kongreto
HOME/ COMPUTERS – used at write letters, filed records and picture, assignment, send social
messages, play games etc.
GNP – elements of the economy need to be balanced so that government can pay foreign and
domestic debts
The Father of the family was not DESPOTIC.
Folkdance from Mexico except POLKA.
Because native surnames proved to be confusing and CRUDE to Spanish
Everything seems to be all right.
In the 19th century the new middle-class spring. The leaders of the propaganda which sowed
seeds to the Philippines.
SURVIVAL – reproduction is necessary
CHRISTIANITY – greatest
ATOM – smallest particle
HIGH SCHOOL/ SECONDARY EDUCATION – Spanish educational degree of 1863
VOLUME – is the softness or loudness of sounds
PITCH – is the highness of lowness of sounds
SOCRATIC METHOD – is question and answer
Iron is FERROMAGNETIC
CHRISTIANITY – is the Spanish greatest legacy
RESIDENCE CERTIFICATE – cedula during Spanish occupation
BUDABIL – is a play NOT from the Spanish Era
PHILIPPINES NATIONAL ANTHEM – composed by Juan Felipe
The real activity of Science is the INTERPRETATION of observations and data
PAGSULONG
Kasingkahulugan: Pag-unlad
Kasalungat: Pagbagsak
ACTA DE TEJEROS – rejected Aguinaldo as president
To show phonological awareness skill in SEGMENTING student say the word and they say the
syllable or sound.
To show phonological awareness skill in BLENDING student say the sounds individually then
say them fast while the teacher combine blocks and letters.
FRANCISCO DAGOHOY – is the leader of the longest revolt
VIBRIO CHOLERA – is comma-slapped bacteria
FOLKLORE – is the philosophy of our folks during the Pre-Hispanic days
NON-RENEWABLE RESOURCES – run out of the production of fossil fuels, petroleum and
coal
FORMALLY – CONVENTIONALLY – two words lack correlation
FORMERLY – HERE TO FORE – two words lack correlation
CHECK AND BALANCE – is the principle behind the power given the legislature to impeach
the president for the loss of public trust
GESTURES – culturally de fires
HABIT – people communicate
PAGLUSTAY – ang katiwaliang ginagawa ng opisyal ng gobyerno na paggamit ng pera ng
bayan para sa pansariling kapakanan
SODIUM CHLORIDE in water is an example of solid-liquid solution

����������� ���������� (1-���)


27

Downloaded by Noime Burtanog (noimeburtanog@gmail.com)


lOMoARcPSD|38673565

1. What is the origin of the word Education?


A. Word 'Educate'
B. Edu and 'Catum'
C. ‘E’ and ‘Catum’
D. None of these
2. Which of the following statements is correct?
A. Education is an art
B. Education is a science
C. It is neither an art nor science
D. To some extent it is art and to some extent it is science
3. What is called education acquired without any specific purpose, fixed period and place?
A. Formal Education
B. Informal Education
C. Indirect Education
D. Individual Education
4. Which one of the following sentences is correct about the nature of teaching?
A. It is remedial
B. It is diagnostic
C. It is diagnostic as well as remedial
D. All the above statements are correct
5. What is the compulsory element of learning?
A. Tendency to know
B. Ability to read
C. Bright Mind
D. None of these
6. What is the place of principal in an educational institute?
A. Owner of the school
B. Founder of the school
C. Manager of the school
D. Overall head of the school
7. If a student failed in any class what should be done to him?
A. He should be kept in the same class
B. He should be advised to leave studies
C. He should be given a chance to improve and sent to the next class after he improves
D. All the above methods are right
8. Why are curriculum activities used in teaching?
A. To assist the teacher
B. Make teaching easy
C. To make teaching attractive
D. To make teaching interesting, easy to understand and
9. What are the three components of the educational process?
A. Direction, instruction and skill
B. Teaching, learning and practice

28

Downloaded by Noime Burtanog (noimeburtanog@gmail.com)


lOMoARcPSD|38673565

C. Teacher, student and education


D. Education, teacher and books
10. What is teaching through deductive method?
A. From easy to difficult
B. From macro to micro
C. From general to specific
D. From specific to general
11. What is the main centre of informal Education?
A. Family
B. Society
C. Radio and Television
D. All of the above
12. Which is the first school for a child’s education?
A. School
B. Family
C. Friends
D. Society
13. Which one of the following education systems supports scientific progress?
A. Naturalistic Education
B. Idealistic Education
C. Realistic Education
D. None of these
14. What is the meaning of lesson plan?
A. To read the lesson before teaching it
B. To prepare the list of questions to be asked
C. To prepare all that the teacher wants to teach in a limited period
D. To prepare detailed answers of all the questions to be
15. On what depends the values of an educational experience in the eyes of the idealist?
A. The extent to which it satisfies pupil desires
B. The manner in which it affects future experience
C. Whether or not it preserves accepted institutions
D. Whether or not the pupil has been properly motivated
16. Which educational activity is most desirable to the pragmatist?
A. That is beneficial effect upon the future experiences of the pupil
B. Approximates the goals which educational scientists have set up
C. Results from the indiscrimination of the pupil in democratic theory
D. That characterizes by spontaneous, active, continuously pleasurable and practical for
the pupil
17. What is the view point of progressive educators regarding the issue of liberal vs. vocational
education?
A. Vocational ends load one to degrade learning
B. All subjects should have a vocational orientation
C. Liberal arts subject should proceed vocational training
D. Vocational and liberal education should not be separated

29

Downloaded by Noime Burtanog (noimeburtanog@gmail.com)


lOMoARcPSD|38673565

18. Who was the supporter of Naturalism in Education?


A. Froebel
B. Rosseau
C. Armstrong
D. John Locke
19. What do you mean by curriculum?
A. Sum total of the annual study
B. A child learns through curriculum
C. Sum total of the activities of a school
D. Indicates the course to be taught by the teachers to the students throughout the year
20. Which system of education was propounded by Mahatma Gandhi?
A. Teaching through listening, meditation etc.
B. Teaching through music
C. Teaching by activities
D. All of these
21. Who raised the slogan “Back to Nature”?
A. Realism
B. Naturalism
C. Pragmatism
D. Existentialism
22. Which statement is not correct about Naturalism?
A. A reaction against a mere study of books and linguistic forms
B. A reaction against the degenerated humanism of the Renaissance period
C. A reaction against sophistication, artificiality and paraphernalia in education
D. None of these
23. Who said, “Reverse the usual practice and you will almost always do right” ?
A. Plato
B. Dewey
C. Rousseau
D. Mahatma Gandhi
24. “Human institutions are one mass of folly and contradiction.” Whose statement is this?
A. Dewey
B. Rousseau
C. Bernard Shaw
D. Ravinder Nath Tagore
25. According to which school of philosophy of education, exaltation of individual’s personality
is a function of education?
A. Marxism
B. Idealism
C. Idealism and Marxism both
D. Pragmatism
26. Which is not Naturalism’s aim of Education?
A. To inculcate ethical and moral values in the pupils
B. Education is the notion of man’s evolution from lower forms of life

30

Downloaded by Noime Burtanog (noimeburtanog@gmail.com)


lOMoARcPSD|38673565

C. To equip the individual or the nation for the struggle for existence so as to ensure
survival
D. To help the pupils to learn to be in harmony with and well-adapted to their
surroundings
27. Which school held the view, “God makes all things good; man meddles with and they
become evil”?
A. Marxism
B. Naturalism
C. Pragmatism
D. Existentialism
28. Which school-maintained self-expression with the accompanying cries of “no interference”,
“no restraints”?
A. Truest form of Naturalism
B. Extreme form of Naturalism
C. Most valid form of Naturalism
D. Most widely accepted form of Naturalism
29. Which is not the nature of philosophy?
A. It is a science of knowledge
B. It is the totality of man’s creative ideas
C. It is a planned attempt on search for the truth
D. It is a collective ensemble of various viewpoints
30. Which branch of philosophy deals with knowledge, its structure, method and validity?
A. Logic
B. Aesthetics
C. Metaphysics
D. Epistemology
31. Which school maintained: “Natural impulses of the child are of great importance and are
good in themselves”?
A. Romantic Naturalism
B. Biological Naturalism
C. Mechanical Naturalism
D. Naturalism of physical science
32. Which branch of philosophy examines issues pertaining to the nature of “reality” ?
A. Axiology
B. Ontology
C. Metaphysics
D. Epistemology
33. On what is based the need for teaching philosophy of education?
A. All pupils are not alike
B. Different ways of teaching-learning
C. Different systems of education found in different countries
D. Different philosophies expressed different points of view on every aspect of education
34. What is the goal of education according to Idealism?
A. Realization of moral values
B. Satisfaction of human wants

31

Downloaded by Noime Burtanog (noimeburtanog@gmail.com)


lOMoARcPSD|38673565

C. Perfect adaptation to the environment


D. Cultivation of dynamic, adaptable mind which will be resourceful and enterprising in
all situations
35. The aim of education according to the Existentialists is
A. Objective knowledge
B. Adaptation to practical life
C. A good understanding of the world outside
D. Humanitarian and humanist self- realization
36. The Realist’s aim of education is
A. Self-realization
B. Happy and moral development
C. Spiritual and moral development
D. Total development of personality
37. Naturalist’s conception of man is
A. Man’s very essence of being is his spiritual nature
B. It is spirit rather than animality that is most truly man
C. Nature would have them children before they are men
D. There exists in the nature of things a perfect pattern of each individual
38. Which philosophy of education considers psychology as an incomplete study of and an
inadequate basis of educational theory?
A. Realism
B. Idealism
C. Naturalism
D. Pragmatism
39. Which among the following does not fit into the scheme of educational goals of the Idealists?
A. Skills
B. Care of body
C. Moral values
D. Self-expression
40. Religious education is strongly advocated by
A. Realist
B. Idealists
C. Pragmatists
D. Existentialists
41. Which of the following is said about the idealists?
A. They like “roses”
B. They are content with “briars”
C. They want “roses” and “briars” both
D. They are satisfied neither with “briars” nor with “roses”
42. Which school of philosophy of education advocated Project method of teaching?
A. Idealism
B. Realism
C. Naturalism
D. Pragmatism

32

Downloaded by Noime Burtanog (noimeburtanog@gmail.com)


lOMoARcPSD|38673565

43. Play way method of teaching has been emphasized in the scheme of the education of
A. Realists
B. Naturalists
C. Pragmatists
D. Existentialists
44. Which is the most widely accepted method of education, according to the pragmatists?
A. Heuristic method
B. Learning by doing
C. Lecturing by the teacher
D. Leaving the child free to learn
45. The pragmatists are against
A. The specialist teachers
B. Eternal spiritual values
C. The external examinations
D. Breakdown of knowledge into separate subjects
46. Pragmatism has a greater sense of responsibility than Naturalism with regard to moral
training because
A. They emphasize teaching of values
B. They consider education, basically, a social process
C. They do not want the teacher to abdicate from the scene
D. The free activity which pragmatic- system of education entails does not mean licence;
rather it means a guided activity
47. Which of the following claims of the pragmatists is not acceptable?
A. Training in character through school’s co-curricular activities is possible
B. Training in citizenship is possible through school and community activities
C. Child’s own experience is valuable for adequate development of child’s personality
D. The free activity of the pupil is likely to result in permanent attitudes of initiative and
independence and moral discipline
48. Project method of teaching is an outstanding contribution of
A. Idealism
B. Realism
C. Naturalism
D. Pragmatism
49. Which is the characteristic of the project method?
A. Problematic act
B. A voluntary undertaking
C. Carried in its natural setting
D. Used for all-round-development of child’s personality
50. Which among the following is not essentially desirable in the project method?
A. The task of the project should be full of message for the children
B. The task of the project is as real as the task of the life outside the walls of the school
C. The task of the project involves constructive effort or thought yielding objective
results
D. The task of the project should be interesting enough so that the pupil is genuinely
eager to carry it out

33

Downloaded by Noime Burtanog (noimeburtanog@gmail.com)


lOMoARcPSD|38673565

51. Which is a great disadvantage of the project method?


A. Children are generally not interested in it
B. It consumes much of the time of the child
C. It leaves gaps in the knowledge of the child
D. Teachers, generally, do not like to teach through it
52. Learning by Project Method is technically known as
A. Efficient learning
B. Adequate learning
C. Incidental learning
D. Systematic learning
53. Education, according to the Pragmatist is
A. Wholly purposive
B. Wholly pupil-oriented
C. Wholly society-oriented
D. Wholly interdisciplinary
54. Who among the following is not a follower of Pragmatic Philosophy?
A. Kilpatrick
B. Peshtalozzi
C. John Dewey
D. William James
55. What is not associated with Pragmatism?
A. Purposive education
B. Freedom-based education
C. Education for self-realization
D. Experience-based education
56. Who emphasized realization of Truth, Beauty and Goodness as the aims of education?
A. Realists
B. Idealists
C. Naturalists
D. Pragmatists
57. Which statement about truth is not correct according to the philosophy of Pragmatism?
A. It is eternal
B. It is made by man
C. It is ever changing
D. It is what emerges to be true in actual practice
58. In whose methodology of teaching “Experimentation” is the key-note of?
A. Realism
B. Idealism
C. Pragmatism
D. Existentialism
59. The term “progressive education” related to
A. Realism
B. Idealism

34

Downloaded by Noime Burtanog (noimeburtanog@gmail.com)


lOMoARcPSD|38673565

C. Pragmatism
D. Existentialism
60. Who said, “No fixed aims of education and no values in advance”?
A. Realists
B. Idealists
C. Marxists
D. Progressive educators
61. Which school of philosophy of education stresses the direct study of men and things through
tours and travels?
A. Idealism
B. Marxism
C. Social realism
D. Existentialism
62. Which school believes that all knowledge comes through the senses?
A. Idealism
B. Pragmatism
C. Existentialism
D. Sense Realism
63. Which school raised the slogan “Things as they are and as they are likely to be encountered
in life rather than words”?
A. Realists
B. Idealists
C. Pragmatist
D. Existentialists
64. As Huxley pleaded for the introduction of “a complete and thorough scientific culture” into
schools, he is claimed to be
A. A Realist
B. An Idealist
C. A Naturalist
D. A Pragmatist
65. Realism in education was born out of
A. The enthusiasm of the Renaissance
B. The great religious movement of the 17th century
C. The degeneration of humanism after Renaissance
D. A cleavage between the work of the schools and the life of the world outside that
occurred during the 19th century
66. Which of the following is not criticized by realism in education?
A. Teaching which drifts away from life of the child
B. Teachers denying the value of school co-curricular activities
C. Pupils cramming for knowledge from books for reproducing in examination
D. Organizing schools in a way that is conducive to practical training in citizenship
67. In the light of relevant past events, contemporary events and their understanding should find
a place in the teaching of history. Who maintained this principle?
A. Realist
B. Marxists

35

Downloaded by Noime Burtanog (noimeburtanog@gmail.com)


lOMoARcPSD|38673565

C. Idealists
D. Naturalists
68. The most important thing to keep in mind for a teacher according to Realism in education is
A. The nature of the child
B. The method of teaching
C. Organization of the content to be taught
D. The value and significance of what is taught
69. Which school of philosophy very strongly advocates that education should be vocational in
character?
A. Realism
B. Naturalism
C. Pragmatism
D. Existentialism
70. Which is not an aspect of mind according to the Realists’ theory of knowing?
A. Behaviour
B. Awareness
C. Consciousness
D. Processing of awareness
71. Who believe that “Objects have a reality independent of mental phenomena”?
A. Idealists
B. Realists
C. Naturalists
D. Existentialists
72. Marxist educational philosophy is closer to
A. Realism
B. Idealism
C. Naturalism
D. Pragmatism
73. Which among the following statements is not a characteristic of Marxism?
A. It asserts that physical environment can definitely change the nature of the child
B. Its major objective is the development of child’s personality
C. Its educational philosophy is essentially materialistic
D. It presupposes a reality independent of man’s mind
74. Which school of philosophy of education regrets dualism between cultural, and vocational
curriculum?
A. Idealism
B. Marxism
C. Naturalism
D. Existentialism
75. According to which educational philosophy, socially useful labour must form the central
pivot of the entire school?
A. Marxism
B. Idealism
C. Naturalism

36

Downloaded by Noime Burtanog (noimeburtanog@gmail.com)


lOMoARcPSD|38673565

D. Existentialism
76. Which of the following has been asserted about schools by Marxist educational philosophy?
A. They should not be mere weapons in the hands of the ruling class
B. They should function as deliberate instruments of state policy
C. They should disinterestedly serve society as a whole
D. They should stand above politics
77. Which of the following characteristics is common to Pragmatism, Naturalism and
Existentialism?
A. Emphasis on the individual
B. Emphasis on value education
C. Emphasis on physical environment
D. Emphasis on spiritual aims of education
78. Whose is the ultimate concern- “What is existence”?
A. Idealists only
B. Existentialists only
C. Both Existentialists and Idealists
D. Realists only
79. Which of the following philosophies held that “Men in the world feel lonely and anxious,
being unsure of their meaning and fearful of their annihilation”?
A. Marxism
B. Idealism
C. Pragmatism
D. Existentialism
80. According to Existentialists, the essence of existence means
A. Unity with the ultimate reality
B. Spiritual good and happiness
C. Continuous growth and development
D. Tensions and contradictions which condition loneliness and anxiety
81. Who was the nineteenth century founder of Existentialism?
A. Hegel
B. Rousseau
C. D.J. O’Connor
D. Soren Kierkegaard
82. Who was twentieth century Existentialist?
A. Hegel
B. D.J. O’Connor
C. Jean Paul Sartre
D. Soren Kierkegaard
83. Which of the following is more generally acceptable by modern educationists?
A. Contribution to the welfare of the society should be the only aim of education
B. There should be one single aim of education unchangeable over time and space
C. There is one grand objective of education; and that is the development of the inner
nature of the child
D. Education is bound to have several aims since its concerns are several such as the
individual, the society, the family, the nation and so on

37

Downloaded by Noime Burtanog (noimeburtanog@gmail.com)


lOMoARcPSD|38673565

84. What is development of human potentialities in education?


A. Social aim
B. Specific aim
C. Individual aim
D. Individual as well as social aim
85. What is development of social sense and cooperation among the individuals through
education?
A. Social aim
B. Individual aim
C. National aim
D. Constitutional aim
86. Which among the following is not an acceptable criticism of social aims of education?
A. They are anti-individual
B. Man, in them, becomes only a means to an end
C. They hinder the growth and development of art and literature
D. They are un-psychological as they do not take into account the capacities and
interests of the individual
87. Which among the following is not emphasized by the individual aims of education?
A. Self-expression
B. Individual freedom
C. Development of inner potentialities
D. Development of values of tolerance and non-violence
88. Which of the following statements does not go in favour of the individual aims of education?
A. The society is strong if the individual is strong
B. Society is supreme and all individuals are only parts of it
C. Every individual is unique; development of his potentialities is essential
D. The individual is an asset to the society; his development and growth are necessary
89. Which among the following is the most correct view about social and individual aims of
education?
A. Social aims should be preferred to individual aims
B. Individual aims are implied in the social aims of education
C. Individual aims should be given preference to social aims
D. Individual aims and social aims are only two sides of the same coin
90. Which statement is most acceptable to the academicians about “Bread and butter aim” of
education?
A. It is only partly acceptable
B. It is important for only a section of the society
C. It is equally important along with other aims of education
D. It is the most important aim and should be given top priority by educationists
91. Which of the following does not pertain to intellectual development aim of education?
A. Spiritual development
B. Cultivation of intelligence
C. Training and “formation” of mind
D. Development of cognitive powers
92. Preparing the child for future life as an aim of education is preparing child for

38

Downloaded by Noime Burtanog (noimeburtanog@gmail.com)


lOMoARcPSD|38673565

A. Some suitable vocation


B. A happy married life
C. Some particular course of study
D. Facing all kinds of emergencies and situations of future life
93. The most effective method of character- formation is
A. Teaching by high character teachers
B. Teaching virtues through religious books
C. Organizing specialists’ lectures on importance of values in life
D. Rewarding virtuous behaviors and presenting high character models in the schools
94. Harmonious development of the child aim of education means
A. The state is above the individual citizen
B. The state is an idealized metaphysical entity
C. The state has to give not to take anything from the individual
D. The state is superior to the individual transcending all his desires and aspirations
95. Rigid system of state-education is justified on the basis that the state
A. Has better resources to manage education
B. Is supreme to dictate what shall be taught and how shall be taught
C. Has absolute control over the lives, and destinies of its individual members
D. Has a right and a bounden duty to mould the citizen to a pattern which makes for its
own preservation and enhancement
96. Social aims of education imply the training of
A. Individuals according to their needs
B. The individuals according to the facilities
C. The individuals according to their capacities
D. The individuals for the purpose of serving the needs of the society
97. What does the individual aim of education imply?
A. It should have more and more institutions every year
B. It should be by and large the concern of the private sector
C. It must contribute to the peace and happiness of the whole society
D. Education must secure for everyone the conditions under which the individuality is
most completely developed
98. According to which philosophy of education, childhood is something desirable for its own
sake and children should be children?
A. Realism
B. Idealism
C. Naturalism
D. Pragmatism
99. Who emphasized that education should be a social process?
A. Dewey
B. Rousseau
C. Pestalozzi
D. Vivekananda
100. The social aims of education imply that
A. The state is above the individual citizen

39

Downloaded by Noime Burtanog (noimeburtanog@gmail.com)


lOMoARcPSD|38673565

B. The state is an idealized metaphysical entity


C. The state has to give not to take anything from the individual
D. The state is superior to the individual transcending all his desires and aspirations

PROF. ED LET Reviewer


150 Items with Answer Key
1. At the end of periodical examination, Teacher R administered a summative test in Filipino.
After scoring the test papers she assigned grades to each test score such as 95, 90, 85, 80 etc.
What process did Teacher R use?
A. Ranking
B. Computation
C. Measurement
D. Evaluation
2. It is the study of man’s prehistory through the buried remains of ancient culture, skeletal
remnants of human beings.
A. Anthropology
B. Archeology
C. Ethnology
D. Ethnography
3. Teacher D claims: "If I have to give reinforcement, it has to be given immediately after the
response." Which theory supports Teacher D?
A. operant conditioning theory
B. social cognitive theory
C. cognitive theory
D. humanist theory
4. Visual imagery helps people store information in their memory more effectively. Which is
one teaching implication of this principle?
A. You will not object when your students daydream in class.
B. Instruct students to take notes while you lecture.
C. Encourage your students to imagine the characters and situations when reading a
story.
D. Tell them to read more illustrated comics.
5. Which is essential in the cognitive development of a person according to Vygotsky?
A. independent thinking
B. social interaction
C. individual mental work
D. scientific thinking
6. The design of the 2002 Basic Education Curriculum (BEC) is based on the principles that the
main sources for contemporary basic education are the expert systems of knowledge and the
learner's experience in their context. This shows that the BEC is _____ in orientation.
I. constructivist
II. behaviorist
III. essentialist
A. I and III
B. III only
C. I only
D. I, II, and III
7. Which assessment tool shows evidence of student’s writing skills?
A. project
B. portfolio
C. critiquing sessions
D. daily journal
8. Ask to do a learning task, Joe hesitates and says "Mahirap. Ayaw ko. 'Di ko kaya!" (It's
difficult. I don't like it. I can't do it.) To which problem does the case of the student allude?
I. Unmotivated students
II. Uncaring teachers
III. Extremely difficult learning tasks
IV. incompetent teachers
A. I and III
B. I and II
C. II and III

40

Downloaded by Noime Burtanog (noimeburtanog@gmail.com)


lOMoARcPSD|38673565

D. I, II, and III


9. Teacher S wants to determine immediately the learning difficulties of her students. Which of
the following do you expect her to undertake?
A. Require her students to prepare a portfolio.
B. Administer an achievement test.
C. Administer a diagnostic test.
D. Interview her students directly.
10. NSAT and NEAT results are interpreted against set mastery level. This means that NSAT and
NEAT fall under __________.
A. intelligence test
B. aptitude test
C. criterion-referenced test
D. norm-referenced test
11. The result of the item analysis showed that item no. 4 has a discrimination index of 0.67.
What characteristic could be true about this item?
A. Difficult
B. Valid
C. Easy
D. Average
12. The theme of Vygotsky's socio-cultural theory emphasizes the role of appropriate assistance
given by the teacher to accomplish a task. Such help enables the child to move from the zone
of actual development to a zone of proximal development. Such assistance is termed _____.
A. competency technique
B. scaffolding
C. active participation
D. collaboration
13. Which of these activities is not appealing to the bodily-kinesthetic learners?
A. making math moves
B. doing simple calisthenics
C. sketching/illustrating events
D. joining extramural events
14. Institutions of learning are required to meet the minimum standards for state recognition but
are encouraged to set higher standards of quality over and above the minimum through _____
as provided in Educational Act of 1982.
A. lifelong education
B. voluntary accreditation
C. formal education
D. academic freedom
15. You intend to assess affective attributes such as capacity to feel, attitudes, and behavior.
Which of the following should you establish to ascertain the instrument's validity?
A. construct
B. content
C. criterion-related
D. face
16. Which will be the most authentic assessment tool for an instructional objective on working
with and relating to people?
A. conducting mock election
B. home visitation
C. organizing a community project
D. writing articles on working and relating to people
17. As an effective classroom manager, what should a teacher do?
I. She uses instructional time wisely.
II. She uses her power to punish students for the sake of discipline.
III. She puts to use the available and appropriate materials.
IV. She manipulates colleagues and students so she can meet her goals.
A. I and III
B. II, III, and IV
C. I, II, and III
D. I, II, III, and IV
18. Keeping track of assessment results from one periodic rating to the next is useful in
contributing to the development of a _____.
A. regional plan

41

Downloaded by Noime Burtanog (noimeburtanog@gmail.com)


lOMoARcPSD|38673565

B. annual implementation plan


C. school improvement plan
D. division plan
19. In the K W L technique K stands for what the pupil already knows, W for he wants to know
and L for what he _____.
A. failed to learn
B. he likes to learn
C. needs to learn
D. learned
20. Which is one characteristic of an effective classroom management?
A. It quickly and unobtrusively redirects misbehavior once it occurs.
B. It teaches dependence on others for self-control.
C. It respects cultural norms of a limited group students.
D. Strategies are simple enough to be used consistently.
21. When curriculum content is fairly distributed in each area of discipline, this means that the
curriculum is _____.
A. sequenced
B. balanced
C. integrated
D. continued
22. Direct instruction is for facts, rules, and actions as indirect instruction is for _____.
A. hypotheses, verified data, and conclusions
B. concepts, patterns, and abstractions
C. concepts, processes, and generalizations
D. guesses, data, and conclusions
23. The difficulty index of a test item is 1.0. This means that the test is _____.
A. a quality item
B. very difficult
C. very easy
D. missed by everybody
24. The discrimination index of a test item is -0.35. What does this mean?
A. More from the upper group got the item correctly.
B. More from the lower group got the item correctly.
C. The test is quite reliable.
D. The test item is valid.
25. His aim of education is individual not a preparation for but participation in the life around the
individual.
A. Froebel
B. Spencer
C. Herbart
D. Pestalozzi
26. The following are characteristics of a child-friendly school except for _____.
A. exclusive
B. child-centered
C. gender-sensitive
D. non-discriminating
27. To make the lesson meaningful, systematic and motivating, teachers’ example should be
______.
A. based on higher level skills
B. interesting aided with illustrations
C. easy, simple, and understandable
D. relevant to students’ experience and knowledge
28. Which are the most important concerns about the use of ICT in instruction?
I. Developing appropriate curriculum materials that allow students to construct meaning and
develop knowledge throught the use of ICT
II. Devising strategies to meaningfully integrate technology into the curriculum
III. Using pedagogical skills related to technology
IV. Providing teachers with skills for using software applications
A. I and III
B. II and III
C. III and IV
D. I, II, III, and IV

42

Downloaded by Noime Burtanog (noimeburtanog@gmail.com)


lOMoARcPSD|38673565

29. Which of these non-threatening means of assessing learning outcomes?


I. Portfolio
II. Self-evaluation
III. Peer evaluation
IV. Learning journals
A. I and II
B. I and III
C. I, II, and IV
D. II and III
30. Teacher A's lesson is about the parts of the gumamela. He asked his pupils per group to bring
a real flower to study the different parts. After the group work labeling each part, the teacher
gave a test. What would be the best type of test he can give?
A. essay type
B. matching type
C. diary
D. journal
31. Principal A wants her teachers to be constructivist in their teaching orientation. On which
assumption/s is the principal's action anchored?
I. Students learn by personally constructing meaning of what is taught.
II. Students construct and reconstruct meanings based on experiences.
III. Students derive meaning from the meaning that teacher gives.
A. I and III
B. I only
C. I and II
D. II only
32. The norms in a school culture are centered on the _____.
A. learner
B. teacher
C. principal
D. supervisor
33. The difficulty index of a test item is 1.0. What does this mean?
A. The test item is very good, so retain it.
B. The test item is very difficult.
C. The test item is extremely easy.
D. The test item is not valid.
34. Someone wrote: "Environment relates to the profound relationship between matter, nature,
and society, and in such a context, ICTs bring new ways of living in a more interconnected
society, all of which reduces our dependency on matter and affects our relationship with
nature." What does this convey?
A. Environment and ICT are poles apart.
B. ICT impacts on environment.
C. Environment affects ICT.
D. ICT brings us away from an interconnected society.
35. Educational institutions’ effort of developing work skills inside the school are aimed at
_____.
A. developing moral character
B. inculcating love of country
C. teaching the duties of citizenship
D. developing vocational efficiency
36. Teacher B discovered that her pupils are weak in comprehension. To further determine in
which particular skills her pupils are weak which test should Teacher B give?
A. Aptitude Test
B. Placement Test
C. Diagnostic Test
D. Standardized Test
37. That the quality of Philippine education is declining was the result of a study by EDCOM
which recommended to _____ teachers and teaching.
A. regulate
B. professionalize
C. strengthen
D. improve

43

Downloaded by Noime Burtanog (noimeburtanog@gmail.com)


lOMoARcPSD|38673565

38. As provided in the Republic Act #4670, every teacher shall enjoy equitable safeguards at
each stage of any disciplinary procedures and shall the following except:
A. the right to be informed in writing, of the charges
B. the right to full access to the evidence in the case
C. the right to appeal to clearly designated authorities.
D. All of these
39. The subject matter or content to be learned must be within the time allowed, resources
available, expertise of the teacher, and nature of learners. What criterion is addressed?
A. validity
B. significance
C. interest
D. feasibility
40. A child was shown an amount of water in a glass. The teacher poured the whole amount to a
much taller and narrower glass and marked this glass as the same amount was poured in a
shorter and wider glass, marked glass. When asked which has more water, the child's answer
was "Glass A". In what stage of cognitive development is the child and what is this ability
called?
A. concrete operational stage; conservation
B. formal operational stage; deductive reasoning
C. sensorimotor stage; symbolic functions
D. pre-operational stage; centration
41. In a multiple choice test item with four options and out of 50 examinees, which was the least
effective distracter?
A. the option that was chosen by 13 examinees
B. the option that was chosen by 2 examinees
C. the option that was chosen by 30 examinees
D. the correct answer that was chosen by 5 examinees
42. What is an alternative assessment tool for teaching and learning consisting of a collection of
work/artifacts finished or in-progress accomplished by the targeted clientele?
A. rubric
B. achievement test
C. evaluation instrument
D. portfolio
43. Societal change requires continually deep-seated questions about "good" living. Which of
these did Socrates recognize as the greatest of human virtues?
A. moral wisdom
B. fair justice
C. courage
D. piety
44. Among the following curriculum stakeholders, who has the most responsibility in curriculum
implementation?
A. the learners
B. the school heads
C. the teachers
D. the parents
45. Study this group of tests which was administered with the following results, then answer the
question.
Subject Math Physics English
Mean 56 41 80
SD 10 9 16
Student′s Score 43 31 109

In which subject(s) did Ronnel perform most poorly in relation to the group’s performance?
A. English
B. English and Math
C. Math
D. Physics
46. You like to show a close representation of the size and shape of the earth and its location in
the entire solar system. What is the best instructional aid?
A. picture
B. model
C. realia

44

Downloaded by Noime Burtanog (noimeburtanog@gmail.com)


lOMoARcPSD|38673565

D. film
47. Which of the following techniques of curriculum implementation is fit to the objective of
developing cooperative learning and social interaction?
A. buzz session
B. graded recitation
C. individual reporting
D. lecture
48. To determine your pupil's entry knowledge and skills, which should you employ?
A. interview
B. focus group discussion
C. post-test
D. pre-test
49. We encounter people whose prayer goes like this: “O God, if there is a God; save my soul, if
I have a soul” From whom is this prayer?
A. Stoic
B. Empiricist
C. Agnostic
D. Skeptic
50. What is the implication of using a method that focuses on the why rather than the how?
A. There is best method
B. Typical one will be good for any subject
C. These methods should be standardized for different subjects.
D. Teaching methods should favor inquiry and problem solving.
51. Which software can you predict changes in weather pattern and or trends in the population of
endangered species?
A. word processing
B. spreadsheet
C. desktop publishing
D. database
52. You would like to assess students' ability to write a portfolio. What type of test will
determine their ability to organize ideas and think critically?
A. long test
B. essay test
C. formative test
D. summative test
53. Teacher F asks one student, "Rachel, can you summarize what we have just read? Remember,
the title of this section of the chapter." This is an example of a teacher's effort at _____.
A. scaffolding
B. inspiring
C. directing
D. giving feedback
54. Cooperative learning approach makes use of a classroom organization where students work
in teams to help each other learn. What mode of grouping can facilitate the skill and values
desired?
A. large group
B. homogeneous
C. heterogeneous
D. Wear multicolored dress to catch the students' attention.
55. To solve moral ambiguity among us Filipinos, we must _____.
A. excuse ourselves whenever we do wrong
B. blame our government for not doing anything about it
C. be aware and responsible about the problem
D. be comfortable with the present state affairs
56. In their desire to make schools perform, the DepEd then published the ranking of schools in
NAT results nationwide. As an effect of this practice, what did schools tend to do?
I. Taught at the expense of NAT
II. Conducted review classes for NAT at the expense of teaching
III. Practiced the so-called teaching to the test
A. II and III
B. II only
C. I and III
D. III only

45

Downloaded by Noime Burtanog (noimeburtanog@gmail.com)


lOMoARcPSD|38673565

57. Which one appropriately describes your lesson if you use the cognitive approach?
A. promotes "find out for yourself" approach
B. lecture-dominated
C. rote learning dominated
D. highly directed teaching
58. Which apply/applies to extrinsically motivated learners?
I. Tend to process information superficially
II. Tend to be content with meeting minimum requirements
III. Achieve at high level
A. I and II
B. II only
C. I and III
D. I only
59. To make our children become like "little scientists", which of the following methods should
we employ more often?
I. Inquiry
II. Problem-solving
III. Laboratory
A. II and III
B. I and II
C. I, II, and III
D. I and III
60. Teacher B is a teacher of English as a Second Language. She uses vocabulary cards, fill-in-
the-blank sentences, dictation and writing exercises in teaching a lesson about grocery
shopping. Based on this information, which of the following is a valid conclusion?
A. The teacher is reinforcing learning by giving the same information in, a variety of
methods.
B. The teacher is applying Bloom's hierarchy of cognitive learning.
C. The teacher wants to do less talk.
D. The teacher is emphasizing listening and speaking skills.
61. A mother gives her son his favorite snack every time the boy cleans up his room. Afterwards,
the boy cleans his room everyday in anticipation of the snack. Which theory explains this?
A. operant conditioning
B. social learning theory
C. associative learning
D. Pavlovian conditioning
62. What is the primary fundamental question in examining a curriculum?
A. What educational experiences can be provided that are likely to attain these purposes?
B. What educational purposes should the school seek to attain?
C. How can these educational experiences be effectively organized?
D. How can we determine whether these purposes are attained or not?
63. To elicit more student’s response, Teacher G made use of covert responses. Which one did
she NOT do?
A. She had the students write their response privately.
B. She showed the correct answers on the overhead after the students have written their
responses.
C. She had the students write their responses privately then called each of them.
D. She refrained from judging on the student's responses.
64. Which is the ideal stage of moral development?
A. social contract
B. universal ethical principle
C. law and order
D. good boy/ good girl
65. There is a statement that says, "No amount of good instruction will come out without good
classroom management." Which of the following best explains this statement?
A. Classroom management is important to effect good instruction.
B. There must be classroom management for instruction to yield good outcomes/ results.
C. Classroom management means good instruction.
D. Good instruction is equal to effective classroom management.
66. What does a Table of Specification establish?
A. construct validity
B. content related validity and criterion reference

46

Downloaded by Noime Burtanog (noimeburtanog@gmail.com)


lOMoARcPSD|38673565

C. content validity and construct validity


D. content validity and content related validity
67. A person who had painful experiences at the dentist's office may become fearful at the mere
sight of the dentist's office. Which theory can explain this?
A. generalization
B. classical conditioning
C. operant conditioning
D. attribution theory
68. Which one can help student develop the habit of critical thinking?
A. Asking low level questions
B. A willingness to suspend judgment until sufficient evidence is represented
C. Asking convergent questions
D. Blind obedience to authority
69. Multiple intelligences can be used to explain children's reading performance. Which group
tends to be good readers?
A. linguistically intelligent group
B. spatially intelligent group
C. existentially intelligent group
D. kinesthetically intelligent group
70. Teachers should avoid _____ in assigning student performance-based ratings.
A. arbitrarines and bias
B. unnecessary deductions
C. partiality and calculation
D. unnecessary evaluation
71. Which of the following is NOT a guidance role of the classroom teacher?
A. Psychological Test Administrator
B. Listener-Adviser
C. Human Potential Discoverer
D. Total Development Facilitator
72. Watson applied classical conditioning in his experiments and the results showed that
behavior is learned through stimulus-response associations, specifically the development of
emotional responses to certain stimuli. This helps is in _____.
A. interpreting reflexes as emotions
B. understanding fears, phobias, and love
C. connecting observable behavior to stimulus
D. understanding the role of overt behavior
73. Who claimed that children are natural learners and therefore, must be taught in natural
settings?
A. Piaget
B. Froebel
C. Montessori
D. Kohlberg
74. the benefit of "reading aloud" is that children learn _____.
A. new vocabulary in meaningful contexts
B. to value the presence of their friends as they read together
C. to make predictions by examining pictures and listening for clues
D. to use their imaginations to explore new ideas as they listen to books
75. What does a negatively skewed score contribution imply?
A. The scores congregate on the left side of the normal contribution curve.
B. The scores are widespread.
C. The students must be academically poor.
D. The score congregate on the right side of the normal contribution curve.
76. A high school graduate was refused admission to a university on the grounds that he failed
the admission test. The student insisted that he had the right to be admitted and the act of the
university was a violation of his right to education. Was the student correct?
A. No, the university may refuse the student in its exercise of academic freedom.
B. Yes, education is everyone's right.
C. Yes, especially if he belongs to the Indigenous Peoples' group.
D. No, if the university is exclusively for girls.
77. Which of the following is a correct statement on service contracting scheme?
A. It increases access to education.
B. It works against quality education.

47

Downloaded by Noime Burtanog (noimeburtanog@gmail.com)


lOMoARcPSD|38673565

C. It discriminates against private schools.


D. it is not cost-effective.
78. Student M obtained an NSAT percentile rank of 80. This indicates that _____.
A. he surpassed in performance 80% of her fellow examinees
B. he got a score of 80
C. he surpassed in performance 20% of her fellow examinees
D. he answered 80 items correctly
79. Teachers are encouraged to make use of authentic assessment. Which goes with authentic
assessment?
A. unrealistic performances
B. de-contextualized drills
C. real world application of lessons learned
D. answering high multiple choice test items
80. Global students learn with short bursts of energy. To maintain concentration, they require
______.
A. frequent reminder that they need to concentrate
B. frequent and intermittent breaks
C. short and easy reading materials
D. music while studying
81. A child was punished for cheating in an exam. For sure the child wont cheat again in short
span of time, but this does not guarantee that the child won’t cheat ever again. Based on
Thorndike’s theory on punishment and learning, this shows that _____.
A. punishment strengthens a response
B. punishment doesn’t remove a response
C. punishment removes response
D. punishment weakens a response
82. Faith, hope, and love are values now and forever whether they will be valued by people or
not. Upon what philosophy is this anchored?
A. Idealism
B. Existentialism
C. Realism
D. Pragmatism
83. In a grade distribution, what does the normal curve mean?
A. a large number of students receiving low grades and very few students with high
marks
B. a large number of more or less average students and very few students receiving low
and high grades
C. a large number of students with high grades and very few with low grades
D. all of the students have average grades
84. Lecturer C narrates: "I observe that when there is an English-speaking foreigner in class,
more often than not, his classmates perceive him to be superior." To which Filipino trait does
this point?
A. hospitality
B. friendliness
C. colonial mentality
D. lack of confidence
85. Which violates this brain-based principle of teaching-learning: "Each child's brain is unique
and vastly different from one another."
A. giving ample opportunity for a pupil to explore even if the class creates "noise"
B. making a left-handed pupil write with his right hand as it is better this way
C. allowing open dialogue among students
D. employing MI teaching approaches
86. The child fainted in your class because she has not eaten her breakfast. What is the best thing
for you to do in this situation?
A. Ignore the situation
B. Comfort the child
C. Give the child food
D. Call the parent
87. Which one should teacher AVOID to produce an environment conducive for learning?
A. Tests
B. Seat plan
C. Individual competition

48

Downloaded by Noime Burtanog (noimeburtanog@gmail.com)


lOMoARcPSD|38673565

D. Games
88. You are convinced that whenever a student performs a desired behavior, provide
reinforcement and soon the student learns to perform the behavior on her own. On which
principle is your conviction based?
A. cognitivism
B. behaviorism
C. constructivism
D. environmentalism
89. “Men are built not born.” This quotation by John Watson states that _____.
A. the ineffectiveness of training on a person’s development
B. the effect of environmental stimulation on a person’s development
C. the absence of genetic influence on a person’s development
D. the effect of heredity
90. Which order follows the basic rule in framing interaction?
A. call on a student, pause, ask the question
B. ask the question, call on a student, pause
C. ask the question, pause, call on a student
D. call on a student, ask the question, pause
91. At the preoperational stage of Piaget's cognitive development theory, the child can see only
his point of view and assumes that everyone also has the same view as his. What is this
tendency called?
A. transductive reasoning
B. animism
C. egocentrism
D. conservatism
92. "Vox Populi Est Supreme Lex" is a Latin expression that means what?
A. The Supreme Being is God
B. No one is above the law
C. The voice of the people is the supreme law
D. it is the popular choice
93. Which is not among the major targets of the Child-Friendly School System (CFSS)?
A. All school children are friendly.
B. All children 6-12 years old are enrolled in elementary schools.
C. All children complete their elementary education within six years.
D. All Grade 6 students pass the division, regional, and national tests.
94. Thorndike's law of effect states that a connection between stimulus and response is
strengthened when the consequence is _____.
A. repeated
B. negative
C. pleasurable
D. positive
95. Which is the most reliable tool of seeing the development in your pupils' ability to write?
A. portfolio assessment
B. scoring rubric
C. interview of pupils
D. self-assessment
96. The Philippine constitution directs the teaching of religion in public schools on the following
conditions except for?
A. option is expressed in writing
B. without cost to the government
C. with cost shouldered by the parents or guardians
D. given only at the option of parents or guardians
97. These are also known as “combination classes” organized in barrios/barangays where the
required number of pupils of the same grade levels has not met the required number to make
a separate class thus the teacher apportions class time for instruction to every grade level
within class. These are ______.
A. extension classes
B. heterogeneous classes
C. multigrade classes
D. homogeneous classes

49

Downloaded by Noime Burtanog (noimeburtanog@gmail.com)


lOMoARcPSD|38673565

98. After reading and paraphrasing R. Frost's Stopping by the Woods on a Snowy Evening,
Teacher M asked the class to share any insight derived from this poem. On which assumption
about the learner is Teacher M's act of asking the class to share their insight based?
A. Learners are producers of knowledge, not only passive recipients of information.
B. Learners are meant to interact with one another.
C. Learners are like empty receptacles waiting to be filled up.
D. Learners have multiple intelligence and varied learning styles.
99. "Do not cheat. Cheating does not pay. If you do, you cheat yourself." says the voiceless voice
from within you. In the context of Freud's theory, which is at work
A. id alone
B. superego alone
C. ego alone
D. Id and ego interaction
100. Which of the following assessment tools would you recommend if one should adhere to
constructivist theory of learning?
I. Constructed response test
II. Performance test
III. Checklist of a motor screening test
IV. Observation test
A. I and II
B. II and III
C. I, II, and III
D. I, II, and IV
101. The test in English and Mathematics showed poor results in comprehension and problem-
solving questions. How may the data be used for better learners' performance?
A. Use context clues in vocabulary building.
B. Give more exercises/situations on comprehension questions.
C. Determine weakness in grammatical structures.
D. Involve parents in guiding learners' developing good study habits.
102. Which of the following may be the assumption/s of behaviorists?
I. The mind of a newborn child is a blank slate.
II. All behaviors are determined by environmental events.
III. The child has a certain degree of freedom not to allow himself to be shaped by his
environment.
A. III only
B. I and II
C. I and III
D. II only
103. _____ supports equitable access but on the other hand, quality might be compromised.
A. Open admission
B. School accreditation
C. Deregulated tuition fee
D. Selective retention
104. If you want your students to develop reading comprehension and learning strategies
which one should you employ?
A. reciprocal teaching
B. cooperative learning
C. peer tutoring
D. mastery learning
105. What statement is FALSE with reference to Article VIII “The Teacher and the Learners”
of the Code of Ethics of Professional Teachers?
A. A teacher shall not accept favors or gifts from learners, their parents, or others in their
behalf in exchange for requested concessions, especially if undeserved,
B. A teacher shall not inflict corporal punishment or offending learners nor make
deductions from their scholastic ratings as a punishment for acts which are clearly not
manifestation of poor scholarship.
C. In a situation where mutual attraction and subsequent love develop between teacher
and the learner, the teacher shall exercise utmost professional discretion to avoid
scandal, gossip, and preferential treatment of the learner.
D. A teacher shall maintain at all ties a dignified personality, which could serve as model
worthy of emulation by learners, peers and others.

50

Downloaded by Noime Burtanog (noimeburtanog@gmail.com)


lOMoARcPSD|38673565

106. In the light of the modern concept of teaching, which is a characteristic of effective
teaching?
A. pouring information to the learners
B. allowing learners to learn on their own
C. developing abilities to address the future
D. removing the physical presence of the teacher
107. One facet of understanding, an evidence of learning is perspective. Which is an indicator
of perspective?
A. A bright student refuses to consider that there is another correct solution to the
problem apart from hers.
B. A student explains the arguments for and against the acquittal of Hubert Webb and
group.
C. A teacher cannot accept opinions different from hers.
D. A mother cannot understand why her child's performance is below par.
108. Which among the indicators could be most useful for assessing quality of schooling?
A. participation rate
B. cohort survival rate
C. net enrollment rate
D. drop-out rate
109. The discrimination index of a test item is +0.48. What does this mean?
A. An equal number from the lower and upper group got the item correctly.
B. More from the upper group got the item wrongly.
C. More from the lower group got the item correctly.
D. More from the upper group got the item correctly.
110. Which of the following is NOT an example of a teacher’s nonverbal communication?
A. eye contact
B. gestures
C. pauses
D. voice
111. Which is a valid assessment tool if you want to find out how well your students can speak
extemporaneously?
A. performance test in extemporaneous speaking
B. written quiz on how to deliver extemporaneous speech
C. display of speeches delivered
D. writing speeches
112. Professionalization of teachers and teaching as promulgated in Presidential Decree No.
1006, defines teaching as profession concerned with classroom institution ______.
A. by teachers on fulltime basis
B. at the tertiary level in both public and private institutions
C. at the elementary and secondary levels in both public and private schools
D. by teachers of permanent status
113. Teacher U asked her pupils to create a story out of the given pictures. Which projective
technique did Teacher U use?
A. Rorschach test
B. narrative
C. thematic apperception test
D. reflective
114. The Code of Ethics for Professional Teachers stipulates that educational institutions shall
offer quality education for all Filipino citizens. How is quality education defined in R.A.
9155?
A. Relevance and excellence of education are emphasized to meet the needs and
aspirations of an individual and society.
B. All school-aged children should be provided free and compulsory education.
C. Children with special needs should be mainstreamed with regular classes in the public
schools.
D. Public and private basic education schools should provide relevant education.
115. Teacher A discovered that his pupils are very good in dramatizing. Which tool must have
helped him discover his pupils’ strength?
A. Portfolio assessment
B. Performance test
C. Journal entry
D. Paper-and-pencil test

51

Downloaded by Noime Burtanog (noimeburtanog@gmail.com)


lOMoARcPSD|38673565

116. Which appropriate teaching practice flows from this research finding on the brain: The
brain's emotional center is tied into its ability to learn.
A. Create a learning environment that encourages students to explore their feelings and
ideas freely.
B. Come up with highly competitive games where winners will feel happy.
C. Tell students to participate in class activities or else won't receive plus points in class
recitation.
D. To establish discipline, be judgmental in attitude.
117. Your percentile rank in class is 60%. What does this mean?
A. You got 40% of the test items wrongly.
B. You scored less than 60% of the class.
C. You got 60% of the test items correctly.
D. You scored better than 60% of the class.
118. Which test item is in the highest level of Bloom's taxonomy of objectives?
A. Explain how a tree functions in relation to the ecosystem.
B. Explain how trees receive nutrients.
C. Rate three different methods of controlling tree growth.
D. Write a paragraph that observes coherence, unity, and variety.
119. In qualitative social and behavioral studies, "the investigator is a part of the study." What
are implied in this statement?
I. The researcher processes and analyzes the data himself.
II Data interpretation depends on the orientation of the researcher.
III. The investigator is the only source of information.
IV. Data gathering may be done by others but the analysis is done by the researcher.
A. I and IV
B. II and III
C. I, II, and IV
D. I, II, III, and IV
120. The observable manifestation of student's feelings, thoughts, or attitude are summed up
as behavior. Every high school teacher is expected to contribute to the assessment of the
student's behavior but the grade is reflected in _____.
A. Good Manners and Right Conduct
B. Values Education
C. co-curricular activities
D. curricular activities
121. Which must be present for self-evaluation to succeed?
A. consensus between teacher and student regarding evaluation results
B. teacher's approval of self-evaluation results
C. teacher's monitoring of self-evaluation process
D. student's intrinsic motivation to learn
122. The instructions for a test are made simple, clear and concise. This is part of which of the
following characteristics of a good test?
A. objectivity
B. economy
C. administrability
D. scorability
123. What is the mean of this score distribution: 4, 5, 6, 7, 8, 9, 10?
A. 7
B. 6
C. 8.5
D. 7.5
124. The Early Childhood Care and Development Act provides for the promotion of the rights
of children for survival and development. Which of the following statements is not among its
objectives?
A. Enhance the role of parents as the primary caregivers and educators of their children
form birth onward.
B. Facilitate a smooth transition from care and education provided at home to
community or school-based setting and primary schools.
C. Assist the LGUs in their endeavor to prepare the child for adulthood.
D. Enhance the physical, social, emotional, cognitive, psychological, spiritual, and
language development of young children.
125. Which of the following represents a minuscule curriculum?

52

Downloaded by Noime Burtanog (noimeburtanog@gmail.com)


lOMoARcPSD|38673565

A. textbook that learners use


B. lesson plan that teachers prepare
C. reference materials that supplement the text
D. lecture notes of the teacher
126. Bernadette enjoyed the roller coaster when they went to Enchanted Kingdom. Just at the
sight of a roller coaster, she gets excited. Which theory explains Bernadette's behavior?
A. operant conditioning
B. attribution theory
C. Pavlovian conditioning
D. social learning theory
127. What objectives do effective leaders foster?
I. Attainable
II.Measurable
III.Results-oriented
IV. Specific
V. Time-bound
A. II, III, and IV
B. I, II, III, IV, and V
C. I, IV, and V
D. I, II, III, and IV
128. Thomasites are _____.
A. the soldiers who doubted the success of the public educational system to be set in the
Philippines
B. the first American teacher recruits to help establish the public educational system in
the Philippines
C. the first religious group who came to the Philippines on board the US transport named
Thomas.
D. the devotees of St. Thomas Aquinas who came to evangelize
129. Which is an underlying assumption of the social cognitive theory?
A. People are social by nature.
B. People learn by observing others.
C. People learn by trial-and-error.
D. People learn by association.
130. Researchers found that when a child is engaged in a learning experience, a number of
areas of the brain are simultaneously activated. What is an application of this in the teaching-
learning process?
I. Make use of field trips, guest speakers
II. Do multicultural units of study
III. Stick to the "left brain and right brain" approach
A. II only
B. I only
C. I and III
D. I and II
131. Here is a score distribution: 98, 93, 93, 93, 90, 88, 87, 85, 85, 85, 70, 51, 34, 34, 34, 20,
18, 15, 12, 9, 8, 6, 3, 1. Which is the range?
A. 93
B. 85
C. 97
D. between 51 and 34
132. Which tool should a teacher use if she wants to locate areas which are adversely affecting
the performance of a significant number of students?
A. problem checklist
B. self-report technique
C. autobiography
D. cumulative record
133. Which activity works best with self-expressive people?
A. metaphors
B. kinesthetic activities
C. inquiry
D. independent study
134. centralization: Education Act of 1901; decentralization: _____
A. R.A. 9155

53

Downloaded by Noime Burtanog (noimeburtanog@gmail.com)


lOMoARcPSD|38673565

B. R.A. 9293
C. R.A. 7836
D. R.A. 7722
135. Piagetian tasks state that thinking becomes more logical and abstract as children reach the
formal operations stage. What is an educational implication of this finding?
A. Expect hypothetical reasoning from learners between 12 to 15 years of age.
B. Engage children in analogical reasoning as early as preschool to train them for HOTS.
C. Learners who are not capable of logical reasoning from ages 8 to 11 lag behind in
their cognitive development.
D. Let children be children.
136. This embodies the teacher’s duties and responsibilities as well as proper behavior in
performing them.
A. Code of Ethics for Teachers
B. Magna Carta for Public School Teachers
C. Bill of Rights
D. Philippine Constitution of 1987
137. The class was asked to share their insights about the poem. The ability to come up with
an insight stems from the ability to _____.
A. comprehend the subject that is being studied
B. analyze the parts of a whole
C. evaluate the worthiness of a thing
D. relate and organize things and ideas
138. Which is not a characteristic of authentic assessment?
A. focused on lifelike, meaningful, relevant types of student learning
B. offers opportunities to study problem intensively
C. easy to complete
D. fruitful in terms of genuine learning
139. Schools should develop in the students the ability to adapt to a changing world. This is
adherence to the philosophy of _____.
A. essentialism
B. perennialism
C. progressivism
D. reconstructionism
140. The teacher’s first task in the selection of media in teaching is to determine the ______.
A. choice of the students
B. availability of the media
C. objectives of the lesson
D. technique to be used
141. With which goals of educational institutions as provided for by the Constitution is the
development of work skills aligned?
A. To develop moral character
B. To teach the duties of citizenship
C. To inculcate love of country
D. To develop vocational efficiency
142. Median is to point as standard deviation is to _____.
A. area
B. volume
C. distance
D. square
143. Who is the forerunner of the presence of the Language Acquisition Device?
A. Watson
B. Chomsky
C. Gardner
D. Piaget
144. Which of the following statements best describes metacognition as a strategy for
curriculum augmentation?
A. It is learning how to learn and thinking about how one thinks.
B. It is learning strategies for success.
C. It is learning through interaction with the environment.
D. It is learning through computer-aided instruction.
145. Which psychological theory states that the mind insists on finding patterns in things that
contribute to the development of insight?

54

Downloaded by Noime Burtanog (noimeburtanog@gmail.com)


lOMoARcPSD|38673565

A. Piaget's psychology
B. Kohlberg's psychology
C. Gestalt psychology
D. Bruner's psychology
146. Which is not a characteristic of an analytic learner?
A. learns whole to part
B. enjoys memorizing
C. works on details
D. sequences objects in order
147. The concepts of trust vs. maturity, autonomy vs. self-doubt, and initiative vs. guilt are
most closely related with the works of __________.
A. Erikson
B. Piaget
C. Freud
D. Jung
148. Ask to do a learning task, Joe hesitates and says "Mahirap. Ayaw ko. 'Di ko kaya!" (It's
difficult. I don't like it. I can't do it.) Is it possible to motivate this type of student?
A. Yes, he can do something with his ability.
B. Yes, he can change the nature of the job.
C. No, it is impossible to motivate a student who himself is not motivated.
D. No, motivation is totally dependent on the student. No person outside him can
influence him.
149. As of the Republic Act 7836 the licensure exam for teachers is with the _____.
A. Commission on Higher Education
B. Professional Regulation Commission
C. Department of Education
D. Civil Service Commission
150. Filipinization is violated if _____.
A. an educational institution is owned by a corporation of which 40% of the capital is
owned by Filipino Citizens
B. an educational institution owned by a religious order
C. an American serving as president of the educational institution
D. an educational institution owned by a charitable institution

PROFESSIONAL EDUCATION
Set B
1. Which assumption underlies the teacher's use of performance objectives?
A. Not every form of learning is observable.
B. Performance objectives assure the learrier of learning.
C. Learning is defined as a change in the learner's observable performance.
D. The success of learner is based on teacher performance.
2. In the parlance of test construction what does TOS mean?
A. Table of Specifics
B. Table of Specifications
C. Table of Specific Test Items
D. Team of Specifications
3. A student passes a research report poorly written but ornately presented in a folder to make
up for the poor quality of the book report content. Which Filipino trait does this practice
prove? Emphasis on __________.
A. art over academics
B. substance overporma
C. art over science
D. pormaover substance
4. In a criterion-referenced testing, what must you do to ensure that your test is fair?
A. Make all of the questions true or false.
B. Ask each student to contribute one question.
C. Make twenty questions but ask the students to answer only ten of their choice.
D. Use the objectives for the units as guide in your test construction.
5. Which does Noam Chomsky, assert about language learning for children?
I. Young children learn and apply grammatical rules and vocabulary as they are exposed to
them.
II. Begin formal teaching of grammatical rules to children as early as possible.

55

Downloaded by Noime Burtanog (noimeburtanog@gmail.com)


lOMoARcPSD|38673565

III. Do not require initial formal language teaching for children.


A. I and III
B. II only
C. I only
D. I and II
6. Which Filipino trait works against the shift in teacher's role from teacher as a fountain of
information to teacher as facilitator?
A. Authoritativeness
B. Authoritarianism
C. Hiya
D. Pakikisama
7. If a teacher plans a constructivist lesson, what will he most likely do? Plan how he can
A. do evaluate his students' work
B. do reciprocal teaching
C. lecture to his students
D. engage his students in convergent thinking
8. Who among the following needs less verbal counseling but needs more concrete and
operational forms of assistance? The child who __________.
A. has mental retardation
B. has attention-deficit disorder
C. has learning disability
D. has conduct disorder
9. How would you select the most fit in government positions? Applying Confucius teachings,
which would be the answer?
A. By course accreditation of an accrediting body
B. By merit system and course accreditation
C. By merit system
D. By government examinations
10. Which types of play is most characteristic of a four to six-year old child?
A. Solitary and onlooker plays
B. Associative and cooperative plays
C. Associative and onlooker plays
D. Cooperative and solitary plays
11. A teacher's summary of a lesson serves the following functions, EXCEPT
A. it links the parts of the lesson
B. it brings together the information that has been discussed
C. it makes provisions for full participation of students.
D. it clinches the basic ideas or concepts of the lesson.
12. All of the following describe the development of children aged eleven to thirteen EXCEPT
__________.
A. they shift from impulsivity to adaptive ability
B. sex differences in IQ becomes more evident
C. they exhibit increase objectivity in thinking
D. they show abstract thinking and judgement
13. Teacher T taught a lesson denoting ownership by means of possessives. He first introduced
the rule, then gave examples, followed by class exercises, then back to the rule before he
moved into the second rule. Which presenting technique did he use?
A. Combinatorial
B. Comparative
C. Part-whole
D. Sequence
14. "In the light of the facts presented, what is most likely to happen when?" is a sample thought
question on
A. inferring
B. generalizing
C. synthesizing
D. justifying
15. Teacher E discussed how electricity flows through wires and what generates the electric
charge. Then she gave the students wires, bulbs, switches, and dry cells and told the class to
create a circuit that will increase the brightness of each bulb. Which one best describes the
approach used?
A. It used a taxonomy of basic thinking skills

56

Downloaded by Noime Burtanog (noimeburtanog@gmail.com)


lOMoARcPSD|38673565

B. It was constructivist
C. It helped students understand scientific methodology
D. It used cooperative learning
16. Teacher B uses the direct instruction strategy. Which sequence of steps will she follow?
I. Independent practice
II. Feedback and correctiveness
III. Guided student practice
IV. Presenting and structuring
V. Reviewing the previous day's work
A. V-II-IV-III-I
B. III-II-IV-I-V
C. V-IV-III-II-I
D. I-V-II-III-IV
17. Which are direct measures of competence?
A. Personality tests
B. Performance tests
C. Paper-and-pencil tests
D. Standardized test
18. Under which program were students who were not accommodated in public elementary and
secondary schools because of lack of classroom, teachers, and instructional materials, were
enrolled in private schools in their respective communities at the government's expense?
A. Government Assistance Program
B. Study Now-Pay Later
C. Educational Service Contract System
D. National Scholarship Program
19. Which activity should a teacher have more for his students if he wants them to develop
logical-mathematical thinking?
A. Problem solving
B. Choral reading
C. Drama
D. Storytelling
20. An effective classroom manager uses low-profile classroom control. Which is a low-profile
classroom technique?
A. Note to parents
B. After-school detention
C. Withdrawal of privileges
D. Raising the pitch of the voice
21. Your teacher is of the opinion that the world and everything in it are ever changing and so
teaches you the skill to cope with change. What is his governing philosophy?
A. Idealism
B. Existentialism
C. Experimentalism
D. Realism
22. To come closer to the truth we need to go back to the things themselves. This is the advice of
the
A. behaviorists
B. phenomenologists
C. idealists
D. pragmatists
23. Test norms are established in order to have a basis for __________.
A. establishing learning goals
B. interpreting test results
C. computing grades
D. identifying pupils' difficulties
24. A stitch on time saves nine, so goes the adage. Applied to classroom management, this means
that we __________
A. may not occupy ourselves with disruptions which are worth ignoring because they are
minor
B. must be reactive in our approach to discipline
C. have to resolve minor disruptions before they are out of control
D. may apply 9 rules out of 10 consistently
25. Which criterion should guide a teacher in the choice of instructional devices?

57

Downloaded by Noime Burtanog (noimeburtanog@gmail.com)


lOMoARcPSD|38673565

A. Attractiveness
B. Cost
C. Novelty
D. Appropriateness
26. Based on Piaget's theory, what should a teacher provide for children in the sensimotor stage?
A. Games and other physical activities to develop motor skill.
B. Learning activities that involve problems of classification and ordering.
C. Activities for hypothesis formulation.
D. Stimulating environment with ample objects to play with.
27. Who among the following puts more emphasis on core requirements, longer school day,
longer academic year and more challenging textbooks?
A. Perennialist
B. Essentialist
C. Progressivist
D. Existentialist
28. The search for related literature by accessing several databases by the use of a telephone line
to connect a computer library with other computers that have database is termed
__________.
A. compact disc search
B. manual search
C. on-line search
D. computer search
29. With synthesizing skills in mind, which has the highest diagnostic value?
A. Essay test
B. Performance test
C. Completion test
D. Multiple choice test
30. Based on Piaget's theory, what should a teacher provide for children in the concrete
operational stage?
A. Activities for hypothesis formulation.
B. Learning activities that involve problems of classification and ordering.
C. Games and other physical activities to develop motor skills.
D. Stimulating environment with ample objects to play with.
31. Read the following then answer the question:
TEACHER: In what ways other than the periodic table might we predict the undiscovered
elements?
BOBBY: We could go to the moon and see if there are some elements there, we don't have.
BETTY: We could dig down to the center of the earth and see if we find any of the missing
elements.
RICKY: We could study debris from the meteorites if we can find any.
TEACHER: Those are all good answers but what if those, excursions to the moon, to the
center of the earth, or to find meteorites were too costly and time consuming? How might we
use the elements we already have here on earth to find some new ones?
Question: Which questioning strategy/ies does/do the exchange of thoughts above illustrate?
A. Funneling
B. Sowing and reaping
C. Nose-dive
D. Extending and lifting
32. Which is NOT a sound purpose for asking questions?
A. To probe deeper after an answer is given.
B. To discipline a bully in class.
C. To remind students of a procedure.
D. To encourage self-reflection.
33. The main purpose of compulsory study of the Constitution is to __________
A. develop students into responsible, thinking citizens
B. acquaint students with the historical development of the Philippine Constitution
C. make constitutional experts of the students
D. prepare students for law-making
34. A child who gets punished for stealing candy may not steal again immediately. But this does
not mean that the child may not steal again. Based on Thorndike's theory on punishment and
learning, this shows that __________
A. punishment strengthens a response

58

Downloaded by Noime Burtanog (noimeburtanog@gmail.com)


lOMoARcPSD|38673565

B. punishment removes a response


C. punishment does not remove a response
D. punishment weakens a response
35. As a teacher, you are a reconstructionist. Which among these will be your guiding principle?
A. I must teach the child every knowledge, skill, and value that he needs for a better
future.
B. I must teach the child to develop his mental powers to the full.
C. I must teach the child so he is assured of heaven.
D. I must teach the child that we can never have real knowledge of anything.
36. How can you exhibit expert power on the first day of school?
A. By making them feel you know what you are talking about.
B. By making them realize the importance of good grades.
C. By reminding them your students your authority over them again and again.
D. By giving your students a sense of belonging and acceptance.
37. A sixth-grade twelve-year old boy comes from a dysfunctional family and has been abused
and neglected. He has been to two orphanages and three different elementary schools. The
student can decide on the second-grade level, but he can comprehend orally material at the
fourth or fifth grade level. The most probable cause/s of this student's reading problem is/are
__________.
A. emotional factors
B. poor teaching
C. neurological factors
D. immaturity
38. Teacher Q does not want Teacher B to be promoted and so writes an anonymous letter against
Teacher B accusing her of fabricated lies Teacher Q mails this anonymous letter to the
Schools Division Superintendent. What should Teacher Q do if she has to act professionally?
A. Submit a signed justifiable criticism against Teacher B, if there is any.
B. Go straight to the Schools Division Superintendent and gives criticism verbally.
C. Hire a group to distribute poison letters against Teacher B for information
dissemination.
D. Instigate student activists to read poison letters over the microphone.
39. In self-directed learning, to what extent should a teacher's scaffolding be?
A. To a degree the student needs it.
B. None, to force the student to learn by himself.
C. To the minimum, to speed up development of student's sense of independence.
D. To the maximum, in order to extend to the student all the help he needs.
40. If your Licensure Examination Test (LET) items sample adequately the competencies listed
in the syllabi, it can be said that the LET possesses __________ validity.
A. concurrent
B. construct
C. content
D. predictive
41. Which guideline must be observed in the use of prompting to shape the correct performance
of your students?
A. Use the least intrusive prompt first.
B. Use all prompts available.
C. Use the most intrusive prompt first.
D. Refrain from using prompts.
42. It is not wise to laugh at a two-year old child when he utters bad word because in his stage he
is learning to __________.
A. consider other's views
B. distinguish sex differences
C. socializes
D. distinguish right from wrong
43. In a treatment for alcoholism, Ramil was made to drink an alcoholic beverage and then made
to ingest a drug that produces nausea. Eventually, he was nauseated at the sight and smell of
alcohol and stopped drinking alcohol. Which theory explains this?
A. Operant conditioning
B. Social Learning Theory
C. Associative Learning
D. Attribution Theory
44. Which is a true foundation of the social order?

59

Downloaded by Noime Burtanog (noimeburtanog@gmail.com)


lOMoARcPSD|38673565

A. Obedient citizenry
B. The reciprocation of rights and duties
C. Strong political leadership
D. Equitable distribution of wealth
45. As a teacher, what do you do when you engage yourself in major task analysis?
A. Test if learning reached higher level thinking skills.
B. Breakdown a complex task into sub-skills.
C. Determine the level of thinking involved.
D. Revise lesson objectives.
46. In instructional planning it is necessary that the parts of the plan from the first to the last have
__________.
A. clarity
B. symmetry
C. coherence
D. conciseness
47. In a study conducted, the pupils were asked which nationality they preferred, if given a
choice. Majority of the pupils wanted to be Americans. In this case, in which obligation
relative to the state, do schools seem to be failing? In their obligation to __________.
A. respect for all duly constituted authorities
B. promote national pride
C. promote obedience to the laws of the state
D. instill allegiance to the Constitution
48. The best way for a guidance counselor to begin to develop study skills and habits in
underachieving student would be to __________.
A. have these underachieving students observe the study habits of excelling students
B. encourage students to talk about study habits from their own experiences
C. have them view film strips about various study approaches
D. give out a list of effective study approaches
49. What is most likely to happen to our economy when export continuously surpasses importis a
thought question on __________.
A. creating
B. relating cause-and-effect
C. synthesizing
D. predicting
50. Direct instruction is for facts, rules, and actions as indirect instruction is for __________,
__________, __________.
A. hypotheses, verified data and conclusions
B. concepts, patterns and abstractions
C. concepts, processes and generalizations
D. guesses, data and conclusions
51. Why should a teacher NOT use direct instruction all the time?
A. It requires much time.
B. It requires use of many supplementary materials.
C. It is generally effective only in the teaching of concepts and abstractions.
D. It reduces students’ engagement in learning.
52. The principle of individual differences requires teachers to __________.
A. give greater attention to gifted learners
B. provide for a variety of learning activities
C. treat all learners alike while in the classroom
D. prepare modules for slow learners in class
53. Which questioning practice promotes more class interaction?
A. Asking the question before calling on a student.
B. Focusing on divergent question
C. Focusing on convergent questions.
D. Asking rhetorical questions.
54. Teacher F wanted to teach the pupils the skill to do cross stitching. Her check-up quiz was a
written test on the steps of cross stitching. Which characteristic of a good test does it lack?
A. Scorability
B. Reliability
C. Objectivity
D. Validity

60

Downloaded by Noime Burtanog (noimeburtanog@gmail.com)


lOMoARcPSD|38673565

55. The attention to the development of a deep respect and affection for our rich cultural past is
an influence of __________.
A. Confucius
B. Hegel
C. Teilhard de Chardin
D. Dewey
56. How can you exhibit legitimate power on the first day of school?
A. By making your students feel they are accepted for who they are.
B. By informing them you are allowed to act in loco parentis.
C. By making them realize the importance of good grades.
D. By making them feel you have mastery of subject matter.
57. Study this group of tests which was administered with the following results, then answer the
question.
Subject Mean SD Ronnels's Score
Math 56 10 43
Physics 41 9 31
English 80 16 109
In which subject(s) did Ronnel perform most poorly in relation to the group's performance?
A. English
B. English and Math
C. Math
D. Physics
58. Which holds true to standardized tests?
A. They are used for comparative purposes
B. They are administered differently
C. They are scored according to different standards
D. They are used for assigning grades
59. Study this group of tests which was administered with the following results, then answer the
question.
Subject Mean SD Ronnel's Score
Math 56 10 43
Physics 41 9 31
English 80 16 109
In which subject(s) were the scores most homogenous?
A. Math
B. English
C. Physics
D. Physics and Math
60. What can be said of Peter who obtained a score of 75 in a Grammar objective test?
A. He answered 75 items in the test correctly.
B. He answered 75% of the test items correctly.
C. His rating is 75.
D. He performed better than 5% of his classmates.
61. Quiz is to formative test while periodic is to __________
A. criterion-reference test
B. summative test
C. norm-reference test
D. diagnostic test
62. Which applies when skewness is zero?
A. Mean is greater than the median
B. Median is greater than mean
C. Scores have three modes
D. Scores are normally distributed
63. Value clarification as a strategy in Values Education classes is anchored on which
philosophy?
A. Existentialism
B. Christian philosophy
C. Idealism
D. Hedonism
64. Out of 3 distracters in a multiple choice test item, namely B, C, and D, no pupil chose D as
answer. This implies that D is __________
A. an ineffective distracter

61

Downloaded by Noime Burtanog (noimeburtanog@gmail.com)


lOMoARcPSD|38673565

B. a vague distracter
C. an effective distracter
D. a plausible distracter
65. All men are pretty much alike. It is only by custom that they are set apart, said one Oriental
philosopher. Where can this thought be most inspiring?
A. In a multi-cultural group of learners
B. In multi-cultural and heterogeneous groups of learners and indigenous peoples' group
C. In a class composed of indigenous peoples
D. In heterogeneous class of learners
66. Which group of philosophers maintain that truth exists in an objective order that is
independent of the knower?
A. Idealists
B. Pragmatists
C. Existentialists
D. Realists
67. Each teacher is said to be a trustee of the cultural and educational heritage of the nation and
is, under obligation to transmit to learners such heritage. Which practice makes him fulfill
such obligation?
A. Use the latest instructional technology.
B. Observe continuing professional education.
C. Use interactive teaching strategies.
D. Study the life of Filipino heroes.
68. Teacher B engages her students with information for thorough understanding for meaning
and for competent application. Which principle governs Teacher B's practice?
A. Constructivist
B. Gestalt
C. Behaviorist
D. Cognitivist
69. In which competency do my students find the greatest difficulty? In the item with a difficulty
index of __________.
A. 0.1
B. 0.9
C. 0.5
D. 1.0
70. Rights and duties are correlative. This means that __________.
A. rights and duties regulate the relationship of men in society
B. rights and duties arise from natural law
C. each right carries with it one or several corresponding duties
D. rights and duties ultimately come from God
71. Which describes norm-referenced grading?
A. The performance of the group
B. What constitutes a perfect score?
C. The students' past performance
D. An absolute standard
72. Teacher U teaches to his pupils that pleasure is not the highest good. Teacher's teaching is
against what philosophy?
A. Realism
B. Hedonism
C. Epicureanism
D. Empiricism
73. Standard deviation is to variability as mode to __________.
A. level of difficulty
B. discrimination
C. correlation
D. central tendency
74. Which test has broad sampling of topics as strength?
A. Objective test
B. Short answer test
C. Essay test
D. Problem type
75. I combined several subject areas in order to focus on a single concept for inter-disciplinary
teaching. Which strategy/method did I use?

62

Downloaded by Noime Burtanog (noimeburtanog@gmail.com)


lOMoARcPSD|38673565

A. Problem-entered learning
B. Thematic instruction
C. Reading-writing activity
D. Unit method
76. Which teaching activity is founded on Bandura's Social Learning Theory?
A. Lecturing
B. Modeling
C. Questioning
D. Inductive Reasoning
77. For which may you use the direct instruction method?
A. Become aware of the pollutants around us.
B. Appreciate Milton's Paradise Lost.
C. Use a microscope properly.
78. We encounter people whose prayer goes like this: "O God, if there is a God; save my soul, if
I have a soul" From whom is this prayer?
A. Stoic
B. Empiricist
C. Agnostic
D. Skeptic
79. What measure of central tendency does the number 16 represent in the following data: 14,
15, 17, 16, 19, 20, 16, 14, 16?
A. Mode
B. Median
C. Mode and median
D. Mean
80. Availment of the Philippine Education Placement Test (PEPT) for adults and out-of-school
youths is in support of the government’s educational program towards __________.
A. equitable access
B. quality
C. quality and relevance
D. relevance
81. Which one can best evaluate students' attitudinal development?
A. Essay test
B. Portfolio
C. Observation
D. Short answer test
82. Which is a form of direct instruction?
A. Discovery process
B. Problem solving
C. Programmed instruction
D. Inductive reasoning
83. History books used in schools are replete with events portraying defeats and weaknesses of
the Filipino as a people. How should you tackle them in the classroom?
A. Present them and express your feelings of shame.
B. Present facts and use them as means in inspiring your class to learn from them.
C. Present them and blame those people responsible or those who have contributed.
D. Present them as they are presented, and tell the class to accept reality.
84. Standard deviation is to variability as mean is to __________.
A. coefficient of correlation
B. central tendency
C. discrimination index
D. level of difficulty
85. A mother gives his boy his favorite snack everytime the boy cleans up his room. Afterwards,
the boy cleaned his room everyday in anticipation of the snack. Which theory is illustrated?
A. Associative Learning
B. Classical Conditioning
C. Operant Conditioning
D. Pavlonian Conditioning
86. Bruner's theory on intellectual development moves from enactive to iconic and symbolic
stages. In which stage(s) are diagrams helpful to accompany verbal information?
A. Enactive and iconic
B. Symbolic

63

Downloaded by Noime Burtanog (noimeburtanog@gmail.com)


lOMoARcPSD|38673565

C. Symbolic and enactive


D. Iconic
87. Teacher A discovered that his pupils are very good in dramatizing. Which tool must have
helped him discover his pupils' strength?
A. Portfolio assessment
B. Performance test
C. Journal entry
D. Paper-and-pencil test
88. Which one can enhance the comparability of grades?
A. Using common conversion table for translating test scores in to ratings
B. Formulating tests that vary from one teacher to another
C. Allowing individual teachers to determine factors for rating
D. Individual teachers giving weights to factors considered for rating
89. In the Preamble of the Code of Ethics of Professional Teachers, which is NOT said of
teachers?
A. LET passers
B. Duly licensed professionals
C. Possess dignity and reputation
D. With high-moral values as well as technical and professional competence
90. Which is the first step in planning an achievement test?
A. Define the instructional objective.
B. Decide on the length of the test.
C. Select the type of test items to use.
D. Build a table of specification.
91. A teacher who equates authority with power does NOT __________.
A. shame
B. develop self-respect in every pupil
C. retaliate
D. intimidate
92. Which is a major advantage of a curriculum-based assessment?
A. It is informal in nature.
B. It connects testing with teaching.
C. It tends to focus on anecdotal information on student progress.
D. It is based on a norm-referenced measurement model.
93. Which does NOT belong to the group of alternative learning systems?
A. Multi-grade grouping
B. Multi-age grouping
C. Graded education
D. Non-graded grouping
94. Theft of school equipment like tv, computer, etc. by teenagers in the community itself is
becoming a common phenomenon. What does this incident signify?
A. Prevalence of poverty in the community.
B. Inability of school to hire security guards.
C. Deprivation of Filipino schools.
D. Community's lack of sense of co-ownership.
95. Which can effectively measure students' awareness of values?
A. Projective techniques
B. Moral dilemma
C. Likert scales
D. Anecdotal record
96. Based on Edgar Dale's Cone of Experience, which activity is closest to the real thing?
A. View images
B. Attend exhibit
C. Watch a demo
D. Hear
97. Teacher B is a teacher of English as a Second Language. She uses vocabulary cards, fill-in-
the-blank sentences, dictation and writing exercises in teaching a lesson about grocery
shopping. Based on this information, which of the following is a valid conclusion?
A. The teacher is reinforcing learning by giving the same information in, a variety of
methods.
B. The teacher is applying Bloom's hierarchy of cognitive learning.
C. The teacher wants to do less talk.

64

Downloaded by Noime Burtanog (noimeburtanog@gmail.com)


lOMoARcPSD|38673565

D. The teacher is emphasizing listening and speaking skills.


98. Helping in the development of graduates who are maka-Diyos is an influence of
A. naturalistic morality
B. classical Christian morality
C. situational morality
D. dialectical morality
99. From whom do we owe the theory of deductive interference as illustrated in syllogisms?
A. Plato
B. Socrates
C. Aristotle
D. Pythagoras
100. Studies in the areas of neurosciences disclosed that the human brain has limitless
capacity. What does this imply?
A. Some pupils are admittedly not capable of learning.
B. Every pupil has his own native ability and his learning is limited to this native ability.
C. Every child is a potential genius.
D. Pupils can possibly reach a point where they have learned everything.

PROFESSIONAL EDUCATION
1. Researches established that complete coordination of motor activities is attained at?
a. pre-natal stage
b. childhood stage
c. adolescence stage
d. infancy
2. Parents and teachers are considered as authorities and models by children at the early
childhood stage. What does this statement imply?
a. Parents should enforce strict discipline at home and teachers in school.
b. Teachers and parents should serve as role models at all times.
c. Teachers should demand complete obedience from the learners in school.
d. Parent-teacher conference should always be an activity in school.
3. This is the stage when the learner becomes confused and starts to experience identity crisis.
Which of these stages is it?
a. early childhood
b. early adulthood
c. late childhood
d. adolescent
4. Which of the following will trigger additional development if a child is exposed to more
challenges and stimuli in his surroundings?
a. potentials
b. intelligence
c. emotional development
d. interest
5. Which of the following is usually considered the most important factor in a child’s
observable classroom behavior?
a. intelligence
b. heredity
c. cultural background
d. self-concept
6. Social adjustment means the ability to behave in accordance with?
a. social expectations
b. stereotyped behavior
c. social unrest
d. universal norms
7. Which of these theories holds that human activity is based on the interaction of stimuli and
responses?
a. association
b. vector
c. cognitive-field
d. social-learning
8. The tendency to imitate elders is very strong in the early childhood stage. Teachers should
therefore be very good?
a. role models

65

Downloaded by Noime Burtanog (noimeburtanog@gmail.com)


lOMoARcPSD|38673565

b. counselors
c. facilitators of learning
d. disciplinarians
9. Psychologically, there is unity in all learning. However, there are distinct types of learning
that are recognized. One is affective learning that involves:
a. understanding of the external world through the use of the senses
b. formation of concepts and ideas
c. acquisition of attitudes, ideals, judgment and values
d. acquisition of facts and information
10. Identical twins are more alike than fraternal twins. Which of the following
statements/principles is supported by this?
a. Heredity has a part in determining physical appearance.
b. Intelligence is determined partly by pre-natal nutrition.
c. Environment affects both fraternal and identical twins.
d. Intelligence hinges in physical structure.
11. Children’s interests are generally largely dependent on their experiences. Which of the
following is an implication of this statement?
a. The curriculum should provide vital experiences if the school must use its
opportunities to develop interest.
b. A large part of the subject matter has to be easy if they are to be interesting and within
the learner’s experience.
c. The experience of the child reduces the area in which the school may choose to
develop interests.
d. The school should not develop interest outside the experience of the child.
12. The female gonads which are responsible for the production of egg cells are also called
________.
a. uterus
b. ovary
c. fallopian tube
d. cervix
13. It is the process by which an organism inherits the characteristic traits of the parents.
a. heredity
b. maturation
c. development
d. fertilization
14. The child’s concept of right and wrong is based on external criteria laid down by adults
during the stage of pre-conventional morality. This is based on the ideas of?
a. Erikson
b. Freud
c. Kohlberg
d. Piaget
15. During the pre-operational stage, language skill emerges and the child uses words to
represent ideas. This theory is attributed to ________.
a. Wallace
b. Piaget
c. Kohlberg
d. Dewey
16. When the daughter is competing with the mother for the father’s attention, the daughter is
said to be experiencing ________.
a. electra complex
b. identity crisis
c. oedipus complex
d. sexual deviation
17. Denzel’s mother noticed that her son always wants to get her attention and is jealous of his
father. Denzel then can be classified under what psychoanalytic stage of development?
a. latency
b. genital
c. phallic
d. sexual deviation
18. Classical conditioning theory is always attributed to him for his experiment involving the
dog’s salivation as a reaction to the sound of the buzzer.
a. Bandura

66

Downloaded by Noime Burtanog (noimeburtanog@gmail.com)


lOMoARcPSD|38673565

b. Lewin
c. Pavlov
d. Skinner
19. Section 5, Article XIV, of the Constitution states that academic freedom shall be enjoyed in:
a. state colleges and universities
b. all institutions of higher learning
c. public assemblies
d. all levels of learning
20. As provided for in the Education Act of 1982, how are the institutions of learning
encouraged to set higher standards of quality over and above the minimum required for state
recognition.
a. academic freedom
b. continuing professional education
c. formal education
d. voluntary accreditation

21. Which if these statements regarding professional teachers is the major difference in the
professionalization of teachers and teaching as promulgated in Presidential Decree 1006 and
Republic Act 7836?
a. Assigned at the tertiary level in both private and state colleges and universities.
b. Assigned at the elementary and secondary levels in both public and private schools.
c. Holder of valid professional licensure and certificate of recognition.
d. Appointed on full-time basis and on a permanent status.
22. Education is a continuous process of experiencing and of receiving or reorganizing
experiences, according to John Dewey. Which of the following situations is apt to happen in
education as influenced by Dewey’s philosophy?
a. Education takes place in the school where the individual is exposed to specific, self-
contained experiences.
b. Education may take place anywhere and anytime the individual desires.
c. Education is never completed and goes on throughout life.
d. Education may take place formally or informally to enable the individual to grow.
23. The main function of philosophy of education is to ________.
a. Define the goals and set the direction for which education is to strive.
b. Aid the learner to build his own personal philosophy.
c. Reconsider existing educational goals in the light of society’s needs.
d. Provide the academic background prerequisite to learning.
24. A teacher who subscribe to the pragmatic philosophy of education believes that experience
should follow learning. In her teaching, she therefore exerts effort in?
a. Requiring learners’ full mastery of the lessons.
b. Encouraging learners to memorize factual knowledge.
c. Equipping learners with the basic abilities and skills.
d. Providing learners opportunities to apply theories and principles.
25. “The control and the administration of all educational institutions shall be vested in the
citizens of the Philippines” is stipulated in ________.
a. P.D. 6-A
b. P.D. 1006
c. P.D. No. 176
d. 1987 Constitution
26. The first kindergarten also known as “a garden where children could grow” was the product
of research by ________.
a. Pestalozzi
b. Herbart
c. Rousseau
d. Froebel
27. Which of the following embodies the operation “Return to the Basics”?
a. New Secondary Education Curriculum
b. National Elementary Achievement Test
c. National Secondary Achievement Test
d. New Elementary School Curriculum
28. Which of the following is a mandate of the 1987 Constitution that the state shall maintain to
fight illiteracy?

67

Downloaded by Noime Burtanog (noimeburtanog@gmail.com)


lOMoARcPSD|38673565

a. Formal and non-formal education should be free and compulsory for all children and
adults.
b. Free education in both elementary and high school and compulsory in elementary
education for all children of school age.
c. Education at all levels should be free and compulsory.
d. Free and compulsory education for both elementary and high school for all children
of school age.
29. Which of these philosophies is reflective of that Dewey’s which stresses the development of
an individual capable of reflective thinking specially that of being able to solve the problem
he faces individually or collectively?
a. Developmentalism
b. Experimentation
c. Rationalism
d. Disciplinism
30. Which of the following abilities is stresses by humanistic education?
a. Enjoy the great works of man such as the classics.
b. Learn the different philosophies of education.
c. Make man distinctly civilized, educated and refined.
d. Develop man into a thinking individual.
31. The Athenian ideal of education is to form a cultural soul in a graceful and symmetrical body.
How can this be achieved?
a. by giving more focus on health education.
b. by adapting the philosophy “know thyself”.
c. by improving emphasis on physical education.
d. by a well-balanced development of mind, body and soul.
32. The government prescribes a higher percentage on the administration of educational
institutions to Filipino citizens in order to ________.
a. Ensure the teaching of Filipino.
b. Minimize the unemployment problem.
c. Protect the rights of the citizens.
d. Produce globally competitive graduates.
33. Which of the following was the fundamental reason why John Dewey proposed a
transformation of the public school system?
a. He recognized that a traditional academic education did not serve the ideals of a
democratic society.
b. He recognized that the intellectual motivation of the students was declining.
c. He recognized that industrialization was destroying the home, neighborhood, and
church.
d. He recognized that change and innovation were essential ingredients of the
progressive movement.
34. Basic education includes secondary education. Which of the following contributed to the
establishment of secondary schools?
a. Humanistic education
b. Reformation
c. Rationalism
d. Realistic
35. His philosophy advocated a classical type of liberal education or the study of the humanities.
a. Irving Babbit
b. Jean Jacques Rosseau
c. John Dewey
d. Aristotle
36. What was the philosophy of education in the Philippines before the establishment of public
school system?
a. Idealism
b. Realism
c. Humanism
d. Pragmatism
37. Which of these options was not recommended by EDCOM?
a. provisions for special education
b. institutionalization of pre-school education
c. strengthening of Values Education
d. increasing the number of state colleges and universities

68

Downloaded by Noime Burtanog (noimeburtanog@gmail.com)


lOMoARcPSD|38673565

38. The present military training in our school curriculum is an influence of ________.
a. Greece
b. Athens
c. Sparta
d. Rome
39. The Royal Decree of December 20, 1863 established in the Philippines a system of
________?
a. high school education.
b. intermediate education.
c. higher education.
d. primary education.
40. Whose philosophy expounded the concept of the laboratory school?
a. Plato
b. John Dewey
c. Jean Jacques Rosseau
d. Irving Babbit
41. Whose philosophy influenced the present emphasis on Character Education and Values
Education in our school system?
a. Confucius
b. Tagore
c. Gandhi
d. Bonifacio
42. Who expounded on the need to study the child carefully for individualized instruction?
a. Bocaccio
b. Ascham
c. Erasmus
d. De Feltre
43. Tutoring of learners is now practiced in our school provided it is not done by their own
teacher but by an outsider. Which of these Western philosophies influenced this practice?
a. Egyptian
b. Greek
c. Roman
d. Athenian
44. Whose philosophy inspired the inclusion of Physical Education in the curriculum and the
replacement of lectures with textbooks?
a. De Feltre
b. Bocaccio
c. Petrarch
d. Erasmus
45. The present Philippine Teachers Professionalization Act had its beginning on what period of
Roman History?
a. A.D. 275-259 – government established a monopoly on education
b. A.D. 100 – A.D. 175 – government increased its subsidy for education
c. 132 B.C. – A.D. 100 – Latin literature and grammar were perfected
d. 295 B.C. – 153 B.C. – schools were elementary only
46. He suggested that competition and awards should be used to motivate people.
a. Aristotle
b. Plato
c. Herbart
d. Socrates
47. The singing of the National Anthem is an offshoot of the philosophical ideals of ________.
a. Nationalism
b. Naturalism
c. Pragmatism
d. Socialism
48. The idea of Practical Arts and Home Economics subjects for boys is the philosophy of
________.
a. Confucius
b. Gandhi
c. Rizal
d. Tagore

69

Downloaded by Noime Burtanog (noimeburtanog@gmail.com)


lOMoARcPSD|38673565

49. A popular mass education for the Filipinos under the American regime came into being upon
the passage of ________.
a. Act No. 74 in 1901
b. Education Decree of 1863
c. Monroe Education Survey of 1929
d. Royal Decree of 1865
50. An adolescent combines his ability to use deductive and inductive reasoning in realistic rules
that he can respect and live by. When he does this, how does he perceive his environment?
a. He views the world from his own perspective.
b. He sees the world through the eyes of other people.
c. He interprets events from a limited point of view.
d. He sees events apart from himself and other people.

FEATURES OF K TO 12 CURRICULUM
1. Concerned with the optimum development of the Filipino child
A. Seamless
B. Relevant and responsive
C. Enriched
D. Learner-centered
E. Decongested
2. Understanding mastery and removed unnecessarily repeated competencies
A. Seamless
B. Relevant and responsive
C. Enriched
D. Learner-centered
E. Decongested
3. Transition between grade levels and continuum competencies through spiral progression
A. Seamless
B. Relevant and responsive
C. Enriched
D. Learner-centered
E. Decongested
4. Caters on Filipino learners and needs of the community
A. Seamless
B. Relevant and responsive
C. Enriched
D. Learner-centered
E. Decongested
5. Integrative, Inquiry-based, and constructive approach
A. Seamless
B. Relevant and responsive
C. Enriched
D. Learner-centered
E. Decongested
ALTERNATIVE DELIVERY MODES
6. One teacher for 2 to 4 different grade levels in a single class
A. Multigrade Instruction
B. Modified In School Off School Approach (MISOSA)
C. Instructional Management by Parents, Community, and Teachers (IMPACT)
D. Effective and Affordable Secondary Education (EASE)
E. Open High School Program (OHSP)
7. Given to high school students who incur long term absences or who are permanently unable
to attend school due to time, distance, or physical impairment.
A. Multigrade Instruction
B. Modified In School Off School Approach (MISOSA)
C. Instructional Management by Parents, Community, and Teachers (IMPACT)
D. Effective and Affordable Secondary Education (EASE)
E. Open High School Program (OHSP)
8. Frees children from the four corners of classroom wherein a class is divided into two groups.
Group 1 goes to school and group 2 learns at home or vice versa.
A. Multigrade Instruction
B. Modified In School Off School Approach (MISOSA)

70

Downloaded by Noime Burtanog (noimeburtanog@gmail.com)


lOMoARcPSD|38673565

C. Instructional Management by Parents, Community, and Teachers (IMPACT)


D. Effective and Affordable Secondary Education (EASE)
E. Open High School Program (OHSP)
9. This cater advanced students whose learning needs are not met by conventional learning
system.
A. Multigrade Instruction
B. Modified In School Off School Approach (MISOSA)
C. Instructional Management by Parents, Community, and Teachers (IMPACT)
D. Effective and Affordable Secondary Education (EASE)
E. Open High School Program (OHSP)
10. This was developed by SEAMEO INNOTECH where parents, community and teachers
collaborate to address the large number of dropouts.
A. Multigrade Instruction
B. Modified In School Off School Approach (MISOSA)
C. Instructional Management by Parents, Community, and Teachers (IMPACT)
D. Effective and Affordable Secondary Education (EASE)
E. Open High School Program (OHSP)
11. This refers to the customized course approved by CHED and PRC to deliver to Higher
Education Institutions (HEIs).
A. Revised Teacher Education Program
B. Accelerated Teacher Education Program
C. Higher Education Training Program
D. Tertiary Teacher Education Program
12. The Learner Reference Number (LIS) is composed of how many digits?
A. 6
B. 8
C. 10
D. 12
13. How about the School ID?
A. 6
B. 8
C. 10
D. 12
14. This refers to the permanent record showing historical academic and co-curricular record of
students.
A. Form 137
B. Form 138
C. School Form 1
D. School Form 2
15. This refers to the report card given every school year.
A. Form 137
B. Form 138
C. School Form 1
D. School Form 2
16. This refers to a group of teachers and other personnel engaged in collaborative learning
session to solve shared challenges.
A. Learning Action Cell
B. Teacher’s Quality Circle
C. Mentoring
D. Peer sessions
17. This refers to a set of knowledge, skills, and attitudes (KSA) required to carry out
successfully assigned roles, functions, and responsibilities in an organization.
A. Learning and Development System
B. School Based Management
C. Competency Standards
D. d. Human Resource Development
18. This refers to the range of activities that cover Continuing Professional Development aimed
at improving individual and organizational productivity.
1. Learning and Development System
2. School Based Management
3. Competency Standards
4. Human Resource Development

71

Downloaded by Noime Burtanog (noimeburtanog@gmail.com)


lOMoARcPSD|38673565

19. This refers to professional development activities shall be developed, managed, quality
assured, monitored and evaluated.
A. Learning and Development System
B. School Based Management
C. Competency Standards
D. Human Resource Development
20. These are output-driven undertakings with specific calendar schedule and resource
assignment.
A. Learning Outcomes
B. Activities
C. Learning Resources
D. School Projects
21. This refers to the inculcation of advanced KSV in post-licensure specialization for
professional practice.
A. Post-Graduate Studies
B. Professional Oath
C. Teacher Education Development Program
D. Continuing Professional Development
22. Considers K to 12 Program as a flagship reform strategy
A. BESRA
B. CHED
C. DepEd
D. TESDA
23. This assesses grade 1 learners who may exhibit developmental advancement or delays.
A. Multi-factored Assessment Tool
B. School Readiness Test
C. Early Language, Literacy, and Numeracy Assessment
D. Developmentally Appropriate Practice
24. In SEC 2010, Math and Science are taught using what approach?
A. Discipline-based
B. Spiral progression
25. The grading system used before the implementation of DepEd Order no. 8 series of 2015.
A. Knowledge, Process, Understanding, and Product/Performance (KPUP
B. Written Work, Performance Task, and Quarterly Assessment
C. Understanding by Design
D. OBE
26. This tool measures the readiness of kindergarten pupils upon entering Grade 1.
A. Kindergarten Catch-Up Education Program
B. Early Language, Literacy, and Numeracy Assessment
C. School Readiness Assessment Examination
D. School Readiness Test
27. What school form is used by teachers to register the pupils/students in the Learner
Information System (LIS)?
A. SF 1
B. SF 2
C. SF 3
D. SF 4
28. The following schools are the area of study of SEAMEO INNOTECH in the process of
reviewing the Philippine curriculum before the implementation of the K to 12 Program
EXCEPT
A. Manila Science High School
B. Don Bosco Technical College
C. San Pedro Relocation Center National High School
D. Polytechnic University of the Philippines
29. BESRA is composed of how many Key Reform Thrusts (KRTs)?
A. 3
B. 5
C. 7
D. 9
30. This refers to the complete assessment in basic education to measure attainment in K to 12
program.
A. National Basic Education Competency Assessment (NBECA

72

Downloaded by Noime Burtanog (noimeburtanog@gmail.com)


lOMoARcPSD|38673565

B. End of Grade 12 Assessment


C. College Readiness Test
D. Exit Assessment
31. This is the official list of teacher applicants who obtained a score of 70 and above.
A. Learning Action Cell (LAC)
B. Registry of Qualified Applicants (RQA)
C. Key Result Areas (KRA)
D. Key Reform Thrust (KRT)
32. This refers to the unconditional means by which students learn in formal education.
A. Alternative Learning System
B. Alternative Delivery Mode
C. Multigrade Instruction
D. Modified In School Off School Approach
33. Early Language, Literacy, and Numeracy Assessment is administered in what grade level?
A. Grade 1
B. Grade 3
C. Grade 6, 10, and 12
D. Grade 9
34. This program in public schools aim to provide additional subject to Arabic Language and
Values Education
A. Madrasah Education
B. Arabic Emergent Reading
C. Arabic Language and Islamic Values Education (ALIVE
D. Language Enhancement and Pedagogy
35. In what grade are exploratory subjects taken in the subject TLE?
A. Grade 7 and 8
B. Grade 8 and 9
C. Grade 9 and 10
D. Grade 11 and 12
36. K to 12 curriculum is to holistic development as BEC and SEC is to
A. Functional literacy
B. Discipline-based
C. Core curriculum
D. Concept development
37. The MTB-MLE is composed of how many languages?
A. 10
B. 11
C. 12
D. 13
38. Who was tasked to review the Philippine curriculum before the implementation of the K to
12 program?
A. DepEd
B. PRC
C. SEAMEO INNOTECH
D. PAFTE
39. The following are curricular themes for Kindergarten in the K to 12 curriculum EXCEPT
A. Myself
B. My family
C. My friends
D. My community
40. The math curriculum is composed of how many content areas?
A. 3
B. 4
C. 5
D. 6
41. Assessment for Kindergarten
A. Numerical Grades
B. Letters
C. Checklist and Anecdotal record
D. Anecdotal record
K TO 12 GRADING SYSTEM
41. How many percent is given to written work in Language, AP, and ESP for Grade 1-10?

73

Downloaded by Noime Burtanog (noimeburtanog@gmail.com)


lOMoARcPSD|38673565

A. 30%
B. 40%
C. 20%
D. 50%
42. How many percent is given to written work in Math and Science for Grade 1-10?
A. 30%
B. 40%
C. 20%
D. 50%
43. How many percent is given to written work in MAPEH, EPP, and TLE for Grade 1-10?
A. 30%
B. 40%
C. 20%
D. 50%
44. How many percent is given to performance task in MAPEH, EPP, and TLE for Grade 1-10?
A. 40%
B. 50%
C. 60%
D. 30%
45. How many percent is given to performance task in Math and Science for Grade 1-10?
A. 40%
B. 50%
C. 60%
D. 30%
46. How many percent is given to performance task in Language, AP, and ESP for Grade 1-10?
A. 40%
B. 50%
C. 60%
D. 30%
47. How many percent is given to quarterly assessment in all the subjects for Grade 1-10?
A. 10%
B. 20%
C. 30%
D. 40%
48. How many percent is given to written work in all core subjects for Senior High School?
A. 10%
B. 25%
C. 15%
D. 50%
49. How many percent is given to performance task in all core subjects for Senior High School?
A. 10%
B. 25%
C. 15%
D. 50%
50. How many percent is given to quarterly assessment in all core subjects for Senior High
School?
A. 10%
B. 25%
C. 15%
D. 50%
51. In what year was K to 12 offered to incoming grade 1 and grade 7?
A. 2011-2012
B. 2012-2013
C. 2013-2014
D. 2016-2017
52. In what year did the government put infrastructure to provide Senior High School?
A. 2011-2012
B. 2012-2013
C. 2013-2014
D. 2016-2017
53. If a student failed to pass 3 subjects, he/she will
A. retain in the same grade level

74

Downloaded by Noime Burtanog (noimeburtanog@gmail.com)


lOMoARcPSD|38673565

B. take remedial classes


C. move to the next grade level
D. transfer to other school
54. If a student failed to pass 2 subjects, he/she will
A. retain in the same grade level
B. take remedial classes
C. move to the next grade level
D. transfer to other school
55. If a student passed all the subjects, he/she will
A. retain in the same grade level
B. take remedial classes
C. move to the next grade level
D. transfer to other school
56. What assessment is given to Grade 9 students to determine aptitude and occupational
interest?
A. Career Assessment
B. Specialization Assessment
C. Exit Assessment
D. College Readiness Assessment
57. What assessment is given to Grade 6, 10, and 12 students before they can be promoted to the
next grade level?
A. Career Assessment
B. Specialization Assessment
C. Exit Assessment
D. College Readiness Assessment
58. This refers to the subsidy given by the State to Grade 10 completers.
A. Educational Service Contracting Grant
B. Voucher Program
C. Performance Based Incentive
D. SHS Scholarship
59. The contents of various learning areas in the Senior High School are based on what
standards?
1. Moving-Up Standards
2. College Readiness Standards
3. Employment Readiness Standards
4. Higher Education Standards
60. What agency formulated the standards of the various learning areas in the Senior High
School?
A. PRC
B. CHED
C. DepEd
D. TESDA

Part 1 General Education


150 QUESTIONS With ANSWERS
1. Rennie Boy believes that everything that happens in this world has a cause or a reason. What
scientific attitude is seen in his actions?
A. curiosity
B. firm set of beliefs
C. honesty
D. critical-mindedness
2. _____ it never recycled any waste materials; the company’s environmental record was not
perfect.
A. For
B. In that
C. With
D. Due to
3. Our guest of honor was a loquacious speaker. Loquacious means:
A. reserved
B. verbose
C. reticent
D. spiritless

75

Downloaded by Noime Burtanog (noimeburtanog@gmail.com)


lOMoARcPSD|38673565

4. The sociological concept of home for most Filipino is the presence of _____.
A. families with boarders in addition to the family members
B. nuclear family and extended families
C. families with illegal light and water connections
D. families with pet dogs and cats
5. "A man is known by the company he keeps." This is an example of a _____.
A. report
B. rule
C. poem
D. proverb
6. Which of the following is the ultimate purpose of Human Right Education?
A. to identify threats of human rights
B. to generate awareness of one’s basic rights
C. to check on abuses in the exercise of human rights
D. to understand and change conditions which give rise to human rights violation
7. The members of the federation of cooperatives are _____.
A. primaries
B. secondaries
C. natural persons
D. primary and secondary cooperatives
8. Be sure to fill _____ the forms correctly.
A. over
B. in
C. on
D. with
9. Vascular tissue that transports water in leaves connects directly to which of the following?
A. meristems in the root tip
B. pistils in the flower
C. root hairs in the epidermis
D. xylem in the stem
10. Which statement is true?
A. 5–√ is a rational number.
B. b. 0 is a positive number
C. 3–√ is a real number.
D. 4–√ is an irrational number
11. If a die is rolled, what is the probability of getting a number divisible by 2?
A. 1/6
B. 1/2
C. 1/4
D. 1/3
12. Why do mothers usually soak meat in pineapple juice before cooking it?
A. It makes meat tender.
B. It removes the bad odor.
C. It improves the taste of meat.
D. It preserves the natural taste of meat.
13. What is considered the earliest form of writing in the Philippines?
A. Abakada
B. Roman alphabet
C. Alibata
D. Balitaw
14. Most asteroids are found between the two planets:
A. Venus and Earth
B. Earth and Mars
C. Mars and Jupiter
D. Jupiter and Saturn
15. A wood frame for pouring concrete has an interior perimeter of 14 meters. Its length is one
meter greater than its width. The frame is to be braced with twelve-gauge steel cross-wires.
Assuming an extra half-meter of wire is used at either end of a cross-wire for anchoring,
what length of wire should be cut for each brace?
A. 6 m
B. 7 m
C. 8 m

76

Downloaded by Noime Burtanog (noimeburtanog@gmail.com)


lOMoARcPSD|38673565

D. 12 m
16. New students at the shop should bring _____.
A. hammers wrenches and screwdrivers
B. hammers, wrenches, and screwdrivers
C. hammers, wrenches and screwdrivers
D. hammers wrenches, and screwdrivers
17. The army’s retreat was IGNOMINICUSFUL. This means _____.
A. reputable
B. honorable
C. shameful
D. mysterious
18. Ibigay ang tayutay na nagamit sa sumusunod: Ikaw ang payong ng aking buhay - silungan ng
init - hatid ay proteksyon sa panahong masungit
A. simile
B. personipikasyon
C. metapora
D. paglilipat-wika
19. Which of the following BEST demonstrates the greenhouse principle?
A. a heated aquarium
B. a car with rolled-up windows
C. a microwave oven
D. a solar battery -powered calculator
20. The bank was swarmed with depositors. The bank ______
A. declared an open house
B. declared dividends
C. declared a holiday
D. declared bankruptcy
21. The Rizal Day celebration reminds us about heroes’ worth _____.
A. appreciating
B. emulating
C. reading about
D. studying
22. Which among the following represents the smallest unit of life?
A. embryo
B. organelle
C. cell
D. atom
23. What is the equation of the line whose slope is -2 and whose y-intercept is 3?
A. 2x +3y =6
B. x +2y =3
C. 2x +2y =4
D. 2x +y =3
24. Nag-aaral ako _____ mabuti upang makakuha ako ng iskolarship.
A. nang
B. ng
C. mas
D. lalong
25. Two air masses have the same humidity. Which of the following can also be said about them?
A. They have the same temperature.
B. They have the same volume
C. Both have rain clouds.
D. none of the above
26. _____ I help you, sir?
A. Might
B. Could
C. May
D. -
27. What measure of central tendency can best describe the size of t-shirts commonly used by
teenagers?
A. mean
B. median
C. mode

77

Downloaded by Noime Burtanog (noimeburtanog@gmail.com)


lOMoARcPSD|38673565

D. both a and c
28. _____ tell her professor next time.
A. She does
B. She
C. She'll
D. She's
29. Wise people can _____ with frustrations.
A. cope in
B. cope up
C. cope
D. cope on
30. Which enables the computer to communicate with other computers over the telephone?
A. network
B. modem
C. internet
D. hypermedia
31. Which of the following is expressed clearly, effectively states the idea, and has no structural
error?
A. I learned the distance of the sun from the earth was 90 million miles.
B. I learned that the sun was 90 million miles from the earth.
C. I learned that the sun is 90 million miles from the earth.
D. I learned that the earth and the sun are 90 million miles apart.
32. Basically, the Philippine agrarian reform is a question of _____.
A. land distribution
B. nepotism and corruption
C. graft and corruption
D. environmental degradation
33. I really object _____ in rooms where other people have to eat.
A. people smoking
B. to people smoke
C. people to smoke
D. to people smoking
34. What is the exact date of Rizal’s birth?
A. June 19, 1862
B. June 19, 1858
C. June 19, 1861
D. June 19, 1860
35. Given the following scores of 70, 95, 60, 80, and 100, what can you infer from the measures
of variability that can be obtained from this?
A. The population is very homogeneous.
B. The measures are very unstable.
C. The grades are very scattered.
D. The range of scores is a very reliable measure of variability.
36. Which one of the following is a result of compressing a file?
A. The file is deleted.
B. The file size is smaller.
C. The three-character extension is hidden.
D. All file properties are lost.
37. What is the measure of an interior angle of a regular dodecagon?
A. 120 degrees
B. 130 degrees
C. 140 degrees
D. 150 degrees
38. 1/5 of the width and 1/4 of the length of a rectangular cardboard is cut off. What part of the
original cardboard is the area of the remaining piece?
A. 30% of the original area
B. 40% of the original area
C. 50% of the original area
D. 60% of the original area
39. Which is the "brain" of a computer?
A. CPU
B. peripheral

78

Downloaded by Noime Burtanog (noimeburtanog@gmail.com)


lOMoARcPSD|38673565

C. software
D. RAM
40. Which one of the following is an appropriate routine to shut down a non-responding
application?
A. Close all open documents and press the power button until the computer
automatically switches off.
B. Press "Ctrl + Alt + Delete", then select the non-responding application in the Task
Manager window and click "End Task".
C. Pull out the power cable from the back of the computer.
D. Click "Shut Down" on the on the Start menu and choose "Shut down" from the Shut
Down Windows drop-down list.
41. Why do fisherfolks catch more fish during the new moon than during the full moon?
A. Fishes are more active during the new moon because of the salt content of water is
just right.
B. Fishes look for mates during the new moon.
C. Fishes go near the surface during the new moon because the water is warmer.
D. Fishes are attracted to the light from the fishing boat which fishermen make use
during the new moon.
42. "Mahal kita, mahal kita, hindi 'to bola/ Ngumiti ka man lang sana ako'y nasa langit na." Ang
salitang pampanitikan na ginamit sa linya ng kanta ay nangangahulugang _____.
A. napakasaya
B. pakikipagkaibigan
C. sinisinta
D. laruan
43. Which is the method of growing vegetables and low plants in a nutrient-packed aerated
solution with no soil at all?
A. vegetative cultivation
B. natural farming
C. hydroponics
D. aerial cultivation
44. Which part of the brain is responsible for memory and intelligence?
A. cerebellum
B. cerebrum
C. medulla oblongata
D. thalamus
45. How can the Comprehensive Agrarian Reform Program bring about social equity and
economic prosperity in the country?
I. By distributing the agricultural lands to landless farmers free of charge
II. By transferring the ownership of land to farmers for a value
III. By helping the affected landowners to use their land compensation in viable business
activities
IV. By providing support services to farmer-beneficiaries
A. II, III, and IV
B. I, II, and IV
C. I, II, and III
D. I, III, and IV
46. You are a member of a cooperative which operates taxis, tricycles, and buses. Of which type
of a cooperative are you a member?
A. service cooperative
B. multipurpose cooperative
C. marketing cooperative
D. producers cooperative
47. Which among the following is an example of a transfer payment?
A. profit
B. rent
C. unemployment benefits
D. government purchases
48. Adolfo does not allow his feelings and biases to influence his recording of observations,
interpretation of data, and formulation of conclusions. What scientific attitude is reflected in
his actions?
A. absence of bias
B. concern for the environment

79

Downloaded by Noime Burtanog (noimeburtanog@gmail.com)


lOMoARcPSD|38673565

C. rationality
D. objectivity
49. If a baseball player hits 10 home runs in the first 45 games, at the same rate how many home
runs can he expect to hit during the 162-game season?
A. 38
B. 42
C. 36
D. 40
50. Which method of reproduction provides for the most variety of offspring?
A. cloning
B. sexual reproduction
C. asexual reproduction
D. cellular reproduction
51. The following EXCEPT one are experiences that help students build a global orientation.
A. Teach about the United Nations and its specialized agencies
B. Explore some of the reasons for the differences in ways of living
C. Conduct training with schools from other countries
D. Promote the development of stereotypes and ethnic bias
52. Alin ang paksa sa sumusunod na pangungusap? Ginagawa niya ang pagdarasal araw-araw.
A. ginagawa
B. araw-araw
C. pagdarasal
D. niya
53. What instrument is used to measure the intensity of earthquakes?
A. barograph
B. thermograph
C. seismograph
D. hygrograph
54. Excuse me, can I borrow _____ dictionary?
A. your
B. yours
C. you
D. a
55. Alin ang di karaniwang anyo ng pandiwang HINTAY KA?
A. tay
B. tayka
C. intay
D. teka
56. Alin sa mga sumusunod ang aral na ibinigay ng ANG ALAMAT NI MARIANG MAKILlNG
na ikinuwento ni Jose Rizal?
A. Pagyamanin at pangalagaan ang ating bayan at lahi pagka’t hiyas at yaman natin ito.
B. Pag-ibig ang makapagbabago sa mundong ito.
C. Kabanalan ang magpatawad at tulungan ang isang nagkasala.
D. Dahil sa pagmamalabis at pagsasamantala, maraming biyaya ang sa kanya'y
nawawala.
57. Which is the main goal of drug abuse education?
A. arrest
B. prevention
C. control
D. rehabilitation
58. The amount of substance having 6.02 x 10^23 of any kind of chemical unit is called a/an
_____.
A. mass number
B. mole
C. formula
D. atomic weight
59. How many ways can a committee of 4 people be selected from a group of 7 people?
A. 35
B. 70
C. 140
D. 210
60. A rectangle is 4 times a long as it is wide. If the length of the third side?

80

Downloaded by Noime Burtanog (noimeburtanog@gmail.com)


lOMoARcPSD|38673565

A. 2 x 32
B. 4 x 16
C. 3 x 14
D. 5 x 12
61. What law was passed by Philippine Congress in 1995 which affirms the total integration of
persons with disabilities into the mainstream of society?
A. R.A. 7277
B. R.A. 7784
C. R.A. 7722
D. R.A. 7776
62. Far _____ Terry’s small plane lay the freezing waters of the North Sea, where his father’s
boat had disappeared many years before.
A. down
B. under
C. underneath
D. below
63. To reduce electronic waste, implement a recycling system. Which belong to an e-waste
recycling system?
I. Repair
II. Reuse
III. Upgrading of existing equipment
A. I, II, and III
B. I only
C. I and II
D. II only
64. A molecule or an ion is classified as a Lewis base if it has which of the following properties?
A. donates a proton to water
B. accepts a pair of electrons
C. has resonance structures
D. accepts a proton from water
65. Kailan natin ipinadiriwang ang Lingo ng Wika?
A. Hunyo 13-19
B. Agosto 13-19
C. Agosto 1-31
D. Marso 13-19
66. Excuse me, _____ the time?
A. have you
B. have you got
C. got you
D. do you
67. Nowadays, _____ are much better behaved than their parents were at the same age, don’t you
think?
A. teenagers
B. the teenagers
C. a teenager
D. teen an ager
68. If x = 1 and y = -2, what is the value of the expression, x22xy−4x+3xy2?
A. -9/2
B. 15/2
C. -7/2
D. 17/4
69. Which among the following statements is always true?
A. The supplement of an angle is acute.
B. The complement of an angle is obtuse.
C. The complement of any acute angle is acute.
D. Two supplementary angles are congruent.
70. A losing senatorial candidate may file a protest against the winner. What body will assume
jurisdiction over the case?
A. Supreme Court
B. Electoral Tribunal
C. COMELEC
D. Regional Trial Court

81

Downloaded by Noime Burtanog (noimeburtanog@gmail.com)


lOMoARcPSD|38673565

71. Ano ang pormasyon na pantig sa mga titik na "prin" sa salitang prinsesa?
A. KKPK
B. KKP
C. KPK
D. KP
72. The Philippines belongs to the type of biome called _____.
A. deciduous forest
B. taiga
C. tropical rainforest
D. grassland
73. A square has an area of 400 sq. cm. What is its perimeter?
A. 20 cm
B. 80 cm
C. 100 cm
D. 40 cm
74. If 500 or 25% of a graduating class are girls, how many are graduating?
A. 2000
B. 5000
C. 10000
D. 20000
75. What type of thermo-nuclear reaction occurs in the sun which generates a tremendous
amount of heat energy?
A. fission
B. radiation
C. convection
D. fusion
76. Nobel Prize winners are chosen from different levels. Where was Pres. Quirino considered?
A. politics
B. literature
C. human rights
D. economics
77. Organisms that eat plants only are called _____.
A. ovipores
B. herbivores
C. omnivores
D. carnivores
78. Piliin ang salitang may diptonggo.
A. yoyong
B. kamay
C. lawa
D. tihaya
79. How many prime numbers are there between 1 and 100?
A. 23
B. 24
C. 25
D. 26
80. You cannot share what you do not have, Who portrayed it best?
A. Heaven is built on stilts. One can buy all happiness.
B. The Man scorned on the Cross not redeem the world without suffering.
C. Patience on the rice store cue was tested by a grumbling lady.
D. Words without deeds can never go far.
81. When a person shows the ability to understand and appreciate the similarities and differences
in the customs, values, and beliefs of one's culture, that person is said to _____.
A. possess multicultural literacy
B. possess sociocultural awareness
C. possess global awareness
D. possess social literacy
82. Mr. Lucido deposited 225 pesos in the bank. If his deposit consisted of 29 bills, consisting of
5 pesos, and 10 pesos bills, how many 10 pesos bills did he deposit?
A. 13
B. 14
C. 15

82

Downloaded by Noime Burtanog (noimeburtanog@gmail.com)


lOMoARcPSD|38673565

D. 16
83. Marumi ang baso. Huwag mong ____ yan!
A. inumin
B. inuman
C. uminom
D. pag-inuman
84. When you carry a heavy load with one arm, you tend to hold your free had away from your
body in order to _____.
A. change the weight of your body and load
B. change the mass of your body
C. be ready to grab something in case you fall
D. change the center of gravity of your body and load
85. If a picture frame is 27 cm long and 18 cm wide, what is the ratio of its length to its width?
A. 3:2
B. 2:3
C. 3:5
D. 5:3
86. When a teacher is charged with administrative case committed in the lawful discharge of
professional duties, what right may the teacher invoke for their defense?
A. right to receive compensation in the duration of the case
B. right to be given the due process of law
C. right to be defended by the organization of teachers to which they are a member
D. right to be provided with free legal service by the appropriate office
87. In how many ways can a team of 10 basketball players be chosen from 12 players?
A. 62
B. 64
C. 66
D. 72
88. Find m in the proportion m/12 = 30/24.
A. 3
B. 15
C. 20
D. 25
89. People who are too _____ are liable to be deceived by unscrupulous individuals.
A. wary
B. credulous
C. cynical
D. demanding
90. _____ natin ng mapa si Kenny para hindi siya maligaw.
A. Iwan
B. Mag-iwan
C. Iwanan
D. Pag-iwan
91. Pinandirihan ni Mico ang aso niyang _____.
A. ma-galis
B. mag-alis
C. magalis
D. galis-aso
92. Any individual has the right to question why he is being arrested, and to summon his
accusers to court so that due process may be performed. This right is embodied in which of
the following?
A. reclusion perpetua
B. res ipsa loquitur
C. prison mayor
D. habeas corpus
93. The elevator can carry a maximum load of 605 kg. How many passengers of weight 50.5 kg
each can the elevator hold?
A. 12
B. 11.9
C. 11
D. 10
94. 4 1/5 + 3 2/7 = _____

83

Downloaded by Noime Burtanog (noimeburtanog@gmail.com)


lOMoARcPSD|38673565

A. 7 3/12
B. 7 3/35
C. 7 17/35
D. 7 1/35
95. For the fruit punch, 3050 mL of fruit juice is needed. How much fruit juice is needed in
dekaliters?
A. 0.305
B. 3.05
C. 5.03
D. 0.053
96. "Ang babae ay huwag mong tingnang isang bagay na libangan lamang kundi isang katuwang
at karamay sa mga kahirapan nitong kabuhayan." (E. Jacinto) Ang tungkulin ng wika sa linya
ay _____.
A. pangheuristiko
B. pang-interaksyunal
C. panregulatori
D. pang-instrumental
97. Anong tayutay ang tinutukoy nito? Durog ang katawang bumagsak sa semento si Miguel.
A. pagtutulad
B. pagbibigay-katauhan
C. pagmamalabis
D. pagwawangis
98. Yes, the script was excellent, but the performances were _____ weak.
A. completely
B. extremely
C. utterly
D. absolutely
99. The smallest angle of a triangle is two-thirds the size of the middle angle, and the middle
angle is three-sevenths of the largest angle. Find all three angle measures.
A. 30°, 60°, 90°
B. 45°, 45°, 90°
C. 35°, 45°, 110°
D. 30°, 45°, 105°
100. Tandaan mo na lahat ng paghihirap ni Luis ay _____ sa iyong magandang kinabukasan.
A. tungkol
B. upang
C. sanhi
D. ukol
101. Ang mga mata mo ay tulad ng bituin sa kalangitan. Ano ang ginamit na pagpapahayag
dito?
A. pagtutulad
B. personipikasyon
C. pagwawangis
D. pang-uyam
102. What science deals with the study of outer space?
A. Astronomy
B. Oceanography
C. Paleontology
D. Geology
103. To pass her English Test, Lucille must get 75% of the items correct. Out of 80 questions,
how many must she correctly answer?
A. 55
B. 60
C. 70
D. 65
104. To stimulate the nation's economy and to increase employment, former President
Diosdado Macapagal _____.
A. abolished government repressive controls
B. borrowed money from the World Bank
C. standardized the salaries of government employees
D. gave incentives to home cottage industries

84

Downloaded by Noime Burtanog (noimeburtanog@gmail.com)


lOMoARcPSD|38673565

105. The ratio of the number of red, green, and blue balls in a box is 2:5:6. How many green
balls are there if there are 52 balls in all?
A. 4
B. 8
C. 20
D. 24
106. Which of the following is expressed clearly, effectively states the idea, and has no
structural error?
A. Good teachers possess a capacity for connectedness; not in the method but in their
hearts - a place where intellect, emotion, spirit, and will converge in the human self.
B. Good teachers possess a capacity for cooperation; not in the method but in their hearts
where they knowledge, spirit and emotion converge.
C. Good teachers are capable to promote interrelationship among learners, parents,
school heads, and community.
D. Good teachers have the capacity to develop interpersonal relationship among the
factors in the learning environment.
107. We’re late. By the time we get to the cinema, the film _____.
A. will have started
B. has started
C. will start
D. will have been starting
108. One of the objectives of Philippine Education as provided in the 1987 Constitutions is to
promote respect for human rights. To give flesh to this constitutional mandate, President
Aquino issued an order requiring human rights to be integrated in the school curricula. Which
of the following specifies the said directives?
A. DECS Order No. 61
B. Executive Order No. 27
C. Presidential Decree No. 603
D. Administrative Order No. 2
109. In which part of the flower develop into seeds?
A. pistil
B. ovary
C. ovules
D. ovary wall
110. renewable
A. a resource that is inexhaustible
B. a process where one plate is forced under another
C. the blanket of air surrounding the Earth
D. describes how different types of rocks are formed
111. Which of the following is expressed clearly, effectively states the idea, and has no
structural error?
A. Albert himself told me it was true.
B. Albert, himself, told me it was true.
C. Albert told me it was true himself.
D. Albert told me himself it was true.
112. Did you have _____ on holiday?
A. a good weather
B. good weathers
C. good weather
113. A behavioral psychologist would probably say that _____.
A. introspection is a useful way to study mental processes
B. the mind is most important when analyzing behavior
C. we act based on rewards and punishments received
D. the unconsciousness stimulates our behavior
114. Sino ang naglimbag ng "El Guinto de Pueblo ng panahon ng Amerikano?
A. Sergio Osmena
B. Pascual Poblete
C. Jose Palma
D. Francisco Balagtas
115. When two missing digits in a given number are replaced, the number is divisible by 99.
What is the number?
A. 85021

85

Downloaded by Noime Burtanog (noimeburtanog@gmail.com)


lOMoARcPSD|38673565

B. 85031
C. 85041
D. 85051
116. Which of the following documents embodies the general principles of human rights
which set forth the minimum standard of living for all people pf the world?
A. International Covenant on Civil, Political Rights
B. International Covenant on socio-Economic, Cultural rights
C. Universal Declaration of Human Rights
D. United Nations Declaration on Rights to Development
117. Ibigay ang tayutay na nagamit sa sumusunod: Ilaw ng tahanan - Ikaw ang sandigan -
Lamyos ng pagmamahal - Sa pag-unlad ko'y tangan
A. metonimiya
B. sinekdote
C. onomatopeya
D. pagtawag
118. A secret word that must be entered into the computer before a person is allowed access to
information is called _____.
A. input
B. password
C. program
D. statement
119. Are you going shopping now? No, but _____ later.
A. I might go
B. I might to go
C. I might going
D. I might be go
120. The Code of Ethics of Professional Teachers provides that a teacher has the right and duty
to determine the academic marks and the promotion of learners. What is the obligation of the
teacher in relation to the aforementioned right as mandated in the Education Act of 1982?
A. Refrain from making deductions in students’ ratings for acts not of poor scholarship.
B. Refrain from discriminating high and low achievers in the classes they handle.
C. Refrain from influencing corporal punishment and make it a basic for deductions in
scholastic ratings.
D. Refrain from segregating the fast and slow learners for convenience in rating them.
121. Which of the following is expressed clearly, effectively states the idea, and has no
structural error?
A. A strange car kept blinking the lights at us as he followed closely.
B. A strange car followed closely, and he kept blinking his lights at us.
C. A strange car followed us closely, and its driver kept blinking his lights at us.
D. A strange car, blinking the lights at us, followed closely.
122. Which of the following is the catalytic converter on cars in the control technology?
A. carbon monoxide
B. carbon dioxide
C. nitrogen dioxide
D. sulfur oxide
123. What type of cooperative provides assistance to general public such as transportation,
health and housing?
A. consumer
B. producer
C. credit
D. service
124. Which is at the base of a triangle that shapes us into what we are?
A. environment
B. heredity
C. training
D. school attended
125. _____ up! Or we'll be late.
A. You hurry
B. Be hurrying
C. Hurry
D. Hurry you
126. Which of the following demonstrate sublimation?

86

Downloaded by Noime Burtanog (noimeburtanog@gmail.com)


lOMoARcPSD|38673565

A. dry ice evaporating


B. perspiration drying
C. laundry drying
D. steam forming water droplets
127. Ano ang salitang-ugat ng kanluran?
A. kalan
B. kanlong
C. lunan
D. lunod
128. A teacher wants to group her pupils into groups of 3 or 5 or 6. However, she found out
that if she do that there will always be 1 pupil left. What is the least possible number of
pupils in the class?
A. 81
B. 31
C. 21
D. 11
129. Ibigay ang aspekto ng pandiwa sa sumusunod na pangungusap. Mag-aral sa bahay ng
mga araling ukol sa agham at teknolohiya.
A. pawatas
B. kontemplatibo
C. imperpektibo
D. perperktibo
130. _____ taong masipag sa loob ng bulwagan.
A. May
B. Mayroong
C. A at B
D. Wala ang sagot sa mga pagpipilian.
131. Who has greater moral responsibility and why? The lazy master teacher who does not
teach well or the beginning teacher who is groping while she teaches and so does not teach
well either.
A. The master teacher, who is older and more experienced.
B. The master teacher, who is expected to have more knowledge and freedom.
C. Cannot be determined. Age, experience, or freedom does not necessarily make some
one more or less morally responsible than the other.
D. Both are equally morally responsible.
132. Which of the following is expressed clearly, effectively states the idea, and has no
structural error?
A. I would not want to be he.
B. I would not want to be himself.
C. I would not want to be themselves.
D. I would not want to theirselves.
133. Alin ang tamang babala sa pagtatapon ng basura?
A. Gamitin and basurahan sa inyong bahay, huwag dito
B. Dito and tambakan ng basura
C. Huwag mag-tapon ng basura dito
D. Huwag mag-ipon ng basura dito
134. Which is the MOST important perceived need and problem of the Filipino family?
A. Unemployment or financial problem
B. Proneness to vices
C. Over protectiveness of children
D. Double standard on the roles of male and female
135. Which of the following statements about living things is false?
A. All living things use energy to function.
B. All living things are composed of cells.
C. All living things have a nervous system.
D. All living things are capable of reproduction.
136. Robert Frost wrote the poem "Acquainted with the Night" from which the stanza is taken:
I have been one acquainted with the night.
I have walked out in rain-back in rain.
I have out walked the farthest city light
The poet in the stanza talks of
A. isolation and loneliness.

87

Downloaded by Noime Burtanog (noimeburtanog@gmail.com)


lOMoARcPSD|38673565

B. happiness in having been acquainted with the night.


C. joy getting out of the house.
D. youthful delight playing in the rain.
137. What form of government is characterized by the separation of powers?
A. Parliamentary
B. Presidential
C. Aristocracy
D. Monarchical
138. Which is the BEST evidence that helium gas is lighter than air?
A. Helium has the lowest boiling point of all elements.
B. Helium atoms do not combine with other air atoms.
C. Helium-filled balloons rise in air.
D. By volume, helium makes up only 0.0005% of air.
139. I _____ get up at six o'clock this morning with much difficulty.
A. could
B. managed to
C. can
140. One package is 100 pounds, and the other is 150 pounds. The weight of the second
package is how many times that of the first?
A. 1½ times heavier
B. ½ as heavy
C. none of these
D. 10 pounds heavier
141. Excursions are encouraged to allow participants to see places. All the 400 personnel fell
ill. Doctors’ laboratory findings traced the cause to _____.
A. congestion on the bus
B. food poisoning
C. excitement
D. excessive fat
142. Which of the following facts support the Big Bang theory's explanation of the creation of
the universe?
A. The universe does not expand nor contract.
B. The universe seldom expands.
C. The universe will have background radiation.
D. The universe has no beginning nor end.
143. In a university, the ratio of female professors to the male professors is 8:5. If there are 75
male professors, how many are female professors?
A. 120
B. 180
C. 225
D. 375/8
144. He_____ his leg while he _____ rugby.
A. was breaking - played
B. broke - was playing
C. breaks - is playing
145. Someone wrote: "Environment relates to the profound relationship between matter,
nature, and society, and in such a context, ICTs bring new ways of living in a more
interconnected society, all of which reduces our dependency on matter and affects our
relationship with nature." What does this convey?
A. Environment and ICT are poles apart.
B. ICT impacts on environment.
C. Environment affects ICT.
D. ICT brings us away from an interconnected society.
146. Mantle
A. the thickest layer of the Earth
B. a process where one plate is forced under another
C. the blanket of air surrounding the Earth
D. describes how different types of rocks are formed
147. "Heto na, heto na, heto na, /wahh! Doo bidoo bidoo, bidoo, bidoo." Alin ang nabuong
salita o tunog sa linya na nabuo ayon sa teoryang pooh-pooh?
A. wahh
B. heto

88

Downloaded by Noime Burtanog (noimeburtanog@gmail.com)


lOMoARcPSD|38673565

C. na
D. doo bidoo
148. A town that is located ______ is a location that should have most nearly twelve hours of
daylight and twelve hours of darkness during December.
A. halfway between the Equator and the South Pole
B. close to the Equator
C. close to the North Pole
D. close to the South Pole
149. And now for this evening’s main headline: Britain _____ another Olympic gold medal!
A. had won
B. wins
C. won
D. has won
150. Charity begins at home. What illustrates this value?
A. Millionaires send checks anonymously
B. Patrons check out donations by the size of the advertisement
C. Grandparents stay with the Home for the Aged. They are nuisance to modern families
D. An ambassador in Jeddah shelters battered Filipino OCW’s and avoid social
gatherings to be able to save food for these refugees

GENERAL EDUCATION
1. In the Pacific area, a storm is called_________.
A. Hurricane
B. Typhoon
C. El Nino
D. La Nina
2. Damage to DNA that is not repaired and then replicated can result in genetic disorders.
A. Mutation
B. Embryology
C. Pathology
D. Speciation
3. Which is true about lithosphere?
A. Crust and upper mantle
B. Continents and crust
C. Crust and water on it
D. Crust and the core
4. Which of the following is a heterotroph?
A. Algae
B. Grasshopper
C. Fern
D. Moss
5. Which is true of metalloids?
A. Conduct heat and electricity less effectively than non-metal.
B. Conduct heat and electricity better than metals.
C. Conduct heat better than metals
D. Have properties of both metals and non-metals
6. ___________ is the philosophy of our folks during the pre-Hispanic days.
A. Folk tale
B. Folklore
C. Proverbs
D. Ordeal
7. Right to life, liberty and genuine happiness is fundamental to:
A. Historical rights
B. Civil rights
C. Political rights
D. Constitutional rights
8. What problem can result owing to bias among parents and the youth towards manual or
technical occupations, e.g., Masons, mechanics, plumbers, etc.?
A. Boss mentality
B. White collar mania
C. Blue collar mania
D. Technocrat mind

89

Downloaded by Noime Burtanog (noimeburtanog@gmail.com)


lOMoARcPSD|38673565

9. Who was the first man to circumnavigate the world?


A. Ferdinand Magellan
B. Ruy Lopez de Villalobos
C. Enrique of Malacca
D. Sebastian Elcano
10. The third and the last military governor of the Philippines was.
A. Gen. Arthur MacArthur
B. Gen. Wesley Merritt
C. Gen. Elwell Otis
D. Gen. Douglas MacArthur
11. What caused the fury of Filipinos during the ruthless reign of Gov. Gen. Rafael Izquierdo?
A. Demolition of Masonic sites
B. Execution of Dr. Jose Rizal
C. Execution of GomBurZa
D. Uncovering the propaganda movement
12. __________rejected Aguinaldo as president
A. Acta de Katipunan
B. Acta de Tejeros
C. Acta de Malolos
D. Acta de Propaganda
13. In the 19th century the new _________spring, the leaders of the propaganda which sowed the
seeds of the Philippine revolution.
A. Upper Class
B. Middle Class
C. Lower class
D. First class
14. Spanish educational degree of 1863 provides ___________.
A. High school/secondary education
B. Primary education
C. Tertiary education
D. Free education
15. Which Katipunan member commuted from Cavite to Manila to buy materials use to make
ammunitions?
A. Teresa Magbanua
B. Teodora Alonso
C. Trinidad Tecson
D. Agueda Esteban
16. The term “frailocracia” which refers to the hidden control and domination of the religious
leaders on the colonial government was given by
A. Apolinario Mabini
B. Graciano Lopez Jaena
C. Marcelo Del Pilar
D. Jose Rizal
17. What is meant by Rizal to the Philippine and to the world? Rizal chose peaceful movements
to fight Spain. Rizal is a model of__________?
A. Rebellion
B. Nonviolence
C. Armed struggle
D. Subservience
18. He is the speaker of the Philippine Assembly
A. Emilio Aguinaldo
B. Manuel Quezon
C. Elpidio Quirino
D. Sergio Osmena
19. During the 17th-18th century Hispanic era, what was the highest post a native or Chinese
mestizo can attain at the municipal level, equivalent to local bureaucrats namely the mayors
today?
A. Gobernadorcillo
B. Tenientes de justicia
C. Cabeza de barangay
D. Directorcillo

90

Downloaded by Noime Burtanog (noimeburtanog@gmail.com)


lOMoARcPSD|38673565

20. What is the overriding principle that calls to the impeachment of impeachable officials such
as the president according to the Philippine constitution?
A. Corruption
B. Nepotism
C. Loss of public trust
D. Hidden wealth
21. Ibigay ang salin ng: A stitch in time saves nine.
A. Maagap
B. Eksaherado
C. Bale-wala
D. Magtipid
22. Mahaba ang pahayag ni Socrates kaya kailangan iyong mahalaga na lamang ang dapat na
mabasa kaya gumamit siya ng
A. Sintesis
B. Abstrak
C. Direktang sipi
D. Ellipsis
23. Ang bantas na palugit ay ginagamit sa_________.
A. Komunikasyong di-pasalita
B. Pagsulat na pahayag
C. Pagbigkas na pahayag
D. Pagsama ng pagsulat at pagbigkas na pahayag
24. Tumutulong ito sa mga salitang nakapag -iisa at may kahukugan. Kilala rin itong salitang-
ugat.
A. Paglalapi
B. Di-malayang ponema
C. Malayang ponema
D. Morponemang leksikal
25. __________ang katiwaliang ginagawa ng opisyal ng gobyerno na paggamit ng pera ng bayan
para sa pansariling kapakanan.
A. Paglustay
B. Pagnanakaw
C. Pang-aabuso
D. Panloloko
26. Pakpak, gamugamo, dibdib at paruparo ay mga halimbawa ng salitang.
A. Payak
B. Tambalan
C. Inuulit
D. Maylapi
27. Ito ay isang dulang nagsasalaysay ng buhay at kamatayan ng ating Mahal na Poong
Hesukristo at nagsasabing PASYONG itinahanghal sa entablado.
A. Moro-moro
B. Karagatan
C. Senakulo
D. Duplo
28. __________ piraso ang dala niyang pasalubong.
A. Labingwalong
B. Labimwalong
C. Labinwalong
D. Labing-walong
29. Kung sa pakikinig ang mga layunin malibang ay mauuri bilang apresyatib, mauuri naman sa
pagbabasa bilang.
A. Mapanuri
B. Scanning
C. Intensibo
D. Ekstensibo
30. Alin ang pinakamahabang antas ng wika na impormal na nilikha sa pagsam-sama ng mga
salitang pinaikli o pinahaba?
A. Kolokyal
B. Balbal
C. Lalawiganin
D. Pampanitikan

91

Downloaded by Noime Burtanog (noimeburtanog@gmail.com)


lOMoARcPSD|38673565

31. Umaawit na puso


A. Pagmamalabis
B. Pagwawangis
C. Personipikasyon
D. Palit-tawag
32. Pahayag na pasaklaw o nabubuo sa pamamagitan ng personal na panlasa o pagpili.
A. Probable generalization
B. Analytical generalization
C. Subjective generalization
D. Categorical generalization
33. Ang nalaglag-nahulog ay nasa aspetong
A. Magkahawig
B. Magkapares
C. Magkasalungat
D. Magkasintunog
34. Saan tumatalakay ang akdang “Ang Sampaguitang Walang Bango”?
A. Pagtataksil sa asawa
B. Mapanlinlang sa kapwa
C. Nasalantang sampaguita
D. Kawalang malay ng mga kababaihan
35. Aling salita ang may klaster?
A. Palma
B. Pluma
C. Basta
D. Basket
36. Alin sa mga sumusunod ang pangungusap na may paksa.
A. Nagbabasa sila sa aklatan
B. Walang tubig kahapon
C. Kay ganda ng paglubog ng araw
D. Magandang umaga
37. Uri ng pagbabagong morponemiko na gumagamit ng pagpapalit ng posisyon ng ponema sa
salita.
A. Asimilasyon
B. Paglapi
C. Pagkaltas
D. Metatesis
38. Anong pahayag ang pinakamataas sa pagpapahalaga ng Pilipina bilang Hiyas ng Lipunan?
A. Marikit at mahalagang kalahi
B. Kayumanggi ang balat
C. Mahinhin sa kilos at gawa
D. Mabuting kasambahay
39. Pangatnig na ginagamit sa pagpili, pamukod at pagtangi?
A. Panlinaw
B. Paninsay
C. Pamilang
D. Pamuklod
40. On what county gives the highest salary for teachers?
A. Thailand
B. Singapore
C. Philippines
D. China
41. Which philosophy comprises the official statement of China values during Han dynasty?
A. Confucian
B. Laotician
C. Hanonine Philosophy
D. Laotist
42. What is the term for the unpleasant sensation or frustration when traditional-bound Filipino
goes to the U.S and observe the open display of love and emotions?
A. Cultural relativism
B. Culture cosmopolitanism
C. Culture shock
D. Culture phobia

92

Downloaded by Noime Burtanog (noimeburtanog@gmail.com)


lOMoARcPSD|38673565

43. What contained the concrete promise of the American to Filipinos independence "as soon as
stable government can be established?
A. Preamble of the Jones Law
B. Preamble of the Commonwealth Government
C. Fairfield Bill of 1924
D. First 1919 Parliamentary Mission to the U.S
44. I could have joined the outreach project if I ___________ in the country at that time.
A. Will be
B. Were
C. Am
D. Was
45. Neither the teacher nor the students__________ present in the affair.
A. Is
B. Were
C. Was
D. Am
46. Boiled camote with butter ___________ my favorite snacks.
A. Are
B. Will be
C. Shall be
D. Is
47. Mr. Cortez was concerned about the safety of the students and so he reported the need of
safety measure to ____________ principal.
A. His
B. Her
C. Our
D. Their
48. Corrupt government officials should be ashamed of ___________.
A. Herself
B. Himself
C. Themselves
D. Ourselves
49. The authorities __________ warned the public about the wide spread of flu virus so that they
could take responsibilities.
A. Should have
B. Will have
C. Should be
D. Should
50. If I ___________ known you before, I would have arranged the project with you.
A. Will
B. Could
C. Have
D. Had

SOCIAL SCIENCE MAJOR


1. The Philippines ten-year transaction period before the granting of complete independence
from the Americans.
A. Commonwealth Government - is also known as the Quezon Government.
B. Third Republic
C. Puppet government
D. Revolutionary
2. The Act or Law that led to the granting of ten-year transition period.
A. Here-Hawes Cutting Bill
B. Tydings-McDuffie Law
 this Independence mission was headed by Press. Manuel L. Quezon
C. Parity Rights
D. Independence Law
3. The Chairman of the 1934 Constitutional Convention.
A. Cecillia Munoz Palma
B. Carlos P. Garcia
C. Diosdado Macapagal

93

Downloaded by Noime Burtanog (noimeburtanog@gmail.com)


lOMoARcPSD|38673565

D. Claro M. Recto
 the delegates to the convention were subsequently elected in 1934. In the first
meeting held on July 30 at the session of the House of Representatives, Claro M.
Recto was unanimously elected as its President.
4. The first appointed head of the Department of Education during Commonwealth period.
A. Claro M. Recto
B. Sergio Osmena - appointed by President Manuel L. Quezon.
C. Rafael Palma
D. Jose P. Laurel
5. The “father of social justice.”
A. Claro M. Recto
B. Sergio Osmena
C. Manuel L. Quezon - is also known as the " Father of Social Justice"
D. Jose P. Laurel
6. The leader of Sakdalista uprising
A. Benigno Aquino
B. Benigno Ramos
 the sakdalista coming from the tagalog word SAKDAL, meaning “to accuse”
movement was founded in 1930 by a right-wing leader, Benigno Ramos, a writer
and discontented former government clerk. The name of the movement was based
on Emile Zola’s 1898 letter criticizing the French government, J’ accuse.
C. Isabelo delos Reyes
D. Isabelle Espiritu
7. The last department turned over by the Americans to be government by a Filipino
A. Department of Public Health and Welfare
B. Department of Trade
C. Department of Education
 to ensure quality education in the Philippines and to preserve the interest of US
D. Department of Defense
8. The Rizal Day Celebration reminds us about? Heroes worth
A. Appreciating
B. Reading about
C. Emulating
 it encompasses the 3 domains of learning: Cognitive, affective and psychomotor
D. Studying
9. In Section 6, Article XIV states that the national language of the Philippines is
________________.
A. English
B. Filipino
 The national language of the Philippines is Filipino. As it evolves, it shall be
further developed and enriched on the basis of existing Philippine and other
languages.
C. Tagalog
D. Cebuano
10. In Section 3, Article XIV states that “All educational institutions shall include that study of
_____ as part of curriculum.”
A. History
B. Science & Technology
C. Sports
D. Constitution
 is an aggregate of fundamental principles or established precedents that constitute
the legal basis of a polity, organization or other type of entity and commonly
determine how that entity is to be governed.
11. Who has the power of declare the existence of the state of war?
A. Senate President
B. Chief Justice
C. President
D. Congress
 by a vote of two-thirds of both Houses in joint session assembled, voting
separately, shall have the sole power to declare the existence of a state of war.

94

Downloaded by Noime Burtanog (noimeburtanog@gmail.com)


lOMoARcPSD|38673565

12. “The State shall protect and promote the right of all citizens to quality education at all
levels.” Which government program is in support of this?
A. Exclusion of children with special needs from the formal system
B. Free elementary and secondary education
 Section 2, Article XIV of the 1987 constitution
C. Deregulated and secondary education
D. Re-introduction of the NEAT and NSAT
13. The old name of EDSA during early part of American occupation
A. 19 de Junio
 Epifanio de los Santos Avenue (EDSA, pronounced ed-sah), formerly known as
HIGHWAY 54, is the main circumferential road and highway of Metro Manila in
the Philippines. It is an important commuting artery between the northern and
southern parts of the metropolitan area. EDSA is a partially-controlled access,
mostly 10-lane divided highway (expressway), with three general lanes and the
two outer lanes reserved for buses and other HOV traffic lights.
B. Highway 54
C. Highway 53
D. Highway 52
14. RA 9155 also known as “_____________”.
A. Basic Education Act of 2001
 the Governance of Basic Education Act of 2001, provides the overall framework
for principal empowerment by strengthening principal and leadership goals, and
local school-based management within the context of transparency and local
accountability.
B. Restructured Basic Education
C. Education For All
D. Bridge Program
(RATIONALIZATION: an act instituting a framework of government for basic education,
establishing authority and accountability, renaming the department of education, culture and
sports as the department of education, and for other purposes.)
15. RA 4670 also known as “_____________”.
A. Board of Higher Education
B. The Magna Carta for Public School Teachers
 Sec. 1. Declaration of Policy. It is hereby declared to be the policy of this Act to
promote and improve the social and economic status of public-school teachers,
their living and working conditions, their terms of employment and career
prospects in order that they may compare favorably with existing opportunities in
other walk of life, attract and retain in the teaching profession more people with
the proper qualification, it being recognized that advance in education depends on
the qualifications, the teaching staff and that education is an essential factor in the
economic growth of the nation as a productive investment of vital importance.
C. State of National Emergency
D. Calibrated Pre-emptive Response
16. President Gloria Macapagal Arroyo contribution during his terms?
A. Ten-point Strong Republic agenda
 aggressively on an ambitious agenda of building a "strong republic".
B. SONA 2001
C. Philippines 2000
D. Sustainable Development
17. President Estrada’s MTPDP (Medium-Term Philippine Development Plan)
A. Angat Pinoy 2004
 the Philippines 2000 program would be succeeded by two other socio-economic
programs: Angat Pinoy 2004, instituted by Joseph Estrada, the ten-point Strong
Republic agenda instituted by Gloria Macapagal Arroyo, Tuwid na Daan agenda
instituted by Benigno Aquino III, and the Build Build Build agenda instituted by
Rodrigo Duterte.
B. SONA 2001
C. Philippines 2000
D. Sustainable Development
18. How long can an elected President of the Philippines serve under the 1987 Constitution?
A. Two terms, 6 years each
B. Two terms, 4 years each

95

Downloaded by Noime Burtanog (noimeburtanog@gmail.com)


lOMoARcPSD|38673565

C. One term for 6 years


 under Section 4, Article VII of the 1987 Constitution
D. One term for 4 years
19. Historically, why does the Philippines have a claim over Sabah?
A. The sultan of Sulu, who received the territory as a gift, has given the Philippines
Government Power to reclaim his territory
 the official filing of the claim took place on June 22, 1962. The claims are of
sovereignty, jurisdiction, and proprietary ownership to North Borneo. Philippines
being successor-in interest of the Sultan of Sulu derived its legal and historical
rights in North Borneo.
B. The Malays, the third wave of migrants to the country, own the territory.
C. Sabah was formerly a part of the Philippines archipelago.
D. The Philippines government bought it from the Sultan of Brunei
20. Otherwise known as the Code of Conduct and Ethical Standards for Government employees
and officials.
A. RA 6713
 this Act shall be known as the "Code of Conduct and Ethical Standards for Public
Officials and Employees." Section 2. Declaration of Policies. - It is the policy of
the State to promote a high standard of ethics in public service.
B. RA 1425
C. RA 7610
D. RA 9262
21. Which of the following describes the Battle of Manila Bay between American and Spanish
naval forces?
A. The Americans ships outnumbered those of the Spaniards
 the Battle of Manila Bay took place on 1 May 1898, during the Spanish-American
War. The American Asiatic Squadron under Commodore George Dewey engaged
and destroyed the Spanish Pacific Squadron under Admiral Patricio Montojo y
Pasaron. The engagement took place in Manila Bay in the Philippines, and was
the first major engagement of the Spanish-American War.
B. The Spaniards readily won the battle over the Americans.
C. It was a mock naval battle between the Spaniards and the Americans.
D. The Americans won the battle with great difficulty.
22. What right and duty do Filipinos exercise when they participate in ratifying a proposed
constitution?
A. Citizenship
B. Habeas corpus
C. Suffrage - through plebiscite
D. Bill of Rights
23. The old name of EDSA during the World War II
A. 19 de Junio
B. Highway 54
 was renamed “Epifanio de los Santos Avenue (Edsa),” in recognition of Don
Panyong's genius and contributions to the country's intellectual and artistic
heritage.
C. Highway 53
D. Highway 52
24. Who was the first president of the Republic of the Philippines when independence from
America was granted on July 4, 1946?
A. Pres. Corazon Aquino
B. Pres. Elpidio Quirino
C. Pres. Ferdinand Marcos
D. Pres. Manuel Roxas
 was the last president of Commonwealth and the first president of the third
Republic.
25. According to him President “… this Nation will be great again…!
A. Pres. Corazon Aquino
B. Pres. Elpidio Quirino
C. Pres. Ferdinand Marcos - he delivered this state in one of his inaugural speeches.
D. Pres. Manuel Roxas

96

Downloaded by Noime Burtanog (noimeburtanog@gmail.com)


lOMoARcPSD|38673565

26. He had promised sweeping economic changes, and he did make progress in land reform,
opening new settlements outside crowded Luzon Island. His death in an airplane crash in
March 1957, was a serious blow to national morale.
A. Pres. Ramon Magsaysay - the President of the “Common Tao”
B. Pres. Elpidio Quirino
C. Pres. Ferdinand Marcos
D. Pres. Manuel Roxas
27. Martial law remained in force until 1981 under Proclamation # _________?
A. 1081 - from September 21, 1972 to 1981
B. 1425
C. 9994
D. 7610
28. What in SONA stands for?
A. State of the National Address
B. State of the Nation Approval
C. Status of the Nation Address
D. State of the Nation Address
 is an annual address by the president of the Philippines to a joint session of the
Congress of the Philippines, annually; 4th Monday of July.
29. What is Republic Act 8049?
A. Anti – Hazing Law
 is an act regulating hazing and other forms of initiation rites in fraternities,
sororities, and other organizations and providing penalties therefore.
B. Anti-sexual harassment Law
C. Law Protection Women and Children against all forms of abuses, discrimination and
exploitation
D. Expanded Senior Citizen Act of 2010
30. What is Republic Act 9994?
A. Anti – Hazing Law
B. Anti-sexual harassment Law
C. Law Protecting Women and Children against all forms of abuses, discrimination and
exploitation
D. Expanded Senior Citizens Act of 2010
 is an act granting additional benefits and privileges to senior citizens, further
amending Republic Act No. 7432, as amended, otherwise known as “An Act to
maximize the contribution of Senior Citizens to nation building, grant benefits
and special privileges and for other purposes.
31. Republic Act 9262 adheres with
A. Anti – Hazing Law
B. Anti-sexual harassment Law
C. Law Protecting Women and Children against all forms of abuses, and exploitation
 the Violence against women and their children” refers to any act or a series of acts
committed by any person against a woman who is his wife, former wife, or
against a woman with whom the person has or had a sexual or dating relationship,
or with whom he has a common child, or against her child whether legitimate
D. Expanded Senior Citizens Act of 2010
32. Republic Act 7877 also known as
A. Anti – Hazing Law
B. Anti-sexual harassment Act of 1995
 is an act declaring sexual harassment unlawful in the employment, education for
training environment, and for other purposes.
C. Law Protecting Women and Children against all forms of abuses, discrimination and
exploitation
D. Expanded Senior Citizens Act of 2010
33. Section 5 (5), Article XIV of the 1987 Constitution mandates the Government to give the
highest budgetary to:
A. Department of Interior and Local Government
B. Department of Education
 Section 5 (5), Article XIV of the 1987 Constitution

97

Downloaded by Noime Burtanog (noimeburtanog@gmail.com)


lOMoARcPSD|38673565

C. Department of Science and Technology


D. Department of Public Works and Highways
34. The chairmen of the Constitutional Convention?
A. The delegates elected George W. Bush
B. The delegates elected George Washington
 to preside over the Convention. 70 Delegates had been appointed by the original
states to attend the Constitutional Convention, but only 55 were able to be there.
C. The delegates elected Ronald Reagan
D. Former President of the United States in 1971
35. With peace education in mind, what is not included
A. Dialogue
B. Mediation
C. Cooperation
D. Conflict - is the anti-thesis of peace education.
36. What is the very foundation of genuine peace and reconciliation?
A. Happiness
B. Joy
C. Confrontation
D. Social Justice - in the Article XII of the 1987 Constitution
37. The newly appointed secretary of DSWD by President Rodrigo Roa Duterte?
A. Jinky Soliman
B. Rolando Joselito Bautista
 is a retired Filipino lieutenant general and the incumbent Secretary of Social
Welfare and Development.
C. Pacquito Ochoa, Jr.
D. Butch Abad
38. The first essay of Rizal
A. El Amor Partio
 days after he arrived in Barcelona in June 1882, he wrote the essay El Amor
Patrio (Love of the Country), which contained the reasons behind that deep
fondness.
B. A la Juventud Filipina
C. Sa Aking Kabata
D. Mi Ultimo Adois
39. The Republic Act that requires us to teach Rizal and other Filipinos patriots to tertiary
students
A. RA 7836
B. RA 7610
C. RA 1425
 is an act to include in the curricula of all public and private schools, colleges and
universities courses on the life works and writings of Jose Rizal, particularly his
novels Noli Me Tangere and El Filibusterismo, authorizing the printing and
distribution thereof, and for other purposes.
D. RA 9165
40. What is the Right of suffrage advocates?
A. The Right of the people to vote
B. The Right of the people to vote and be elected in public office
 in the Article V of the 1987 Constitution
C. The Right of the people not to vote
D. The Right of the people for clean elections
41. Where is the International Rice Research Institute (IRRI) located?
A. UP Los Banos, Laguna
 IRRI is an independent, nonprofit, research and educational institute, founded in
1960 by the Ford and Rockefeller foundations with support from the Philippine
government. The institute, headquartered in Los Baños, Philippines, has offices in
17 rice-growing countries in Asia and Africa, and more than 1,000 staff.
B. UP Diliman, Quezon City
C. UP Iloilo
D. UP Manila
42. What is the mountain that separates the continents of Asia and Europe?

98

Downloaded by Noime Burtanog (noimeburtanog@gmail.com)


lOMoARcPSD|38673565

A. Ural mountain
 they are rich in various deposits, including metal ores, coal, precious and semi-
precious stones, and since the 18th century have been the major mineral base of
Russia. In the east, the Ural Mountains separate Europe from Asia. The nations of
Russia and Kazakhstan straddle both continents. Another range, the Kjølen
Mountains, extends along the northern part of the border between Sweden and
Norway.
B. Mt. Kilimanjaru
C. Mt. Everest
D. Mt. Apo
43. The old name of Vietnam
A. Formosa
B. Siam
C. Persia
D. Annam
 Is now Vietnam. The term Vietnam dates from the early 19th century, when the
Nguyen dynasty was founded. The Vietnamese government of this time was on
very close terms with China, and the name was probably a compound derived
from Dai Viet and Annam. Like the Chinese before them, modern colonial powers
also avoided the term Viet.
44. Who are identified as “yellow Race” at the end of the eighteenth century when they were
lumped together into a new racial category?
A. Vietnamese
B. Filipino
C. Mongolian
 a defining moment occurred at the end of the eighteenth century when they were
lumped together into a new racial category called the “Mongolian,” commonly
identified as the “yellow race.”
D. Malaysian
45. The major rivers in China
A. Tigris and Euphrates
B. Hwang-Ho and Yangtze
 the Hwang-Ho river is the second-longest river in China (after the Yangtze River)
and the sixth-longest in the world at the estimated length of 5,464, kilometers
(3,395 mi).
C. Nile and Indus
D. Aegean and Mekong
46. The World biggest peninsula
A. Arabian peninsula
 it is the world’s largest peninsula and covers 3,237,500 Km2 (1,250,000 mi2). The
area is an important part of the Middle East and plays a critical geopolitical role
due to its vast reserves of oil and natural gas.
B. Iberian peninsula
C. Korean peninsula
D. Italy
47. The boot-shaped peninsula
A. Arabian peninsula
B. Iberian peninsula
C. Korean peninsula
D. Italy
 the country’s total area is 301,230 km2, of which 294,020 km2 is land and 7,210
km2 is water including the islands.
48. Known as the “Isla de Pintados” by the Spaniards
A. Luzon
B. Visayas
 the Visayans are most painted people especially Iloilo provinces.
C. Mindanao
D. Isla Verde
49. Man’s conviction is his or her _________

99

Downloaded by Noime Burtanog (noimeburtanog@gmail.com)


lOMoARcPSD|38673565

A. Norms
B. Belief
 We tend to internalize the beliefs of the people around us during childhood.
C. Values
D. Mores
50. Also known as the Bangkok Declaration of 1967
A. APEC
B. UN
C. EU
D. ASEAN - this was held in Thailand in 1967

PROFESSIONAL EDUCATION REVIEWER


1. Which Republic Act provides government assistance to students and teachers in private
education?
a. RA 7836
b. RA 6728
c. RA 7784
d. RA 6675
2. In the preamble of the CODE of ethics for Professional Teachers, which of the following
teacher descriptions is included in the Code of Ethics for Professional Teachers?
a. With satisfactory teaching performance
b. Passed the licensure exams for teachers
c. Duly licensed professional
d. Persons of dignity and reputation
3. Which stage according to Erickson is the stage of trust and mistrust?
a. Young adulthood
b. Adolescence
c. Early childhood
d. Infancy
4. In the history of education system, including that of the Philippines, which system was first
and last remained a partner of other systems of education?
a. Formal
b. Non-formal
c. Informal
d. Pre-school
5. Who among the following subscribe to the theory that the good, the true and the beautiful are
universally valid in all places at all times?
a. Realists and idealists
b. Realist and existentialists
c. Pragmatists and idealists
d. Pragmatists and realists
6. The 1987 Constitution mandates the state to encourage among others self-learning,
independent and out-of-school study programs particularly those that respond to community
needs. Which is an off shoot of this mandate?
a. Institutionalization of early childhood care and development
b. Principal empowerment program
c. Multi-grade teaching
d. Practices of community-based learning
7. Authors claim that education is given greater in the 1987 Philippine Constitution than in the
past Constitution. Which article provision on education is found only in the 1987
Constitution?
a. Support to teacher education
b. Provision on assigning the highest budget priority to education
c. The establishment of the integrated system of education
d. The creation of CHED
8. According to Hurlock, studies of children’s play revealed that toy play reaches its peak
during the
a. Middle childhood years
b. Early childhood years
c. Babyhood

100

Downloaded by Noime Burtanog (noimeburtanog@gmail.com)


lOMoARcPSD|38673565

d. Late childhood
9. In a DECS memorandum issued last 1998, the new name for multi-grade class is:
a. Friendly school
b. School-of-the-future
c. Paaralan ng bahay
d. Escuela nueva
10. The student-centered curriculum belongs to which group of educators?
a. Reconstructionist-Essentialist
b. Progressivist, Perennialist
c. Perennialist, Essentialist
d. Progressivist, Reconstructionist
11. Daniel Goleman talks about emotional intelligence. Which of the following characterize a
student with a high degree of emotional intelligence?
a. Sensitive to points of view and feeling of others
b. Prodded by others
c. Obsessed with achieving at any cost
d. Unable to delay gratification
12. The history of curriculum development includes the hidden curriculum approach. Which is
the content emphasis of the hidden curriculum?
a. Students’ experiences and activities
b. Implicit processes and social norms
c. Student needs and interest
d. Introspection and choice
13. Teacher T is concerned with conceptual matters since reality is mental teacher T’s thinking is
quite-
a. Realistic
b. Pragmatic
c. Existential
d. Idealistic
14. The authoritarian setting in the Filipino home is reinforced by a classroom teacher who-
a. Asks open – ended questions
b. Encourages pupils to ask questions
c. Prescribes what pupils should do
d. Is open to suggestions
15. A number of researchers found that the effects of maternal employment on children’s
achievement are-
a. Hardly established
b. Positive and negative
c. Fully established
d. Negative
16. Which one correctly describes the phenomenon of latchkey children?
a. They turn to empty homes after school either to sit by the television or to roam the
streets
b. They are truant school children
c. They are engaged in child labor
d. They are engaged in child labor
17. Teacher Q regards the student as a spiritual entity and as part of the larger spiritual universe.
To what philosophy does teacher Q subscribe?
a. Idealism
b. Existentialism
c. Realism
d. Pragmatism
18. The 1987 Constitution mandates the state to assign the highest budgetary priority to-
a. Health
b. Education
c. Social work
d. Defense
19. Moral development in early childhood is characterized by-
a. Questioning rules
b. Willful disobedience of rules
c. Acting out of conviction
d. Acting without knowing why they do so

101

Downloaded by Noime Burtanog (noimeburtanog@gmail.com)


lOMoARcPSD|38673565

20. According to Piaget, by adolescence boys and girls must have reached what stage?
a. Concrete operations stage
b. Sensorimotor stage
c. Formal operations stage
d. Pre-operational stage
21. Ruth dances well. She can figure out how something works or how to fix something that is
broken, without asking for help. Based on Garden’s theory of multiple intelligence under
what intelligence is she strong?
a. Musical
b. Logical-mathematical
c. Spatial intelligence
d. Kinesthetic intelligence
22. Who among the following believes that learning requires disciplined attention regular
homework, and respect for legitimate authority?
a. Reconstructionist
b. Perennialist
c. Essetialist
d. Progressivist
23. Which developmental task is expected of the adolescent according to havughurst?
a. Achieving new and more mature relations with age-mates of both sexes
b. Skilled games
c. Getting started in an occupation
d. Learning to get along with age-mates
24. Mr. Roy wants his pupils to be creative. Which of the following will he REFRAIN from
doing?
a. Heighten the student’s sense of unusual
b. Develops students’ ability to recognize and analyze problems and relationships
c. Encourage sustained attention in an area of interest
d. Develop solely skill mastery
25. The creation of Student’s Loan Fund to give equal opportunity to all persons who desire to
pursue higher education is the essence of:
a. RA 6014
b. RA 7836
c. RA 6655
d. RA 7784
26. In his/her teaching, Teacher D moves from particular instances to tentative generalizations
that are Subjected to further verification. Teacher D engages himself/herself in:
a. Intuition
b. Inductive logic
c. Deductive logic
d. Philosophical analysis
27. Which of the correct statement of emotional intelligence is based on Daniel Goleman’s
theory?
a. Emotional intelligence is feeling approximately and effectively
b. Emotional intelligence id being nice to people
c. Emotional intelligence is giving free rein to feeling
d. Emotional intelligence changes less considerably than IQ through life.
28. Which of the following is not a subject-centered curriculum?
a. Perennialist
b. Back-to-basics
c. Values-centered
d. Subject area
29. Which program is assisted financially by the World Bank and OECF and is meant for the
improvement of elementary education in the SRA provinces?
a. Program for Decentralized Education
b. First Elementary Education Project
c. Kinesthetic intelligence
d. Second Elementary Education Project
30. Mr. Z, A fifty-five year old American citizen, even though very educationally qualified to be
president an educational institution in third country cannot be appointed as President
because: He_____
a. Does not belong to a religious group

102

Downloaded by Noime Burtanog (noimeburtanog@gmail.com)


lOMoARcPSD|38673565

b. Is nearing retirement
c. Is not Filipino Citizen
d. Is not an official of a corporation
31. Which of the following is not a hazard to the mastery of developmental tasks?
a. Unfavorable social judgment
b. Bypassing of stage of development as a result of failure to master the tasks for that
stage of development
c. Crisis when individuals pass from on stage to another
d. Inappropriate or impossible expectations
32. Teacher U emphasizes to her students the importance of deep personal reflection on one’s
commitments and choices. Teacher U subscribes to which Philosophy?
a. Realism
b. Idealism
c. Existentialism
d. Pragmatism
33. The constitutional provision on language has the following aims, Except:
a. To make Filipino the national language and medium of instruction and official
communication
b. To make the regional dialect as auxiliary media of instructions in regional schools
c. To make Filipino the sole medium of instruction
d. To maintain English as a second language
34. Which periods in the life span of an individual are characterized by growth spurt?
a. Early childhood and adolescence
b. Prenatal period and early childhood
c. Early childhood
d. Pre-natal and puberty
35. The introduction of non-formal education in line with the Constitutional Provision on:
a. Protection of teachers
b. Promoting the rights of all citizens to make quality education accessible to all
c. Optional religious instructions
d. Promoting the rights of all citizens to quality education at all levels
36. Teacher X is often times frustrated. The students in her class hardly volunteer to recite and to
do other learning-related task. This is a proof of the Filipino’s:
a. Sense of humor
b. Lack of resourcefulness
c. Lack of reflection
d. Passivity and lack of initiative
37. The tendency to emphasize so much on school beautification to the detriment of pupil’s
performance illustrated the:
a. Filipino’s sense of humor
b. Filipino’s lack of reflection
c. Filipino’s love for “Porma” at the expense of substance
d. Filipino’s lack of seriousness
38. A president body to study Philippines education created by virtue of Executive Order No. 46
during the incumbency of DECS secretary Andrew Gonzales with an aim to study Philippine
Education is the:
a. Presidential Commission on Educational Reform
b. Philippine Commission to Survey Philippine Education
c. Survey to outcomes of elementary Education
d. Commission on Philippine Education
39. Which is a danger signal of adolescent mal-adjustment?
a. Inappropriate body-build
b. Irresponsibility as shown in neglect of studies in favor of having a good time
c. Attraction to the opposite sex
d. Prolonged treatment as children
40. To democratize access to secondary education, public secondary education was made free. In
whose presidency was this implemented?
a. Ferdinand Marcos
b. Joseph Estrada
c. Corazon Aquino
d. Fidel Ramos
41. Which is not a characteristic of democratic discipline?

103

Downloaded by Noime Burtanog (noimeburtanog@gmail.com)


lOMoARcPSD|38673565

a. Child obeys blindly


b. Child understands the meaning of rules
c. Child is given punishment us related to the misdeed
d. Child has opportunity to express his/her opinion
42. The three A’s of happiness according to Hurlock are:
a. Adjustments, affection, altruism
b. Attitude, ability, adjustment
c. Acceptance, affection, and achievement
d. Affection, ability, attitude
43. Babyhood is often referred to as a “Critical period” in the development of personality
because:
a. At this time the foundations are laid upon which the adult personality structure will be
built
b. Changes in the personality pattern take place
c. The brain grows and develops at such an accelerated rate during babyhood
d. At this time the baby is exposed to many hazards both physical and psychological
44. Who among the following stressed the processes of experiences and problem solving?
a. Plato
b. Hegel
c. Dewey
d. Aristotle
45. Teacher L believes that creation of knowledge in by way of the learners’ interaction with their
environment. Teacher L is more of:
a. An idealist
b. A pragmatist
c. An existentialist
d. A realist
46. To reach out clientele who cannot be in the classroom for one reason or another, which of the
following was established
a. Non-formal education
b. Informal education
c. Pre-school education
d. formal education
47. The encouragement of self-learning, independent, and out-of-school study programs as stated
in the 1987 constitution has given rise to:
a. The implementation of open universities and distance learning programs
b. Institutionalization of early childhood care and development
c. The conduct of NEAT and NSAT
d. The clamor for a Grade VII
48. Which of the following statement on developmental tasks is wrong?
a. Failure to master developmental tasks at a certain developmental stage has far
reaching consequences in a person’s development
b. The mastery of development tasks is a result of physical maturation, societal pressure
and individual’s aspiration
c. There are essential skills expected to be acquired and mastered in each developmental
stage
d. Retirees are not expected to work on mastering certain develop mental tasks
49. The 1987 Philippines Constitution states the following, Except:
a. The official languages of the Philippines are Filipino and English
b. The national language of the Philippines is Filipino
c. Filipino is the Tagalog of the Tagalog –speaking provinces
d. The government shall initiate and sustain the use of Filipino as a medium of official
communication and as a language of instruction in the education system
50. Which among the following has been said to play an irreplaceable role in the education of the
young?
a. Community
b. School
c. Church
d. Home
51. For the drill method to be effective there should be:
a. Much practice on a few skills
b. Little practices on few skills

104

Downloaded by Noime Burtanog (noimeburtanog@gmail.com)


lOMoARcPSD|38673565

c. Much practice on many skills


d. Little practice on many skills
52. Which of the following classroom management practices runs counter to the reality therapy
approach of William Glasser?
a. The teacher asks a misbehaving student what he or she is doing
b. Teacher requires the students to prepare a plan in writing and sign it as means of
increasing personal motivation to maintain and fulfill the plan
c. The teacher evaluates pupil’s behavior as good or bad
d. Teacher assists the students in making realistic plans to change behavior
53. Which of the following does not apply to mastery learning?
a. It makes use of varied instructional time for different groups of students
b. It makes use of norm-reference tests
c. It is effective in teaching basic skills
d. It requires the use of carefully crafted instructional objective
54. The teacher’s role in problem solving methods is:
a. To test the conclusions
b. To set up the problem
c. To propose ways of obtaining the needed data
d. To help the learners what is it that is being solved
55. To nurture creativity in student, teacher should:
a. Vary the length and difficulty of question
b. Allow for one-minute wait-time
c. Ask convergent questions
d. Emphasize the necessity of giving right answers
56. The content of a good lesson plan is self-sufficient. This means that:
a. The content should match with student’s aptitude
b. The content should help students learn how to learn
c. The content should be broad and treated thoroughly
d. The content should be verifiable
57. Which one can NOT help individuals reduce their own fears about change?
a. Reflect on the extent of change in one’s life so far and how it has coped with
b. Identify shortfalls in skills or knowledge and take action to remedy them
c. Identify skills which will be useful in a new situation
d. Keep feeling of anxiety to themselves
58. The following are generating thinking skills, Except:
a. Connecting new ideas
b. Predicting
c. Classifying
d. Inferring
59. Mrs. Valdez wants to generates as many ideas as she can as the class is about to embark on a
community outreach program. Which of the following will she employ?
a. role playing
b. brainstorming
c. brainwashing
d. simulation
60. Which teaching practice will most likely decrease learner’s attention?
a. Questioning
b. Assessing learning as an integral part of instruction
c. Assessing learning as an integral part of instruction
d. Teaching by telling
61. For efficiency, if in the process of teaching a teacher realizes that too little has been planned,
which of the following may she NOT do?
a. Discuss possible problems of the new assignment.
b. Pose additional questions to explore various facets of the content
c. Drill the students on the major points of the lesson.
d. Give a quiz
62. Correct practice makes perfect. This maxim is based on Thorndike’s law of exercise and the
finding that reinforcement of a response increases the likelihood its occurrence. Whose
research finding is the underlined statement?
a. Skinner
b. Bruner
c. Palo

105

Downloaded by Noime Burtanog (noimeburtanog@gmail.com)


lOMoARcPSD|38673565

d. Lewin
63. Here is a lesson objective: Given a microscope and a slide, the students must be able to focus
the prepared slide. Applying Robert F. Mager’s principle, which does this lesson objective
lack?
a. Criterion measure
b. Condition
c. Performance
d. time element
64. Which is a focusing thinking skill?
a. Identifying key concepts
b. Summarizing
c. Obtaining information
d. Clarifying through inquiry
65. In the use of television in the classroom, which of the following should be avoided?
a. The TV program lasts the whole class period.
b. There is a pre-viewing orientation
c. Select programs that match the learners’ level of interest and maturity
d. Life should be left on if students are to take notes
66. Based on Victor Lowenfield’s classification, which sequence in the developmental stages of
children in art is followed?
a. Scribbling stage, pre-schematic stage, schematic stage, dawning realism stage,
pseudo-realistic stage
b. Scribbling stage, pre-schematic stage, schematic stage, pseudorealistic stage, dawning
realism stage
c. Pre-schematic stage, schematic stage, scribbling stage, pseudorealistic stage, dawning
realism stage
d. Pre-schematic, schematic stage, scribbling dawning realism stage, pseudo-realistic
stage
67. In relation to teacher’s ratings research suggests the following, Except one:
a. Teachers favor self-evaluation over all other forms of evaluation
b. Peer and supervisory evaluation are not reliable
c. Student raters of teachers are more reliable and valid than other raters
d. Student work and test outcomes are supplementary sources for evaluating teachers
68. Suppose defined 3 levels of computer-assisted instruction: dialogue, tutoring, practice-drill.
Which ones are two higher levels in the correct order?
a. Tutoring and dialogue
b. Practices-drill and tutoring
c. Dialogue and tutoring
d. Practice-drill and dialogue
69. In what condition is the use of the lecture method appropriate?
a. Higher cognitive learning is sought
b. The information is not available
c. The subject matter is quite easy
d. Long term learning is desired
70. Which refers to the time when students learn at a maximum level?
a. Wait time
b. Engaged time
c. Allocated time
d. Academic learning time
71. What is a sign of the underachiever in the classroom?
a. Resist authority and carry on a power struggle with a teacher.
b. Holds back from class participant unless sure of self
c. Frustrated about quality of work
d. Minimum work output.
72. If after calling on a number of students, a teacher is unable to obtain the desired response,
what should teacher do
a. Ask leading questions
b. Re-teach parts of the lesson that need re-teaching
c. Probe student’s answers.
d. Rephrase the questions
73. For global competitiveness, a school must embark on a proactive change. Which one is a
characteristic of proactive change?

106

Downloaded by Noime Burtanog (noimeburtanog@gmail.com)


lOMoARcPSD|38673565

a. radical and inventive


b. imitative of others
c. late in the game
d. problem-driven
74. Which guideline on the use of the chalkboard should a teacher AVOID?
a. Establish routine uses for the chalkboard
b. While writing, proceed from right to left
c. Don’t talk to the chalkboard while writing on it
d. Limit your board writing to major ideas
75. To teach the democratic process to the pupils, Mabuhay Elementary School decided that the
election of class and school officers shall be patterned after local election of the class and
school officers shall be patterned after local election process. There are qualifications set for
candidates, limited period for campaign and rules for posting campaign materials, etc. Which
of the following did the school use?
a. Simulation
b. Philips 66
c. Role playing
d. Symposium
76. Which is NOT a characteristic of preventive discipline.
a. Proactive
b. Anticipatory
c. Inventive
d. Reactive.
77. One criterion that has been proposed in the selection and organization of content in the
development section of a lesson plan is balance. When does lesson content possess balance.
a. The content should have practical application for the learners
b. The content is not cluttered by masses of more trivial content
c. The content should enable the learners to experience the broad sweep of content and
give them the opportunity to go deeper
d. The content should help student learns how to learn
78. The burnout malady gets worse if a teacher doesn’t intervene to change whatever areas he or
she can control. Which one can renew a teacher’s enthusiasm.
a. Engage in self-pity
b. Stick to the job
c. Judge someone else as wrong
d. Initiate changes in job.
79. Although learning can take place anywhere and anytime, the more systematic the teacher, the
greater the probability for success. This points out the need for teachers to:
a. Specify their objectives
b. Plan their lessons
c. Set their teaching priorities
d. Select their instructional materials
80. Below are teacher’s effective classroom management practices, Except One.
a. Teachers make sure students understand and follow rules and procedures
b. Disruptive behavior is handled every weekend
c. Teachers clearly establish consequences for not following rules
d. Teachers spend more time in the beginning of the year explaining and reminding
students of rules.
81. A teacher’s summary of a lesson serves the following functions except;
a. It links the parts of the less
b. It makes provisions for full participation of the students
c. It clinches the basic ideas or concepts of the lesson
d. It brings together the information that has been discussed
82. Which ones are projected visuals?
a. Graphs on a book
b. Models
c. Realias
d. Slides
83. The lesson is on the pros and cons of capital punishment. Mr. Milan wants to do high level
thinking and to develop a view of capital punishment from a different perspective. Which
technique will be most appropriate?
a. Lecture

107

Downloaded by Noime Burtanog (noimeburtanog@gmail.com)


lOMoARcPSD|38673565

b. Simulation
c. Role playing
d. Panel discussion
84. Which is used to emphasize individualized instruction?
a. Tutorial group
b. Philips 66
c. Inquiry group
d. Task group
85. Here a question: FROM THE DATA PRESENTED FORM GENERALIZATIONS THAT
ARE SUPPORTED BY THE DATA. Under what type of question does this test item fall?
a. Evaluate
b. Convergent
c. Divergent
d. Application
86. Ms. Estira cannot bring her pupils to the sea for a lesson on marine community. Which of the
following will be closest to an actual experience of marine community?
a. Motion picture on marine community
b. Description of marine community
c. Pictures of marine community
d. Taped lesson of marine community
87. Which can NOT help organizations reduces fears about change?
a. By training people in new skills
b. By keeping people informed about plans
c. By cladestine moves
d. By consultation
88. After a lesson in air pollution, teacher C gives each pupil mimeographed sheets, which
contain statements to which each is expected to react. Examples of these statements are. Do
you in any way contribute to air pollution? What solutions do you propose to minimize, if not
eliminate, air pollution? Then the students are grouped for sharing. In this case what does the
teacher make use of
a. Value sheet
b. Conflict story
c. Rank ordering
d. Contrived incident
89. Which orders consist of the goal-oriented instructional model?
a. Pre-assessment, specification of objectives, pre-assessment, instruction, evaluation
b. Specification of objectives, pre-assessment, instruction evaluation
c. Pre-assessment, specification of objectives, instruction, evaluation
d. Specification of objectives, evaluation, pre-assessment, instruction
90. Teacher B’s lesson was on abortion. She wanted to get her student’s maximum participation
and reaction. She showed slides on abortion accompanied by an audiotape that presented the
reasons for and against abortion. Teacher took a stand in favor of abortion and came up with
all her justifications. At the end, however, she made clear her stand on abortion. Which did
the teacher use?
a. She made use of value clarifying discussion
b. She made use of contrived incident
c. She made use of simulation
d. She played the role of the devil’s advocate
91. In one of the pages of her reference material, Ms. Estrada finds the life cycle of a frog.
Assuming that the following are available to her, the quickest way to effectively present the
life cycle of a frog to the class is by way of
a. An overhead projector
b. An opaque projector
c. Describing the life cycle
d. A drawing on the chalkboard
92. Below are questions that must be considered in developing appropriate learning activities/
experience, Except one
a. Do the experiences encourage pupils to inquire further?
b. Do the experiences save the pupils from learning difficulties?
c. Can be experiences profit the pupils?
d. Are the experiences in accordance with an increasing amount of learning?
93. Which order of the senses goes with an increasing amount of learning?

108

Downloaded by Noime Burtanog (noimeburtanog@gmail.com)


lOMoARcPSD|38673565

a. Sight, hearing, touch, taste and smell


b. Taste and smell, sight, touch, hearing
c. Taste and smell, touch, sight, hearing
d. Taste and smell, touch, hearing, sight
94. Which of the following statements on peer tutoring is true?
a. Teacher has time with more severe learning problems
b. The atmosphere is threatening
c. The social skills of pear tutors are not enhanced
d. the peer tutor’s achievement goes down
95. A lesson in Philippines history was presented by Mrs. Palaroan by making her class view a
videotape on EDSA revolution. The videotape which Mrs. Palaroan used in class points to
what property of instructional media?
a. Manipulative
b. Multi-sensory
c. Fixative
d. Distributive
96. Read the following teacher-student- situation.
“Teacher: Why is the process called photosynthesis?
Student: I don’t know. “
Which questioning technique should the teacher use?
a. Concept
b. Prompting
c. Clarification
d. Multiple response
97. Miss Lee’s objective is to focus student’s attention on and quickly create interest in a
problem or concept. She makes use of:
a. Task group
b. Panel discussion
c. Tutorial group
d. Philips 66
98. The content of the lesson must be feasible. This means that:
a. The lesson should be interesting to the student
b. The lesson must be within the capacity of the students
c. The teacher must consider the time needed and resources available
d. The lesson must be useful
99. Which questioning behavior is appropriate:
a. Not allowing a student to complete a long response
b. Repeating all student’s response
c. Allowing choral responses
d. Asking varied questions
100. Research on teacher effective practices has shown the following except:
a. Planning has title impact on student learning
b. Questioning strategies are ineffective monitoring technique
c. Teaching procedures on classroom routines early in the school year are essential
d. Directives should be few and best delivered in a casual manner
101. Teacher Y wants to measure student’s ability to organize thoughts and ideas. Which type
of test is most appropriate?
a. Objectives test
b. Limited response essay
c. Extended response essay
d. Short answer type of test
102. Which of the following refers to organized services specially designed to improve the
adjustment of students?
a. Group growth sessions
b. Test profiles
c. Cumulative records
d. Guidance program
103. Which tests survey existing problems of academic progress?
a. Achievement tests
b. Interest inventories
c. Problem checklist
d. Aptitude tests

109

Downloaded by Noime Burtanog (noimeburtanog@gmail.com)


lOMoARcPSD|38673565

104. Which of the following statements about guidance is NOT true?


a. Guidance is a continuous process
b. Guidance is concerned with “whole” student, not only with his or her intellectual and
academic aspect
c. Guidance is telling a student what is best for him, and advising him about what he
should do
d. Guidance is primary concerned with prevention rather than cure
105. Which of the following is NOT a guidance service?
a. Psychological testing
b. Counseling
c. Observation
d. Education placement
106. Which of the following tests is given at the end of instruction?
a. Summative
b. Placement test
c. formative test
d. diagnostic test
107. Special or exceptional students are such in certain aspects, Except for one:
a. Sensory abilities
b. Gender orientation
c. Mental characteristics
d. Communication abilities
108. Information from achievement tests can be used in the following, Except in:
a. Grading a student
b. Labeling pupils
c. Establishing proficiency
d. Giving feedback regarding effectiveness of learning
109. Which of the following occurs when the teachers have a general tendency to rate all
individuals’ performance at approximately the same position on the rating scale?
a. Personal bias error
b. Halo effect
c. Logical error
d. Severity error
110. The environment must be interactive to facilitate learning, which of the following
situations is an example of this?
a. The class copies a list of facts concerning the habitat of insects
b. The teacher lectures on the habitat of insects
c. The class goes out and discovers the habitat of insects
d. The teacher shows posters of the habitat of insects
111. When teachers, administrators and counselors come together to synthesize or coordinate
in interpreting data about a student for the purpose of a more intensive study, this is called
a. Longitudinal study
b. Case study
c. Case conference
d. Case analysis
112. Which type of test can help teachers predict the probability of success of students in
certain areas or endeavors?
a. Achievement
b. Intelligence
c. Aptitude
d. Personality
113. Which of the following correctly describes dyslexia?
a. It is a developmental expressive writing disorder
b. It is a developmental arithmetic disorder
c. It is a developmental reading disorder
d. It is a developmental articulation disorder
114. Free and compulsory education as mandated in the 1987 constitution holds true
for_____________
a. Elementary and secondary education
b. Education in the primary grades
c. All children of school age
d. Grade VI pupils

110

Downloaded by Noime Burtanog (noimeburtanog@gmail.com)


lOMoARcPSD|38673565

115. Readiness pretests is given at the beginning of an instructional unit is a:


a. Summative tests
b. Formative test
c. Placement test
d. Diagnostic test
116. The distribution of the scores is negatively skewed. This means that:
a. Most of the score are low
b. Most of the scores are high
c. The score is concentrated on the left of the distribution curve and most of the score
are low
d. The scores are concentrated on the left of the distribution curve
117. Aside from professional competence, what other important qualification must a teacher
have for guidance work?
a. Similar cultural background with the students
b. Personal qualities, such as warmth, open-mindedness, interest in people
c. Affiliation with a professional group
d. Several years of experience as teacher
118. Which of the following is meant to determine student performance at the beginning of
instruction?
a. Diagnostic assessment
b. Placement assessment
c. Summative assessment
d. Summative assessment
119. Mean is to central tendency as standard deviation is to:
a. Discrimination
b. Level of difficulty
c. Correlation
d. Variability
120. The following characterize school guidance programs, Except one:
a. A part of every school activity
b. Separate from the general life of the school
c. A function sheared by all
d. Located in every part of the school
121. Which factor is most likely to make a school’s guidance program succeed?
a. Administrative support
b. The availability of the specialist
c. The favorable attitude of teachers toward guidance
d. Located in every part of the school
122. The results if this type of test serve as basis for remedial instruction. What is the type of
test referred to?
a. Prognostic test
b. Speed test
c. Diagnostic test
d. Achievement test
123. The difficulty index of a test item is 1. This implies that:
a. The test item must be moderate in difficulty because 50% got the item correctly and
50% got it wrongly
b. The test item must be very difficult because nobody got the item correctly
c. The test item must be neither difficult because nor easy because this depends on the
ability of the students
d. The test item must be very easy because everybody got the item correctly
124. In his conduct of item analysis Mr. Milanes discovered that a significant greater number
from the lower group of the class that from the upper group got item number 10 correctly.
This implies that:
a. The test item is reliable
b. The test item has a negative discriminating power
c. The item has a positive discrimination power
d. The tests item is valid
125. Guidance is concerned with
a. All students, even if some seem not to have problems
b. Students who manifest undesirable behavior only
c. Underachieving students only

111

Downloaded by Noime Burtanog (noimeburtanog@gmail.com)


lOMoARcPSD|38673565

d. Students with emotional problems only


126. Which statement explains the primary focus of the non-directive approach in counseling?
a. It focuses in the person’s responsibility and capacity to discover more appropriate
behavior
b. It places confidence in a person’s ability to deal with his or her needs through a
realistic process
c. it helps confidence to develop a more rational to deal with his or her needs through a
realistic process
d. it modifies behavior by providing appropriate learning conditions and experiences
127. An algebra test was designed to measure what the students learned at the end of the
course. It was also to predict success in future mathematics subjects. This algebra test
functioned as-
a. Diagnostic and placement test
b. An aptitude test
c. An achievement test
d. An aptitude and an achievement test
128. Giftedness is a form of exceptionality. Students who fall under this category demonstrate
high performance in special areas, such as those below-
a. Athletic prowess
b. Creative thinking
c. General intellectual ability
d. Visual and performing arts
129. Which of the following is the least stable measure of central tendency?
a. Median and mode
b. Mode
c. Median
d. Mean
130. Which assessment is concerned with identifying learning difficulties during instruction?
a. Summative assessment
b. Placement assessment
c. Formative assessment
d. Visual and performing arts
131. These are significant information about a student, gathered through the use of various
techniques, assembled, summarized and organized in such a way that they may be used
effectively:
a. Cumulative records
b. Test profile
c. Personal inventories
d. Case studies
132. Research findings show that autism is
a. Either more prevalent among girls or among boys depending on their nationality
b. Equally prevalent among boys and girls
c. More prevalent among boys than among girls
d. More prevalent among girls than among boys
133. In interpreting test results which statements are true?
I. A raw score itself is meaningless but becomes meaningful once it is interpreted
II. An analysis of test results is useless without interpretation
III. Test result interpretation is possible without analysis
IV. The use of statistical techniques gives meaning to student’s scores
a. I, II, IV
b. I, II, III
c. I, III, IV
d. II, III, IV
134. This guidance service to help students carry out their plans and act on their choices so
that they become adjusted to their chosen field or career is called:
a. Counseling
b. information
c. placement
d. follow-up
135. The tendency for the rather to use only the lower end of rating scale in rating
performance is referred to as
a. Personal bias error

112

Downloaded by Noime Burtanog (noimeburtanog@gmail.com)


lOMoARcPSD|38673565

b. Severity error
c. Generosity error
d. logical error
136. What must be employed to continuously strengthen a school guidance program
a. Assignment of responsibilities
b. research
c. Evaluation
d. Program Planning
137. Which tests are given before instruction?
a. Placement
b. Formative
c. Summative
d. Achievement
138. The operation “Return to the basics” saw its embodiment in the _________
a. National Elementary Achievement Test
b. New Secondary Education Curriculum
c. National Secondary Achievement Test
d. New Elementary School Curriculum
139. The following are computed difficulty indices: 1, . 80, .50, .30 which one indicates the
most difficulty item?
a. 30
b. 50
c. 1
d. 80
140. Which of the following explains why the plateau phenomenon of ten found in human
being does NOT exist in animal learning experiments?
a. Animals are unable to profit from errors
b. The incentive in animal learning is constant
c. Animals have weak incentive in learning
d. Animals are unable to spurt in learning
141. All are components of remedial guidance in the classroom, Except:
a. Dealing with discipline problems
b. Dealing with poor study habits and skills
c. Dealing with underachievement
d. Dealing with career choices or vocational plans
142. Which method provides a more or less objective assessment of different aspects of an
individual?
a. Home visits
b. Counseling interview
c. Observations
d. Standardized testing
143. Which of the following behavior indicates that a child has developed conventional
maturity? The behavior is based on _________
a. The desire to avoid severe physical punishment by a superior authority
b. Personal decisions based on his satisfaction
c. The expectations of the group or society in general to gain approval
d. Internalized ideals to avoid self-condemnation rather than social censure
144. A school guidance program should arise mainly out of:
a. The curriculum and other programs of the school
b. The school administrator’s directives from his/her perceptions
c. The students’ needs and the problems
d. The availability of guidance resources in the schools
145. A test in which the options are dependent upon a foundation of some sort such as
graphical representation, paragraph, and pictures is:
a. Contained-options test
b. Setting-and-options test
c. Structured-response
d. Stem-and-options test
146. In the scoring of essay tests, experts advise teachers to score all answer to an easy
question before going to the next question for which reason?
a. It reduce the chances that ratings will be influenced by a halo effect
b. It is more convenient

113

Downloaded by Noime Burtanog (noimeburtanog@gmail.com)


lOMoARcPSD|38673565

c. It is easier to apply the criteria more uniformly when considering only a single task at
a time when going from task to task for each student
d. Checking is faster this way
147. Whose performance does stanine of 5 represent?
a. Performance of the upper 40%
b. Performance of the upper 20%
c. Performance of the lower 20%
d. Performance of the middle 20%
148. In making and reporting observations of students’ behavior, which ones should be
avoided?
a. Descriptions of student’s gestures and expressions
b. The names of other students with whom the one being observed interact
c. Opinions and judgment about students
d. Direct quotations of student’s statements
149. This is the stage when the learner become confused and starts to experience identity
crisis. This is the ____________
a. Late childhood stage
b. Early childhood stage
c. Early adulthood stage
d. Adolescent stage
150. All are basic functions of psychological tests EXCEPT ONE.
a. For selection or admission
b. Identification of students who need special attention
c. For promotion or acceleration
d. Integration of life experiences and future directions of students
151. Upon what shall a teacher base the evaluation of the learner’s scholarship
a. Merit, learner’s scholarship
b. Merit, quality of performance
c. Merit, attendance
d. Quality of performance, learner’s scholarship
152. The following are some of what a professional teachers may NOT do EXCEPT:
a. Accept remuneration from tutorials other than what is authorized for such service
b. Make deductions from student’s scholastic ratings as a punishment
c. Base the evaluation of the learner’s work on merit and quality of academic
performance
d. Inflict corporal punishment an offending learner
153. The value of education, deference to authority, and filial piety in the Philippines in an
influenced of
a. Taoist tradition
b. Confucianist tradition
c. Buddhist tradition
d. Hindu tradition
154. Which statement regarding teacher’s ethical behavior is wrong?
a. A teacher may not fall in love with his/her students
b. Teachers shall support one another at all times when the best interest of the learners,
the school, or the profession at stake
c. A teachers may submit to the proper authorities any justifiable criticisms against an
associate
d. It is every teacher’s responsibility to seek correctives for any unprofessional and
unethical conduct of any associate
155. Which is the first and foremost concern of a teacher? The interest and welfare of:
a. Learners
b. Colleagues
c. Parents
d. The teaching profession
156. The ideal society in the context of Plato is one where:
a. There is hope
b. Love prevails
c. Justice reigns
d. Faith matters
157. The survivors of a shipwreck are now packed in a lifeboat which may capsize unless
partly reloaded. Which of the following acts will be morally justifiable?

114

Downloaded by Noime Burtanog (noimeburtanog@gmail.com)


lOMoARcPSD|38673565

a. Pushing the young and old ones and leaving them behind to drown
b. Trying to save everyone as much as possible
c. Shooting some passengers and leaving them behind
d. Doing nothing and waiting for the boat to capsize
158. For the advocates of value clarification, values are
a. Universal
b. Personal
c. Objective
d. Unchanging
159. With regard to business, which does the Code of Ethics NOT say about teachers?
a. A teacher shall maintain a good reputation with respect to debts loans and other
financial matters
b. No teachers shall be financially interested in any commercial venture involving
textbooks and other school commodities where he/she can exercise official influence
c. no teacher shall act as agent of textbooks and other school commodities where he/she
can exercise official influence
d. a teacher may not engage in any kind of business
160. The more consumeristic a person is, the more he/she gets attached to this material world,
the farther he is from the universal self, the more miserable he/she becomes. This is a
teaching
a. Confucianism
b. Taoism
c. Hinduism
d. Buddhism
161. Who believed that there should be less reliance on the school, and the world of work?
a. Paolo Freire
b. Ivan Illich
c. Paul Goodman
d. Jean Paul Sartre
162. Human dignity is inherent in every person. This means that
a. The inner worth of a person depends on what he has
b. The inner worth of a person depends on how he/she looks
c. The inner worth of a person depends on what he can do
d. The inner worth of a person is in him/her she looks
163. Which of the following runs counter to the development of honesty and accountability?
a. Being vigilant about weights and measures used in the markets
b. Equal payment of government employee’s productivity pays
c. Payment of just wages to workers and employees
d. Transparent in operations of the barangay and other organizations through public
reporting
164. Which are said to be our wellspring of excellence?
a. Third largest English-speaking nation, tiger economy, inventor of people’s power
revolution
b. Tiger economy, superior human resources
c. Tiger largest English-speaking nation, superior human resources, inventor of people’s
power revolution
d. Tiger economy, the inventor of people’s power revolution, superior, human resources
165. Which among these goals for change was proposed by the moral Recovery Program?
a. A sense of justice and outrage over its violation, a sense of national pride, a sense of
seriousness
b. A sense of national guide, a sense of seriousness, sense of common good
c. A sense of national guide, a sense of seriousness, sense of national justice and outrage
over its violation
d. A sense of national guide, a sense of the common good, a sense of justice and outrage
over its violation
166. The code of ethics for teachers states that school officials shall encourage and attend to
the professional growth of all teachers. According to the Code school officials can do this by:
a. Giving them due recognition for meritorious performance by allowing them to
participate in conferences and training programs recommending teachers for
promotion
b. Recommending teachers for promotion, organizing teachers in a professional
organization

115

Downloaded by Noime Burtanog (noimeburtanog@gmail.com)


lOMoARcPSD|38673565

c. Allowing them to participate in conferences and training programs


d. Organizing teachers into a professional organization, giving them due recognition for
meritorious performance and recommending teachers for promotion
167. Which thrust on value formation is intended to help the students identify and be aware of
their values?
a. Analysis
b. Value clarification
c. Value inculcation
d. Moral development
168. The idea that there should be less reliance on the school and greater use of education
potential of the community and the world of work is the essence of:
a. Community-based education
b. Formal education
c. Multi-grade teaching
d. Mobile teaching
169. It is said that the big drama development is played out in small scenes. Which one does
not serve as a further explanation of this statement?
a. Ordinary things done by ordinary people are the solid building blocks on which a
nation develops
b. Details can make a significant difference
c. Much of our personal development and progress of the nation depends on how
extraordinary well we do the ordinary things
d. Let us give attention to major things because they are the ones that matter
170. Which of the following characteristics does not apply to Plato’s just society?
a. The artists are creative
b. The leaders are wise
c. The soldiers are courageous
d. The working class are temperant
171. Martin Buber’s “dialogic principle” requires as restructuring of society which can be
achieved by:
a. Communism which cannot tolerate multiplicity and freedom
b. Fascism which cannot allow dialogue
c. Democracy which requires centralistic political set up
d. Democratic socialism which advocates a genuine dialogue life
172. Which of the following statements about society is TRUE?
a. Revolution produces the opposite of their goals
b. Ethnic cleansing leads to social unanimity
c. “Social education” leads to a mature society
d. Cooperative education is a means to social education?
173. Which of the following statements defines ethics?
a. The science of correct thinking
b. The study of being a general
c. The study of the nature of human knowledge
d. The study of rightness or wrongness of human knowledge
174. Which of the following dose not characterize fascism?
a. A government of laws and not of men
b. Rigid hierarchy and authority of single leader
c. Militarist, nationalist and dictatorial regime
d. It justifies any of its means by its end
175. Which of the following is not Anarchism?
a. Freedom and responsibility
b. Social organization without hierarchy
c. Extreme view of individual freedom
d. Destruction of the state and its supporting institution
176. In which of the following acts would you attribute responsibility?
a. A barkeeper sells liquor to a minor not knowing he is an adult
b. A barkeeper sells liquor to a minor not knowing it is illegal to do so
c. A barkeeper sells liquor to a client not knowing it is contaminated with poison
d. A barkeeper sells liquor not knowing it would cause the death of a diabetic person
177. The values of the people are revealed in the following approaches, Except One
a. When they are asked to choose freely among alternative
b. When they act out what they choose but for a short while and not repeatedly

116

Downloaded by Noime Burtanog (noimeburtanog@gmail.com)


lOMoARcPSD|38673565

c. When they act out what they choose


d. When they are asked to choose among alternatives
178. To attain harmony in society people must rule their subjects with benevolence. whose
teaching was this?
a. Mohammad
b. Kung-fu-zu
c. Buddha
d. Lao Tzu
179. The concept that school failed to encourage positive learning and were opposed to the
production of independently-minded adults was popularized by:
a. Paolo Freire
b. John Dewey
c. Paul Goodman
d. Ivan Illich
180. Which type of conscience is possessed by one who is extremely rigorous and is
constantly afraid of committing evil?
a. Scrupulous
b. Lax conscience
c. Certain conscience
d. Certain conscience
181. The following EXCEPT ONE are the strength of Filipino character
a. Pakikipagkapwa-tao
b. Family orientation
c. Frankness
d. Joy of humor
182. Which of the following is the Aristotelian definition of happiness?
a. One’s ultimate achievements in life
b. A state made perfectly the aggregation of good things
c. The satisfaction of all desires and the absence of pain
d. The experience of the fulfillment of possibilities
183. One of the following statements about passion is NOT TRUE?
a. Passion does not affect voluntariness
b. Passion may completely destroy freedom
c. passion tends to blind the judgment of the intellect
d. passion cannot affect freedom
184. Which does not characterize a non-violent society?
a. Solves problems in non-adversarial way
b. Prefers indigenous ways of solving problems
c. A passive society
d. Solves problems in a non-confrontational way
185. Which statement holds true of values clarification?
a. Values are objective; no person has the right set of values to pass on to others
b. It is meant to help students get at their own feelings, ideas, and belief’s; no person has
the right set of values to pass on to others
c. No person has the right set of values to pass on to others; values are independent of
time
d. Values are independent of time, place, and persons; meant to help students get at their
own feelings and ideas
186. Based on the Code of Ethics for teachers, which of the following may teachers NOT do?
a. Welcomes the opportunity to lead in barangay
b. Studies and understands local customs and traditions
c. Deducts points from students’ scholastic ratings as a form of punishment
d. Attends church and worship of his choice joy and humor
187. Which type of justice implies the duty of one individual to give another what he is due?
a. International justice
b. Distributive justice
c. Social justice
d. Commutative justice

PROFESSIONAL EDUCATION BULLETS


1. The most reliable measure of central tendency when there are extreme scores

117

Downloaded by Noime Burtanog (noimeburtanog@gmail.com)


lOMoARcPSD|38673565

Median
2. Iah's score from her LET are the following: 92, 88, 91. What is the median?
91
3. What can be inferred from a low standard deviation?
Scores are homogenous
4. A high negative discrimination index means that:
More from the bottom group answered the test question correctly
5. When a student scored P80 on a single test, that means:
He scored higher than 80 percent of the class
6. Which of the following is an accurate depiction of an authentic assessment?
Performing first-aid measures to scald burns
7. Which of the following is a characteristic of a norm-referenced testing?
The performance depends on the scores of his batch mates
8. Which of the following is a correct statement about validity and reliability?
A valid test is always reliable
9. Which of the following is a characteristic of a norm-referenced testing?
The performance depends on the scores of his batch mates
10. A difficulty index of .92 means that the item is:
Very easy
11. Which of the following is an accurate depiction of an authentic assessment?
Performing first-aid measures to scald burns
12. Which of the following is an example of a formative test?
Quiz
13. Which of the following types of tests is most vulnerable to biases?
Essays
14. A leptokurtic distribution signifies that:
All scores are average
15. If a distribution is skewed to the right, this implies that:
The scores are very low
16. Which of the following is the best method to increase the reliability of a test material?
Increase the number of items
17. This thinking strategy refers to narrowing down ideas from big concept to smaller ones?
Convergent Thinking
18. This refers to the collection of works, artifacts, and pieces of a student and may serve as a
basis for assessment:
Portfolio
19. Which of the following does not describe a holistic rubric in scoring?
It uses at least 3 or more basis
20. Which of the following types of tests is most vulnerable to biases?
Essays
21. Withitness means:
That the teacher knows everything that happens around the four corners of the classroom
22. When a teacher jumps from one topic to another without assessing of the students are ready
to absorb the instructions, this practice depicts:
Thrust
23. What can be inferred from a low standard deviation?
Scores are homogenous
24. If a distribution is skewed to the right, this implies that:
The scores are very low
25. When a teacher is able to perform multiple, different activities at the same, this shows:
Overlapping
26. Which is an incorrect practice in test construction?
When using numbers as options, arrange them in a descending order
27. Under the new version of Bloom's Taxonomy of conitive processes, which of the following
belongs to the top level?
Generating
28. Which is the basic form of cognitive process according to Bloom's?
Knowledge
29. Which of the following forms of punishment is least likely to affect students negatively?
Surprise quiz
30. The following are examples of extrinsic motivation, except?
Community service

118

Downloaded by Noime Burtanog (noimeburtanog@gmail.com)


lOMoARcPSD|38673565

31. This is also known as the Enhanced Basic Education Act of 2013.
K-12
32. Subject matter, Activities, Evaluation, Assignment, Sequence of a lesson plan
Determine the objectives
33. A student makes the teachers and his classmates busy and asks everyone to give him special
attention. What is the student's hidden message?
The students want to feel connected
34. Which of the following serves as a pre-requisite to employment and will ensure that only
competent teachers will be granted privilege to teach to schools?
Licensure and registration
35. Which of the following correctly describes inductive reasoning?
Arriving to a main idea demo smaller topics
36. Which of the following laws prescribed licensure examination for teachers and will
strengthen the regulation of the practice of teaching in the Philippines?
RA 7836
37. Which of the following will disqualify a teacher to become a member of the board of
professional teachers?
A naturalized Filipino citizen that has 15 years of continuous teaching experience
38. Which of the following shall receive the highest budgetary allocation according to the
Philippine Constitution?
Education
39. According to the law, she can be referred to as a para-teacher. Which of the following is not
correct about para-teachers?
These teacher failed to reach the minimum GWA of 75 but have grades below 75. Kylie
scored 74.80 from the recent LET.
40. Which of the following is most likely be related to existentialism?
Free choice
41. Which of the following is NOT true about periodic merit examination for teachers according
to RA 7836?
This shall encourage continuing personal growth and development
42. While teaching Mathematics, Teacher Janus noticed that his students show interests on
dancing. He then decides to change the topic and teaches concept about dancing and shows to
them dance performances from the internet. What philosophy of education is shown?
Progressivism
43. If a teacher wishes to enjoy study leave, she will get how many percent of her salary while on
study leave?
60%
44. What is the focus of education during the Commonwealth Period?
Nationalism
45. Which of the following is the focus of Spartan Education?
Military training
46. Teacher Vanessa receives a death threat because she gave her student a failing grade. She
knows that passing the student will make her less credible as a teacher but will save herself
from being harmed. She then decided to pass the student to avoid danger. Which of the
following is observed by Teacher Vanessa?
Principle of Double Effect
47. Which of the following actions of a teacher violated the Code of Ethics for Professional
Teachers?
Having practice prejudice and eliminating discrimination against any learner
48. This type of conscience makes the person see that he sins but actually he/she does not.
Scrupulous
49. K-12 curriculum is what type of subject-centered curriculum?
Spiral
50. Which is the closest to the real thing?
Performing real life task
51. Which of the following helps the learner to retain information best?
Doing
52. A globe is an example of a __________
Model
53. Which of the four pillars of learning focuses on the competencies of a certain individual on a
particular skill?
Learning to do

119

Downloaded by Noime Burtanog (noimeburtanog@gmail.com)


lOMoARcPSD|38673565

54. Which of the following involves enactive representations according to Bruner?


Action-based
55. Which does not show acculturation?
Learning from your American parents their traditions
56. A Venn Diagram is most likely to be used when:
Discussing the similarities and differences of prokaryotic and eukaryotic cells
57. Teacher Iah wants to show her students the percentage of Math major, science major, social
science major, english major and tle major based on the whole population of BSED students
in the school where she teaches. Which of the following types of graph should she use?
Circle graph
58. Which of the following traits of Filipino can be considered both a negative and a positive
trait?
Pakikisama mentality
59. The ability of a child to realize that 1 liter of bottled water has the same quantity even if the
water will be placed on a plastic pitcher or a pale is called:
Conservation
60. Ana is a 13-year-old high school student who thinks that death penalty should be
implemented in her own countries as she believes that there are other ways to punish
oppressors and teach them to be better citizens. She should be under what stage of Piaget's
theory?
Formal operational
61. Lawrence Kohlberg focuses more on which of the following aspects of child development?
Morality
62. Which does not show acculturation?
Learning from your American parents their traditions
63. A toddler wishes to go the restroom alone and wishes to care for himself if not treated
properly will have develop or lead to which of the following psychosocial stages based on
Erikson's model?
Doubt
64. This is known as a girl's psychosexual competition with her mother for the possession of her
father.
Electra Complex
65. The parts of human personality in which instinct and primary processes manifest:
Id
66. Which of the following shall receive the highest priority according to Abraham Maslow's
model on Warmth humanistic needs?

67. In Grace Goodell's Reading Skills Ladder, which of the following takes precedence?
Basic sight words
68. Which of the following is not true about Language Acquisition among children?
Chomsky explains in his model the children acquired language solely through exposure.
69. This model shows reading as an active process that depends on reader characteristics, the
text, and interactive the reading situation

70. A child who has spatial intelligence will most likely enjoy which of the following activities?
Solving puzzles
71. It is the average, most commonly used and in greatly affected by extreme scores.
Mean
72. Who is the proponent of respondent conditioning?
Ivan Pavlov
73. It is the middlemost of measures of central tendency and most reliable when there are
extreme scores.
Median
74. Which of the following teacher most likely demonstrates Bandura's work in teaching?
She shows the student how a dance should be performed.
75. It is the most frequently used measures of central tendency.
Mode
76. Which of the following refers to the law of effect by Thorndike?
If the end result will be beneficial to the student he/she will probably perform well.
77. It is the highest score minus the lowest score and is the simplest of measures of variability.
Range
78. ZPD is concept popularized by:

120

Downloaded by Noime Burtanog (noimeburtanog@gmail.com)


lOMoARcPSD|38673565

Vygotsky
79. It is how spread the scores are from the mean and most reliable measures of variability
Standard Deviation
80. It is the square of standard deviation.
Variance
81. Very easy/reject
0.81-1.00
82. Very difficult/reject
0-0.20
83. Easiness
Difficulty index
84. Differentiate Upper group to lower group
Discrimination index
85. Difficult/revise
0.21-0.40
86. More from upper group/retain
Positive Discrimination index
87. Moderate/retain
0.41-0.60
88. Easy/revise
0.61-0.80
89. More from lower group/reject
Negative discrimination index
90. Cannot determine/reject
Zero discrimination index
91. Measures what it intends to measure
Validity
92. Consistency
Reliability
93. Standard, criteria, specific target
Criterion referenced
94. Others, class, batch mates
Norm referenced
95. Scattered, far from the mean, heterogenous
High standard deviation
96. Pen and paper, multiple choice, cognitive
Traditional assessment
97. Divide by 9. Median is S5
Stanines
98. Clustered, near from the mean, homogenous
Low standard deviation
99. Real-life application, holistic
Authentic Assessment
100. Peakedness
Kurtosis
101. Before instruction, strengths and weaknesses
Diagnostic
102. Divide by 100. Median is P50
Percentile
103. Normal curve, bell-shaped, most scores-average, few scores are high and low
Mesokurtic
104. Taller, more peaked, almost if not all scores are average
Leptokurtic
105. Divide by 10. Median is D5
Decile
106. During instruction progress, gaps, quizzes
Formative
107. Flat curve, scores are heterogenous
Platykurtic
108. Divide by 4. Median is Q2
Quartile
109. After instruction, evaluative learning

121

Downloaded by Noime Burtanog (noimeburtanog@gmail.com)


lOMoARcPSD|38673565

Summative
110. Not biased, multiple choice, matching type, wide level of objectives, guessing
Objective
111. Guide for scoring
Rubrics
112. Biased, essay, wide sampling of ideas, bluffing
Subjective
113. Single basis
Holistic
114. Memorization
Knowledge
115. Multiple basis
Analytical
116. Understanding
Comprehension
117. Use
Application
118. Classroom Management Model
Kounin
119. Breaking down
Analysis
120. Eyes at the back
Withitness
121. Synthesis
Putting together
122. Multi-tasking
Overlapping
123. Jumping from previous topic to new topic and vice versa
Flip-flop
124. Judgment
Evaluation
125. Narrowing
Convergent thinking
126. Not able to return
Truncation
127. The students are not ready
Thrust
128. Widening
Divergent thinking
129. Increase a response
Reinforcement
130. Weakens a response
Punishment
131. General to specific
Deductive reasoning
132. Outside
Acculturation
133. Specific to general
Inductive reasoning
134. Inside
Enculturation
135. External, shallow, money
Extrinsic motivation
136. Cognitive development
Piaget
137. Internal, noble, social work
Intrinsic motivation
138. Senses and motor (0-2)
Sensorimotor
139. Hidden message
notice me
140. Keeps others busy

122

Downloaded by Noime Burtanog (noimeburtanog@gmail.com)


lOMoARcPSD|38673565

Attention seeking
141. Egocentric, children tend to see only their point of view (2-7)
Pre-operational
142. Hidden message:
I am hurting
143. Violent
Revenge seeking
144. (7-12)
Concrete operational
145. (12-above) Abstract reasoning
Formal operational
146. Hidden message:
I want to help
147. Gets bossy
Power seeking
148. Classical conditioning or respondent conditioning
Pavlov
149. Hidden message:
show me how
150. Refuses to participate
Isolation/Withdrawal
151. Operant conditioning
Skinner
152. Back to basics
Essentialism
153. Social learning theory Modelling

154. Imitation
Albert Bandura
155. Traditional
Perennialism
156. Trust vs. Mistrust
Infant
157. Free choice
Existentialism
158. Autonomy vs. Shame and doubt
Toddler
159. Change
Progressivism
160. Initiative vs. Guilt
Preschool
161. Social change
Social Reconstructionism
162. Industry vs. Inferiority
School age
163. Practical use
Pragmatism
164. Identity vs. Role confusion
Adolescence
165. Labor/vocational training
Japanese
166. Values, beliefs, affective, attitude
Idealism
167. 2 options but each has both positive and negative effect
Double effect
168. Senses
Empiricism
169. 2 negative options but the other one is less evil
Lesser evil
170. Intimacy vs. Isolation
Young adult
171. With knowledge and intention

123

Downloaded by Noime Burtanog (noimeburtanog@gmail.com)


lOMoARcPSD|38673565

Formal cooperation
172. Without knowledge and intention
Material cooperation
173. Create new knowledge
Constructivism
174. Pleasure
Hedonism
175. Generativity vs. Stagnation
Middle adult
176. More people will benefit
Utilitarianism
177. Environment
Behaviorism
178. Sure
Certain
179. Ego integrity vs. Despair
Late adult
180. Unsure
Doubtful
181. Survival
Pre-Spanish
182. Hypocrite
Pharisaical
183. Mouth
Oral
184. Religion
Spanish
185. Insensitive
Callous
186. Thinks she’s right but she’s not
Lax
187. Anus
Anal
188. Thinks she’s wrong but she’s not
Scrupulous
189. Nationalism
Commonwealth
190. Sex organs
Phallic
191. 10%
Read
192. 20%
Hear
193. Free basic education
American
194. None
Latency
195. 30%
See
196. 50%
Hear and see
197. Sex organs through opposite sex
Genital
198. Reader to book
Top down
199. 70%
Say and write
200. Book to reader
Bottom up
201. 90%
Do
202. Action

124

Downloaded by Noime Burtanog (noimeburtanog@gmail.com)


lOMoARcPSD|38673565

Enactive
203. Active process
Interactive
204. Images
Iconic
205. Language
Symbolic
206. Trend or progress
Line graph
207. Comparison
Bar graph
208. Percentage
Circle or pie graph
209. A process by which a conditioned response is lost.
Extinction
210. NCBTS
National Competency based -teacher standards

GENERAL KNOWLEDGE

WORLD GEOGRAPHY
i. The longest River in the World? Nile River
ii. The highest mountain peak in world? Mount Everest (8,850 m)
iii. The Largest Lake of the World? Caspian Sea (371,000 sq km)
iv. The deepest lake of the world? Lake Baikal
v. Highest dam in world?
Jinping-I Dam 305 m (1,001 ft) in China on Yalong river.
Second Highest is Nurek Dam 300 m (980 ft) Tajikistan on Vakhsh river
6. The largest Dam in World? Three Gorges Dam
7. The largest Mosque in World? Masjid al Haram (Saudi Arabia)
8. Longest road tunnel in the World? Lærdal Tunnel
9. Longest rail tunnel in the world? Seikan Tunnel
Seikan Tunnel (Japan - 53.85 Km) 1988
Gotthard base tunnel (57km) 2016
10. The world's highest elevation on land? Mount Everest
(The 1954 elevation of Everest, 29,028 ft. (8,848 m) was revised on Nov. 11, 1999, and
now stands at 29,035 ft. (8,850 m).
11. The world's lowest elevation on land? Dead Sea
(The Dead Sea is a salt lake bordering Jordan to the east, and Israel to the west. Its
surface and shores are 429 meters (1,407 ft) below sea level).
12. An imaginary line which runs across and passes through the centre of the Earth called?
Earth's axis
The Earth's axis tilts at an angle of 23.5° away from the plane of the ecliptic. It's
because of this tilt that seasons occur on Earth.
13. The longest day in Northern Hemisphere? June 22
The sun will reach its northernmost point in the sky known as the summer solstice on
June 22. Earth's closest star will seem to pause briefly before beginning its move
southward again. In the Northern Hemisphere, this marks the longest day of the year.
14. The shortest day in Northern Hemisphere? December 22
December 22 is called Winter Solstice in the Northern Hemisphere, where it is the
shortest day of the year.
15. The imaginary line passing round the Earth midway between the north and south poles
called? Equator
It divides the Earth into two equal halves: the Northern and Southern hemispheres.
16. The equidistant lines drawn east and west of the Greenwich Meridian known as? Longitudes
They denote angular distances of a place due east or west of the Greenwich Meridian.
They converge at the two poles.
17. The world's largest airport in terms of land area? King Fahd International Airport
King Fahd International Airport (KFIA) is the world's largest airport in terms of land area
located 20 kilometers northwest of Dammam, Saudi Arabia. The airport's basic
infrastructure was complete by the end of 1990, which allowed the Allied forces engaged
in the first Gulf War in early 1991 to use the field for the storage of military aircraft.

125

Downloaded by Noime Burtanog (noimeburtanog@gmail.com)


lOMoARcPSD|38673565

18. The world's busiest airport by passenger traffic? Hartsfield-Jackson Atlanta International
Airport
It has been the world's busiest airport by passenger traffic since 1998.
19. The world's largest land animal? African Bush Elephant
The adult African bush elephant generally has no natural predators due to its great size.
20. The largest bay in the world? Bay of Bengal
The Bay of Bengal, the largest bay in the world, forms the northeastern part of the Indian
Ocean. Roughly triangular in shape, it is bordered mostly by India and Sri Lanka to the
west, Bangladesh to the north, and Burma (Myanmar) and the Andaman and Nicobar
Islands to the east.
21. The world's smallest bird? Humming Bird
Male bee hummingbirds which live in Cuba, weigh 0.056 ounces and are 2.75 inches in
length.
22. The world's largest bird? Ostrich
The ostrich is the largest living species of bird and lays the largest eggs of any living bird.
23. The world's tallest building? Burj Khalifa
It is the tallest artificial structure in the world, standing at 829.8 m. serves of cobalt at
7,100,000 metric tons. The Democratic Republic of the Congo (DRC) currently produces
63% of the world's cobalt.
24. The world's longest big-ship canal? Suez Canal
The canal was constructed in the mid-1800s and as has been recognized as a maritime
route to be open at all times, to shipping vessels of all countries in order to facilitate
continuity in maritime trade operations irrespective of global conflicts.
25. The world's busiest ship canal? Kiel Canal
The Kiel Canal is a 98-kilometre-long freshwater canal in the German state of Schleswig-
Holstein.
26. The world's largest cemetery? Wadi Al-Salam Cemetery
It is also known as the valley of peace. It is more than 1400 years old. Wadi Al-Salam is
the most sacred cemetery of Muslims.
27. The world's largest church? Basilica of our lady of peace
The church building was inspired by the Basilica of St. Peter in 1989. It can
accommodate up to 18,000 people.
28. Country has the largest area? Russia
Russia is the world's largest country in area with 17,098,242 sq.km.
29. Country has the smallest area? Vatican
The world's smallest country is the Vatican. It is also known as the Holy See. This
country fits within the Italian capital city of Rome.
30. Country has the largest population? China
China has the largest population with 1,367,640,000 as on January 13, 2015.
31. Country has the smallest population? Vatican
The population is just over 800.
32. The world's largest desert? Sahara desert
Sahara is the largest desert in the world. It is in north Africa, which spans an area
measured at roughly 3.5 million square miles.
33. The world's largest dome? Cowboys Stadium
It replaced the partially covered Texas Stadium which opened in 1971 and served as the
Cowboys' home through the 2008 season. It was completed on May 27, 2009.
34. The world's longest epic? Mahabharata
Mahabharata is the longest epic in the world. Mahabharata has more than 74,000 verses,
long prose passages, and about 1.8 million words in total. Mahabharata was written by
Ved Vyasa in Sanskrit.
35. The world's tallest fountain? King Fahad fountain
King Fahad fountain is also called Jaddah fountain. It is visible throughout the Jeddah
city. It uses saltwater from Red Sea instead of fresh water.
36. The world's largest gorge? The Grand Canyon
The Grand Canyon is a gorge carved by the Colorado River and is contained in the Grand
Canyon National Park in Arizona. It is one of the first national parks in the United States.
37. The largest gulf in the world? The Gulf of Mexico
The Gulf of Mexico formed approximately 300 million years ago as a result of plate
tectonics. Plate tectonics is a scientific theory that describes the large-scale motion of
Earth's lithosphere.
38. The world's biggest hydroelectric plant? Three Gorges

126

Downloaded by Noime Burtanog (noimeburtanog@gmail.com)


lOMoARcPSD|38673565

The dam project was completed and fully functional as of July 4, 2012.
39. The world's largest island? Greenland
Greenland has no land boundaries and 44,087 km of coastline.
40. The world's largest sand island? Fraser Island
It was inscribed as a World Heritage site in 1992. The island is considered to be the
largest sand island in the world at 1840 sq. km
41. The deepest lake in the world? Baikal
Lake Baikal is the most voluminous freshwater lake in the world, containing roughly
20% of the world's unfrozen surface fresh water.
42. The world's tallest lighthouse? Jeddah Light
Jeddah Light is a concrete and steel lighthouse built in 1990. It is located in Jeddah in
Saudi Arabia. It has a range of 46 kilometers and emits three white flashes every 20
seconds.
43. The world's highest mountain range? Himalaya
Three of the world's major rivers the Indus, the Ganges and the Brahmaputra arise in the
Himalayas.
44. The world's largest mountain range? Andes
Located along the entire western coast of South America, the Andes mountain range is
about 4,500 miles (7,242 kilometer) long.
45. The world's oldest museum? Ashmolean museum
The Ashmolean Museum founded in 1677 from the personal collection of Elias Ashmole,
was set up in the University of Oxford to be open to the public.
46. The largest newspaper in the world by circulation? Yomiuri Shimbun
The Yomiuri Shimbun is a Japanese newspaper published in Tokyo, Osaka, Fukuoka, and
other major Japanese cities.
47. The world's largest ocean? The Pacific Ocean
The Pacific separates Asia and Australia from the Americas. It may be further subdivided
by the equator into northern (North Pacific) and southern (South Pacific) portions. It
extends from the Antarctic region in the South to the Arctic in the north.
48. The world's largest peninsula? Arabian peninsula
The Arabian Peninsula is located in Southwest Asia and contains the countries of Jordan,
Iraq, Kuwait, Bahrain, Qatar, United Arab Emirates, Oman, Yemen and Saudi Arabia.
49. The world's rainiest place? Cherrapunji
This place under the East Khasi Hills district of Meghalaya is also called the rain capital
of the world. This place is locally called Sohra.
50. The world's hottest place? Death Valley
It is the lowest and driest area in North America.
51. The world's largest railway station by platform capacity? Grand Central Terminal
Grand Central Terminal is located in New York City, USA. It has 44 platforms.
52. The world's longest motorable road? Pan American highway
The Pan-American Highway system is mostly complete and extends from Prudhoe Bay,
Alaska, in North America to the lower reaches of South America.
53. The world's largest sea? Philippine sea
The Philippine Sea is a marginal sea and a part of the North Pacific Ocean. It is the
largest sea in the world with reference to the surface area.
54. The tallest statue? Spring Temple Buddha
The current tallest statue is the Spring Temple Buddha located in the Zhaocun township
of Lushan County, Henan, China. It is a statue depicting Vairocana Buddha and was built
in 2002.
55. The tallest structure in the world? Burj Khalifa
The building officially opened on 4 January 2010.
56. The largest temple in the world? Angkor Wat
Angkor Wat (City Temple) is a vast temple complex in Cambodia. The Angkor temple
stands on a raised terrace above the rest of the city.
57. The tallest living tree in the world? Coast Redwood
The coast redwood has a conical crown, with horizontal to slightly drooping branches.
58. The longest railway tunnel? Seikan tunnel
It passes through Tsugaru Strait connecting the Hokkaido and Honshu islands.
59. The highest waterfall in the world? Angel falls
It is a waterfall in Venezuala.
60. Longest Canal in the World? Panama Canal

127

Downloaded by Noime Burtanog (noimeburtanog@gmail.com)


lOMoARcPSD|38673565

This canal Connects Mediterranean Sea and Red Sea. After 10 years of Construction, it is
opened in 1869.This Canal is managed by Egypt
61. The Largest Ocean in the World? Pacific
62. The deepest lake in the world? Baikal
It is a fresh water lake located in Siberia, Russia
63. Highest Waterfalls in the world? Angel falls
These falls are on the Gauja river located in Venezuela. Height of these falls are 3,212
feet.
64. Smallest Bird in the World? Hummingbird
These birds weigh generally 1.5 -2 gm and length is 5-6 cm. These are very fast flying
birds.
65. The Largest diamond in the world? Cullinan
The largest stone is called the "Star of Africa I," or "Cullinan I," and at 530 carats, it is
the largest-cut fine-quality colorless diamond in the world. The second largest stone, the
"Star of Africa II" or "Cullinan II," is 317 carats.
66. By area Which country is Biggest in the world? Russia
It covers one eighth of the earth with 170,75,400 square kilometers
67. By Electorate Which Country is largest? India
The Indian National Congress, Credit: Austin Yoder, CC BY ND 2.0.
With more than 814 million electorate India stands as the largest democracy of the world.
At the time of its independence, few expected it to survive as a democracy given a large,
poor and uneducated population
68. Largest desert in Asia is? Thar
It is in China covers parts of North and Northwest China.
69. Largest Continent in the World? Asia
70. Smallest Continent in the World? Australia
71. Tallest Statue in the World is ____? Spring Temple Buddha
This statue is located in Lushan County, China. The height of the statue is 420 ft
72. Biggest Park in the world? Yellow stone park
73. Biggest flower in the world? Rafflesia
The flower with the world's largest bloom is the Rafflesia arnoldii. This rare flower is
found in the rainforests of Indonesia. It can grow to be 3 feet across and weigh up to 15
pounds! It is a parasitic plant, with no visible leaves, roots, or stem.
74. Which bird lays largest egg? Ostrich
These eggs weigh 1.45 kilograms.
75. Smallest Ocean in the world? Arctic
76. Shortest river in the world? Roe
The Roe River is recognized by the Guinness Book of World Records as the world's
shortest river. The Roe flows 200 feet between Giant Springs and the Missouri River near
Great Falls. The Roe River competes with the D River in Lincoln City, Oregon for the
title of the shortest river.
77. Fastest Running Animal on land? Cheetah
The fastest land animal is the cheetah, which has a recorded speed of 109.4–120.7 km/h
(68.0–75.0 mph). The peregrine falcon is the fastest bird and the fastest member of the
animal kingdom with a diving speed of 389 km/h (242 mph).
78. Busiest international Airport in the world? Dubai International Airport
79. Coldest Place in the World? Vostok
Vostok is operated as a permanent Antarctic station by the Russian Federation and carries
out a wide range of scientific activities. It is situated in the center of East Antarctica on
the Polar Plateau at an altitude of 3,488 meters and is the most isolated of any Antarctic
Base. It is also known as the coldest place on earth.
80. Largest delta in the World? Sunderbans
It is also called as green delta as the region is more fertile land.it is also known as Ganges
delta, Ganges -Brahmaputra delta.
81. Largest peninsula in the World? Arabia
It has to be the Arabian Peninsula, it is the largest and longest, spanning at 3,237,500
square kilometers, the peninsula consists of the countries of Yemen, Oman, Qatar,
Bahrain, Kuwait, Saudi Arabia and the United Arab Emirates as well as parts of southern
Iraq and Jordan.
82. The Largest Library in the World? United states Library of Congress
This library is established 200 years back.
83. Largest Railway Station in the World? Grand Central Terminal,Newyork

128

Downloaded by Noime Burtanog (noimeburtanog@gmail.com)


lOMoARcPSD|38673565

84. Biggest Iron ore mine is located in which country? Brazil


The name of the biggest iron ore mine is Carajas.
85. Largest Diamond mine is situated in which country? Botswana
Orapa is the largest diamond mine located in Orapa town. It is considered as diamond
capital of the country.
86. The largest reserve of Bauxite of the world? Australia
Guinea has the world's largest bauxite reserves and is the biggest exporter of the ore
while Australia tops the world in mine production. — The Huntly mine in Australia,
owned by Alcoa World Alumina, is the world's largest, producing 18 million tonnes in
2006
87. Countries does India have its longest international boundary? Bangladesh (4,096.7)
88. Countries is famous for livestock ranching in the world? Argentina
Ranching is the practice of raising herds of animals on large tracts of land. Ranchers
commonly raise grazing animals such as cattle and sheep. Some ranchers also raise elk,
bison, ostriches, emus, and alpacas. The ranching and livestock industry is growing faster
than any other agricultural sector in the world.
89. The country with the world's largest natural gas reserves is? Russia
Nearly 80% of the world's total proven natural gas reserves are located in ten countries.
Russia tops the list, holding about a quarter of world's total gas reserves, followed by Iran
and Qatar in the Middle East.
90. "The Three Gorges Dam", the world's largest hydro-power project, is located in? China
The Three Gorges Dam is a hydroelectric gravity dam that spans the Yangtze River by the
town of Sandouping, located in Yiling District, Yichang, Hubei province, China. The
Three Gorges Dam is the world's largest power station in terms of installed capacity
(22,500 MW).
91. The largest proven oil reserved of the world lies in? Venezuela
Proven oil reserves in 2013 (billion barrels)
92. The longest rock tunnel of the world is located in? Japan
Lærdal. Built in 2000, the tunnel is located in Norway (Lærdal -Aurland). It’s part of the
E16 road. It’s 24,510 km (15.2 mi) long. It’s 1 tube, and is the longest road tunnel in the
world.
Yamate Tunnel. Built in 2010-2015, the tunnel is located in Japan (Tokyo). It’s part of the
C2, Shuto Expressway. It’s 18,200 km (11.5 mi) long. It’s 2 tubes, and is the longest in-
city road tunnel in the world.
93. Countries has the lower density of population? Canada
94. Country is the leading producer of Uranium? Canada
Canada, the second biggest uranium producer in the world, hosts the world's biggest - the
McArthur River uranium mine - while Kazakhstan, the largest uranium producing
country, operates three of the top 10.
95. The largest irrigation canal in the world? Indira Gandhi canal
96. The world’s tallest waterfalls is? Angel Falls
World's Highest Waterfall. Angel Falls (Salto Ángel) in Venezuela is the highest waterfall
in the world. The falls are 3230 feet in height, with an uninterrupted drop of 2647 feet.
Angel Falls is located on a tributary of the Rio Caroni.
97. The world’s largest freshwater lake by volume is? Lake Baikal
The deepest lake in the world is Lake Baikal in Siberia. Its deepest point is 1637 meters.
It holds 20% of the world's fresh surface water and is the world's largest freshwater lake
by volume.
98. The world’s largest lake is the Caspian Sea
Caspian Sea is often regarded as the world's largest lake, though it contains an oceanic
basin (contiguous with the world ocean until 11 million years ago) rather than being
entirely over continental crust. Largest of the Great Lakes by volume, having more water
than the other four combined.
99. The world’s second highest mountain is K2.
K2 is the second-highest mountain on Earth, after Mount Everest and is located on the
border of Pakistan and China. It is also known as the Savage Mountain.
100. The world’s highest mountain is? Mount Everest
101. The world’s largest animal is? Blue Whale
102. The world’s largest land animal is? African bush elephant
The African bush elephant, also known as the African savanna elephant, is the larger of
the two species of African elephants and the largest living terrestrial animal.
103. The world’s largest reptile is Saltwater crocodile

129

Downloaded by Noime Burtanog (noimeburtanog@gmail.com)


lOMoARcPSD|38673565

The largest living reptile, a representative of the order Crocodilia, is the saltwater
crocodile (Crocodylus porosus) of Southern Asia and Australia, with adult males being
typically 3.9–5.5 m (13–18 ft) long.
104. The world’s largest bird is? Common ostrich
Common ostrich Height: Male: 2.1 – 2.8 m (Adult), Female: 1.7 – 2 m (Adult)
105. The largest eggs in the world today are the eggs laid by the Ostrich
106. The tallest tree in the world is Redwood or Quindio wax palm
107. The largest peninsula in the world is Arabia
108. The biggest irrigation scheme in the world (Llyod Barrage) is located in Pakistan
109. The tallest minaret in the world (Sultan Hassan Mosque) is located in Egypt
110. Lake Superior, the largest fresh water lake in the world is located in Canada
Combined, Lake Michigan–Huron is the largest fresh water lake by area in the world. If
Lake Huron and Lake Michigan are considered two separate lakes, Lake Superior is
larger than either. location: United States, Canada.
111. The biggest city in world in area is New York.
112. The world’s first artificial satellite was Sputnik 1.
History changed on October 4, 1957, when the Soviet Union successfully launched
Sputnik I. The world's first artificial satellite was about the size of a beach ball (58 cm. or
22.8 inches in diameter), weighed only 83.6 kg. or 183.9 pounds, and took about 98
minutes to orbit the Earth on its elliptical path.
113. What was the first satellite in the Philippines? DIWATA1
114. The longest river in Europe is Volga River
115. The first person who entered into space was Yuri Gagarin
116. The first person who walked on the Moon was Neil Armstrong
117. The first spacewalk was performed by Alexey Leonov
118. The first space station was Salyut 1
119. The first moon walker (remote-controlled robot) was Lunokhod 1
120. The first spacecraft to LAND on the Moon was Luna 9
The Luna 9 spacecraft, launched by the Soviet Union, performed the first successful soft
Moon landing on 3 February, 1966.
121. The first spacecraft to land on the Venus was Venera 7
On December 15, 1970 an unmanned Soviet spacecraft, Venera 7, became the first
spacecraft to land on another planet.
122. The first spacecraft to land on the Mars was Mars 3
After the Mars 2 lander crashed on the Martian surface, Mars 3 lander became the first
spacecraft to attain soft landing on Mars on 2 December 1971.
123. The world’s oldest surviving federation is United States
124. The highest "cold desert" in the world is Katpana Desert
Katpana Desert are located at Skardu, Gilgit-Baltistan in Pakistan, this desert is also
known as the "Cold Desert". Katpana Desert is situated at an elevation of 2,226 metres
above sea level.
125. The first female who led a Muslim state was Benazir Bhutto
Benazir Bhutto was a Pakistani politician who served as Prime Minister of Pakistan from
1988 to 1990 and again from 1993 to 1996. She was the first woman to head a democratic
government in a Muslim majority nation.
126. Which country has the largest number of UNESCO World Heritage Sites? Italy
127. Which country is the world’s largest producer of cobalt, a chemical element? DR Congo
Cobalt extraction. The United States Geological Survey estimates world reserves of
cobalt at 7,100,000 metric tons. The Democratic Republic of the Congo (DRC) currently
produces 63% of the world's cobalt.
128. The world’s largest landlocked country is Kazakhstan
Kazakhstan is the largest landlocked country in the world with an area of 2,724,900 km²
and the ninth-largest in the world.

PROF.ED 200 ITEMS QUESTION


1. As parent and the same time a teacher, which of the following will you do to show your
cooperation to the PTA project in your school to be financed with the proceeds of the sales of the
school canteen where food prices are little bit higher?
a. Bring food for you and your children, but always make it a point to buy in the school canteen.
b. buy all your food in the school canteen but request for a discount
c. bring food enough for you and your children but do not eat in the school canteen
d. buy all your food from the school canteen even if you cannot afford to do everyday

130

Downloaded by Noime Burtanog (noimeburtanog@gmail.com)


lOMoARcPSD|38673565

2. According to the existentialist, every person in the same predicament and has the same
possibilities. What does this imply?
a. Every person must go to school.
b. Every person must go through the same from school.
c. Every person must earn a college degree.
d. Every person must be given access to education.
3. You are very much interested in a quality professional development program for teachers.
What characteristic should you look for?
a. prescribed by top educational leaders
b. dependent and availability of funds
c. required for the renewal of professional license
d. responsive to identified teachers’ need
4. The singing of the National anthem is an offshoot of philosophy of ____.
a. Nationalism c. naturalism
b. pragmatism d. Socialism
5. The environment in order to facilitate, learning must be interactive. Which of the following
best typifies this kind of environment?
a. the child goes out and discovers for himself some rock or fossil.
b. the child listens to a lecture on the fossils given by the teacher
c. the child summarizes the section on the fossils in his science textbook
d. The child copies a list of facts concerning fossils on the blackboard.
6. A teacher is said to be a “trustee of the cultural and educational heritage of the nation and is
under obligation to transmit to learners such heritage”. Which practice makes the teacher fullfill
such obligation?
a. Use interactive teaching strategies.
b. Use the latest educational technology
c. Observe continuing professional education.
d. As a class, study the life of Filipino heroes.
7. For more efficient and effective management of schools as an agent of change, one proposal is
for the DepEd to cluster remote stand-alone schools under one lead school head. Which factor
has the strongest influence on this proposal?
a. Psychological c. Geographical
b. Historical d. Social
8. What does the acronym EFA imply for school?
a. the acceptance of exclusive schools for boys and for girls
b. the stress on the superiority of formal education over that of alternative learning system
c. practice of inclusive education
d. the concentration on formal education system.
9. Which Republic Act provides government assistance to the students and teachers in private
school?
a. RA 7784 c. RA 7836
b. RA 6728 d. RA 7722
10. the authoritarian setting in the Filipino home is reinforced by a classroom teacher who:
a. encourage pupils to ask questions
b. prescribes what pupils should do
c. is open to suggestions
d. ask open ended question
11. Who among believes that learning requires disciplined attention, regular homework and
respect for legitimate authority?
a. essentialist c. Progressivist
b. existentialist d. Reconstructionist
12. The Constitutional provision on language has the following aims EXCEPT:
a. to make the regional dialect as auxiliary media of instructions in regional school
b. to maintain English as second language
c. to make Filipino the sole medium of instruction
d. to make Filipino the national language and medium of instruction and communication
13. The tendency to emphasize o much on school beautification to the detriment of pupils
performance illustrates the:
a. Filipino’s lack of seriousness
b. Filipino’ lack of reflection
c. Filipino’s sense of humor
d. Filipino’s love for “porma”

131

Downloaded by Noime Burtanog (noimeburtanog@gmail.com)


lOMoARcPSD|38673565

14. Which I Not a characteristics of democratic discipline?


a. child has opportunity to express his/her opinion
b. child given punishment is related to the misdeed
c. child understand the meaning of rules
d. child obeys blindly
15. Who among the following reasons stressed the processes of experience and problem solving?
a. Dewey c. Socrates
b. Aristotle d. Plato
16. The wide Acceptance of ‘bottom up” management style has influenced the schools to practice
which management practice?
a. Exclusion of politicians from the pool of guest speaker during graduation exercises
b. prescription of what ought to be done from the central Office
c. Involvement of students, parents, teachers, and community in the school planning
d. Allowing schools to do what they think is best.
17. Which characterizes the perfectionist type of students?
a. does not volunteer or initiate
b. Give up easily
c. Rarely complete tasks
d. often anxious fearful or frustrated about the quality of work
18. The failure of independent study with mot Filipino students may be attributed to students’
a. unpreparedness of schooling
b. ambivalence
c. high degree of independence
d. high degree of dependence on authority
19. Despite opposition from some school officials, DepEd has continuously enforced the “no
collection of fees” policy during enrolment period in public schools. Is this policy in accordance
with EFA goals?
a. No, it violates the mandate of quality education
b. Yes, it somewhat eliminates gender disparities
c. Yes, it supports equitable access to basic education
20. Which of the following measures should a teacher do to a principal whom she would like to
file a case of sexual harassment w/o violating the relationship of the teacher to her superior?
a. present the case before competent authority and prepare to prove the charge
b. writ an anonymous letter to a higher school official to denounce the superior
c. call a parent-teacher meeting and denounced the superior
d. encourage the other teacher and students to hold a demonstration to oust the superior
21. To earn units for promotion, a teacher pays her fees but does not attend class at all. Does this
constitute professional growth?
a. Not immediately but yes after promotion.
b. It depends on the school she is enrolled in.
c. No, it simply earning MA units for promotion.
d. Yes, just enrolling in an MA program
22. Kounin claims that “with-it-ness” is one of the characteristics of an effective classroom
manager. What is one sign of with-it-ness?
a. Giving attention to students who are having difficulty with school work.
b. seeing only one portion of the class but intensively
c. knowing where instructional materials are kept
d. Aware of what’s happening I all parts of the classroom.
23. A student collapsed in her social studies class. It was found out that he did not eat her luch.
What principle is shown in the situation?
a. Psychological need c. Ecological need
b. Physiological need d. Safety need
24. Which techniques (s) enable (s) a teacher to identify and eventually assists students which
interpersonal difficulties?
a. Anecdotal record c. Cumulative record
b. Personal inventory d. Sociogram
25. Which is considered the “brain” of the computer?
a. CPU c. Video screen
b. Software d. keyboard
26. Zero standard deviation means that:
a. The student scores are the same
b. 50% of the scores obtained is zero

132

Downloaded by Noime Burtanog (noimeburtanog@gmail.com)


lOMoARcPSD|38673565

c. More than 50% of the score obtained zero


d. Less than 50% of the scores obtained zero
27. Which of the least authentic mode of assessment?
a. Paper-and-pencil test in vocabulary
b. Oral performance to assess students to socialize students spoken communication skills
c. Experiments in science to assess in the use of scientific methods
d. Artistic production for music or art subject
28. What must a teacher do to ensure orderly transitions between activities?
a. Allow time for the students to socialize in between activities
b. Have the materials ready at the start of the activity
c. Assign fewer exercises to fill the allotted time.
d. Wait the students who lag behind
29. When teacher tries to elicit clarification on a student response or solicits additional
information, which of these should he use?
a. Directing c. Structuring
b. probing d. Cross examining
30. A negative discrimination index means that:
a. More from the lower group answered the test items correctly
b. the items cloud not discriminate between the lower and upper group
c. more from the upper group answered the test item correctly
d. Less from the lower group got the test item correctly
31. ‘’When more senses are stimulated, teaching and learning become more effective.” What is
an application of this principle?
a. Appeal to students’ sense of imagination
b. use multisensory aids
c. make your students touch the instructional material
d. Use audio visual aids because the eyes and the ears are the most important senses in learning.
32. I combined several subject areas in order to focus on a single concept for interdisciplinary
teaching. What strategy did I use?
a. Reading-writing activity c. Lecture
b. Thematic instruction d. problem-centered learning
33. A teacher is a facilitator of learning and of the development of the youth. Which practice
NOT keeping with this role a facilitator?
a. Considers the multiple intelligence of learners
b. Humiliates misbehaving pupils
c. Dialogs with parents and with other member of the community
d. Keeps himself abreast with educational trends
34. Which one indicates a teacher’s genuine enthusiasm and pride in teaching?
a. Sticking to teaching for the moment that there are no better offers
b. Telling everyone that he went to teaching for there was no other choice them.
c. Engaging himself in continuing professional education
d. Belittling the re-numeration one gets from teaching
35. Teacher A teaches English as a second Language. She uses vocabulary cards, fill-in-the-blank
sentences, dialogues, dictation and writing exercises in teaching a lesson about grocery shopping.
Based on this information, which of the following is a valid conclusion?
a. The teacher wants to make her teaching easier by having less talk.
b. The teacher is emphasizing reading and writing skills
c. The teacher is teaching in variety of ways because not all students learn in the same manner.
d. The teacher is applying Bloom’s hierarchy of cognitive learning
36. To teach the democratic process to the pupils, Biag Elementary School decided that the
election of class officers shall be patterned after local elections. There are qualifications set for
candidates, limited period for campaign and rule for posting campaign materials, etc. Which of
the following did the school use?
a. Symposium c. Panel discussion
b. Simulation d. Debate
37. Which statement applies when scores distribution is negatively skewed?
a. The mode is lesser than the median c. The mode and median are equal
b. The median is higher than the mode d. The mean is lesser than the mode
38. In the Preamble of the Code of Ethics of Professional Teachers, which is not mentioned about
teachers?
a. Duly Licensed professionals c. LET passer
b. Possess dignity and reputation d. With high moral values

133

Downloaded by Noime Burtanog (noimeburtanog@gmail.com)


lOMoARcPSD|38673565

39. What does a skewed score distribution mean?


a. The scores are concentrated more at one end or the other end
b. The mode, the mean and the median are equal
c. The mean and the median are equal
d. The scores are normally distributed
40. Teacher C, a reading teacher, advised her class to “read between the lines.” What does she
want her pupils to do?
a. Make an educated guess
b. Determine what is meant by what is stated
c. Apply the information need
d. Describe the characters in the story
41. On which constitutional provision is the full or partial integration of capable deaf and blind
students in the classroom based? The provision on
a. Providing citizenship and vocational training to adult citizen
b. Protecting and promoting the right of all citizen to quality education
c. Academic freedom
d. Creating Scholarship for poor and deserving students
42. Teaching the cognitive, psychomotor and affective domains is based on the concept that the
learner is a:
a. Moral and feeling being c. Thinking, feeling and acting being
b. Material and an acting being d. Spiritual and material being
43. Which of the different types of the test covers and wide variety of objective?
a. True-false c. Matching type
b. Multiple choice d. Essay
44. Teacher wants to compare 2 concepts. With which technique can be accomplish this best?
a. K-W-L technique c. Spider web
b. Venn diagram d. Fishbone diagram
45. When necessary conditions are present, the use of inductive method is preferred because
a. There is greater active participation on the part of the pupils
b. It gives the teacher more time to rest
c. It needs only few instruction materials
d. Academic time is used wisely
46. Teacher B teaches his/her students that pleasure is NOT the highest good. What teacher
teaches is contrary to which philosophy?
a. Empiricism c. Hedonism
b. Realism d. Idealism
47. Which best indicates the effectiveness classroom activities?
a. The laughter and employment of students
b. The application of concept learned in daily life
c. The utilization of varied techniques and approaches
d. The variety instructional material used
48. Which is the most obvious and familiar way of reporting variability?
a. Range of scores c. Standard error of the mean
b. Standard deviation d. Distribution of raw scores
49. If the teacher’s pattern in questioning consists of calling on a student then asking the question
a. All students may be encouraged to participate
b. the student called to answer may be able to think well of his answer
c. The rest of the class may just dictate the answer
d. The rest of the class may not engage themselves in thinking of the answer
50. Teacher L says: “ If it is billiard that brings students out of the classroom, let us bring it into
the classroom. Perhaps, I can use it to teach Math.” To which philosophy does Teacher L adhere?
a. Reconstructionism c. Existentialism
b. Essentialism d. Progressivism
51. A child refuses to obey orders or displays negativism as a development trait. How may you
best handle him?
a. take every opportunity to praise him for every positive attitudes display
b. detain him after office hours for him to do what he has been ordered to do
c. insist on compliance to the same degree required of pupils
d. avoid giving him orders or if you do and he objects take the order back
52. Which term refers to the collection of student’s products and accomplishment for a period of
evaluating purposes?
a. Portfolio c. Anecdotal record

134

Downloaded by Noime Burtanog (noimeburtanog@gmail.com)


lOMoARcPSD|38673565

b. Observation report d. Diary


53. Which practice is an offshoot of B.F. Skinner’s theory of operant conditioning?
a. ensuring mastery of language c. use of scaffolding
b. use of programmed instruction d. considering multiple intelligence
54. In testing, which of the following is referred to as cultural bias?
a. Test items are more familiar in some culture
b. Some culture do better on tests than others
c. Test will show who is more cultured
d. Cultured people do better on tests
55. You have a pupil who is so talkative, naughty and aggressive that he is burden to the entire
members of the class. How would you remedy this problem?
a. Call the parent for a dialogue c. Reprimand him always
b. Report the case to the principal d. Talk to him seriously
56. Which of the following is the best situation wherein you can balance responsibility and
accountability?
a. A teacher paid on an hour basis, takes her time with the subject matter till end of the period
b. A teacher paid on an hour basis, teaches as much as she could for duration of the period
c. A teacher paid on an hour basis, spends most of the time on the latest gossips in showbiz
d. A teacher paid on an hour basis, entertain her students w/ stories till the end of the period.
57. Section 5, article XIV, of the Constitution states that academic freedom shall be enjoyed in
a. Public assemblies’ c. All level of learning
b. State college and Universities d. All institutions of higher learning
58. Which of the following should a teacher do if she cannot pay the monthly instalment of an
appliance she got from a department store in their own?
a. Reject any notice of demand for payment to make the impression that she will be receive
b. Move to another neighbourhood to escape payment
c. Inform the manager of the store personally and make a satisfactory arrangement of payment on
or before the due date of payment
d. Offer to return the used appliance to the store on the condition that she will be refunded on the
monthly instalment she paid.
59. Freud expounded that there is a period when young boys experience rivalry with their father
for their mother’s affection. This is
a. Oedipus Complex c. Achilles syndrome
b. Electra complex d. Cassandra syndrome
60. Education is a lifelong process. This simply means that education-
a. May take place formally or informally to enable the individual to grow
b. May take place anywhere and anytime the individual so desire
c. Is a Continuous process of experiencing and recognizing experience
d. Take place in the school where the individual is expose, self-contained experiences.
61. The tendency to imitate elders is very strong in the early childhood stage. Teacher should
therefore be very good-
a. Counselors c. Role model
b. Disciplinarians d. Facilitator of Learning
62. Teacher Z always checks on entry knowledge and skills before she proceeds to her new
lesson. On which principle is Teacher Z practice grounded?
a. Effective teaching proceeds from the concrete to abstract
b. learning increase when the learning is relevant
c. Attention is essential for learning
d. New learning builds on previous learning
63. Which of the following is the best time for a teacher to set up routine activities that will
contribute to effective classroom management?
a. As soon as the student have established
b. Daily as the start of the session
c. During his homeroom day
d. On the very first day of school
64. Teacher R wants to develop his students’ creativity. Which type of questions will be MOST
appropriate?
a. Synthesis questions c. “What if….” questions
b. Fact questions d. Analysis question
65. You want your student to answer the questions at the end of the reading lesson: ‘’ What did I
learn today?”, ‘What still puzzles me?” What did I enjoy, hate, accomplish in class today?” ,
“How did I learn from the lesson?’’ , Which of the following are you asking them to do?

135

Downloaded by Noime Burtanog (noimeburtanog@gmail.com)


lOMoARcPSD|38673565

a. Work on an assignment c. Work on a drill


b. make entry journal d. Apply what they learn
66. If the student is encourage to develop himself to the fullest and must satisfy his hierarchy of
needs, the highest need to satisfy according to Maslow is ___________.
a. psychological need c. belongingness
b. self-actualization d. safety need
67. In Social Studies class, Teacher I presents a morally ambiguous situation and asks students
what they would do. On whose theory is Teacher I’s technique based?
a. Bandura c. Kohlberg
b. Freud d. Erickson
68. Based on Freud’s theory, which operate/s when a student strikes a classmate at the height of
anger?
a. Ego c. Id and Ego
b. Id d. Erickson
69. The Computer r for score in Math and Science is 0.92. What does this mean?
a. Math score is positively related to Science score
b. The higher the Math score, the lower the Science score
c. math score is not in any way related to Science score
d. Science score is slightly related to Math score
70. Which type of the tests is most appropriate if Teacher Y wants to measure students’ ability to
organize thoughts and ideas?
a. Short answer c. Alternative response
b. Essay d. Multiple Choice
71. If Teacher wants to measure her students’ ability to discriminate, which of these is an
appropriate type of the test item as implied by the direction?
a. “Outline the Chapter on The Cell.”
b. “Summarize the lesson yesterday.”
c. “Group the following items according to shape”
d. “State a set of principles that can explain”
72. A test item has a difficulty index of 0.89 and a discrimination index of 0.44. What should the
teacher do?
a. Reject the item c. Make it a bonus item
b. Retain the item d. Make it a bonus item and reject it.
73. What can be said of Arielle who obtained a score of 75 out of 100 items in a Grammar
objective test?
a. She performed better than 25% of her classmate
b. She answered 25 items in the test correctly.
c. Her rating is 75.
d. She answered 75% of the test items correctly.
74. The criterion of success in Teacher D’s objective is that “the pupils must be able to spell 90%
of the words correctly.” Ana and 24 others in the class spelled only 40 out of 50 words correctly
while the rest scored 45 and above. This means that Teacher D ___________.
a. attained her lesson objectives
b. did not attained her lesson objective because of the pupils lack of attention
c. failed to attain her lesson objective as far as the 25 pupils are concerned
d. attained her lesson objective because of her effective spelling drill
75. For her discussion of the topic from various perspectives, it is BEST to hold __________.
a. brainstorming c. debate
b. Symposium d. panel discussion
76. Availment of the Philippine Education Placement Test for adult and out-of-school youth is in
the support of the government’s educational program towards _______________.
a. Relevance c. Equitable access
b. Quality d. Quality and Relevance
77. Which MDG goal is related to the state’s goal for equality education?
a. MDG 1 c. MDG 3
b. MDG 2 d. MDG 4
78. Which illustrates vicarious punishment?
a. We feel so bad to the classmate who is punished for being tardy so we convince him go to
school on time.
b. Out of comparison, we volunteer to get punished in the place of friend
c. We charge to experience our being punished
d. See someone who get punished for habitual tardiness. In effect, we are fees likely to be tardy

136

Downloaded by Noime Burtanog (noimeburtanog@gmail.com)


lOMoARcPSD|38673565

79. Which objective in the effective domain is the lowest level?


a. To accumulate examples of authencity
b. To support view against abortion
c. To respond positively to a comment
d. To formulate criteria for honestly
80. In instructional planning, it is necessary that the parts of the plan from the first to the last
have:
a. Symmetry c. Conciseness
b. Coherence d. Clarity
81. The military training requirements among students in the secondary and tertiary levels can be
traced as a strong influence of the
a. Spartans c. Chinese
b. Romans d. Athenians
82. To promote effective practice, which guideline should you bear in mind? Practice should be
_________.
a. Difficult for students to learn a lesson
b. Arrange to allow students to receive feedback
c. Done in an evaluative atmosphere
d. Take place over a long period of time
83. By what name is direct instruction or Socratic Method also known?
a. questioning method c. indirect method
b. Morrison method d. mastery learning
84. Which does NOT belong to the group of alternative learning systems?
a. Multi-age grouping c. non-grade grouping
b. Multi-grade grouping d. graded education
85. In the social-constructivist perspective, the child is given a task but he/she cannot accomplish
it alone without the assistance of adults or peers around him/her.
a. ZPD c. Social learning theory
b. the Socratic method d. peer tutoring
86. Which term applies to the search for related literature by computer access of data bases on
disc kept in libraries?
a. On-line research c. Compact disc computer research
b. Manual research d. computer research
87. The whole child concept of education is contrary to the:
a. Academic essentials approach to education
b. Progressivist approach to education
c. Focus of humanistic approach to education
d. The naturalistic approach to education
88. Here is a lesson objective in literature: “Using the six description of the elements of a good
short story identify in writing the six elements in the short story by O. Henry with complete
accuracy.” Applying Robert Mager’s principle on writing performance objectives, which is the
criterion measure?
a. “ the six element in the short story by O. Henry with complete accuracy
b. “identify in writing”
c. “ with complete accuracy”
d. “ using the six description of the element for a short story”
89. With Bloom’s cognitive taxonomy in mind, which objective is in the highest level?
a. To rate a project along relevance, originally and craftsmanship
b. to state the assumption that underlies the given statements
c. To state generalization from the data given
d. To write a paragraph that observe s unity from the data given
90. Which refer to a single word or phrases that tell the computer to do something with a
program or file?
a. Computer program c. Computer language
b. Password d. Command
91. What is the mean of the score distribution: 4, 5,6,7,8,9,10?
a. 7.5 c. 6
b. 8.5 d. 7
92. Which is the teaching approach for a kindergarten makes real world experiences of the child
the focal point of educational stimulation?
a. Situation approach c. Electric approach
b. Traditional approach d. Montessori approach

137

Downloaded by Noime Burtanog (noimeburtanog@gmail.com)


lOMoARcPSD|38673565

93. Which is a characteristic of an effective classroom management?


a. Coercive c. Imposed
b. Reactive d. Preventive
94. Which can run counter to the encouragement you to give your students to ask question?
a. Knitted eyebrow when a question is raised
b. eye to eye contact
c. radiant face
d. an encouraging hand gesture
95. Which one can help students develop the habit of critical thinking?
a. Blind obedience of authority
b. Asking convergent questions
c. Asking low level questions
d. A willingness to suspend judgement until sufficient evidence is presented
96. In instructional planning, which among the three: unit, plan, course plan, lesson plan is most
specific?
a. Course c. Resource
b. Unit d. Lesson
97. Teacher Mar wants to develop in her pupil’s comprehension skills. What order of skills will
she develop?
I. Literal comprehension III. Critical Evaluation
II. Interpretation IV. Integration
a. II-III-IV-I c. I-II-III-IV
b. III-IV-I-II d. IV-III-II-I
98. If the student thinks about hi/her thinking, he is involved in the process called:
a. Higher-order thinking c. Creative thinking
b. Critical thinking d. Metacognition
99. The use of drills in the classroom is rooted on Thorndike’s law of:
a. Readiness c. Exercise
b. Effect d. Belongingness
100. The following are some drill techniques, EXCEPT
a. Challenging students to be above the level of the class
b. Asking pupils to repeat answers
c. Giving short quiz and having students grade papers
d. Assigning exercises from a workbook
101. The first American teachers in the Philippines were
a. Soldiers c. Missionaries
b. Graduate of the normal school d. Elementary graduates
102. Which questioning technique promotes more classroom interaction?
a. Focusing in convergent question
b. Calling on the student before asking question
c. focusing on divergent question
d. Asking rhetorical question
103. Teacher V wants to check prior knowledge of his pupils about water pollution. She writes
the main topic water pollution in the center of the chalkboard and encircles it. Then, she asks the
pupils to provide information that can be clustered around the main topic. Which technique did
the teacher employ?
a. Vocabulary building c. Semantic Mapping
b. Demonstration d. Deductive teaching
104. Who were the Thomasites?
a. The soldiers who doubted the success of the republic educational system to be set in the
Philippines
b. The first American teacher-recruits to help establish the public educational system in the
Philippines
c. The first religious group who came to the Philippines on board the US transports Thomas
d. The devotee to St. Thomas Aquinas who came to evangelize Filipinos
105. To build a sense of pride among Filipino youth, which should be done?
a. Re-study our History and stress on our achievements as a people
b. Re-study our history from the perspective of our colonizers.
c. Replace the study of folklores and myths with technical subjects
d. Set the study of local history
106. “Specialization is knowing more and more about less and less. Then it is better to be
generalist”, claims Teacher F. On which philosophy does Teacher F learn?

138

Downloaded by Noime Burtanog (noimeburtanog@gmail.com)


lOMoARcPSD|38673565

a. Essentialism c. Perenialism
b. Progressivism d. Existentialism
107. Teacher Lei gives his students opportunity to be creative because of his conviction that
much learning results from the need to express creativity. On which theory of teacher Lei
conviction anchored? _______________ theory
a. Behaviorist c. Cognitive
b. Associationist d. Humanist
108. Which teaching activity is founded on Bandura’s social learning theory?
a. Questioning c. Modeling
b. Inductive reasoning d. Interactive reasoning
109. I like to develop the synthesizing skills of my students. Which one should I do?
a. Ask my student to formulate a generalization from the data shown in the graphs
b. Direct my students to point out w/c part of the graph are right and w/c part is wrong
c. Ask my students to answer the questions beginning with “What if….”
d. Tell my students to state data presented in the graph.
110. To reach out to clientele who cannot be in the classroom for one reason or another, which of
the following was established?
a. Informal c. Pre-school education
b. Special education (SPED) system d. Alternative learning delivery
111. Nicolle enjoys games like scrabble, anagrams, and password. Which type of intelligence is
strong in Nicolle?
a. Interpersonal intelligence
b. Logical and mathematical intelligence
c. Linguistic intelligence
d. Spatial intelligence
112. Teacher Cora observes cleanliness and order inter classroom to create a conductive
atmosphere for learning. On which theory is her practice based?
a. Psychoanalysis c. Behaviorism
b. Gestalt psychology d. Humanistic psychology
113. Which learning principle is the essence of Gardner’s theory of multiple intelligence?
a. Almost all learners are linguistically intelligent
b. Intelligence is not measured in one form
c. Learners have different IQ level
d. Learners have static IQ
114. Based on Edgar Dale’s Cone of Experience, which activity is farthest from the real thing?
a. Watch a demo c. Listen to a lecture
b. View image d. Attend exhibit
115. Which is a proactive management practice?
a. Tell them that you enforce the rules on everyone, no exception
b. Set and clarity your rules and expectations on Day 1
c. Punish the misbehaving pupils in the presence of their classmate
d. Stress on penalty for every violation
116. For integrative purposes, which method/approach should be used?
a. Metacognitive approach c. Thematic approach
b. Cooperative approach d. Constructive approach
117. Which quotes goes with a proactive approach to discipline?
a. “An ounce of prevention is better than a pound of cure.”
b. “Do not make a mountain of a mole.”
c. “Walk your talk”
d. “ Do not smile until Christmas”
118. What does the principle of individual differences require teacher to do?
a. Give less attention to gifted learners.
b. Provide for a variety of learning activities.
c. Treat all learners alike while teaching.
d. Prepare modules for slow learners in class.
119. Teachers are encouraged to make use of authentic assessment. Which goes with authentic
assessment?
a. De-contextualized drills
b. Unrealistic performances
c. Answering multiple choices test items
d. Real word application of lesson learned

139

Downloaded by Noime Burtanog (noimeburtanog@gmail.com)


lOMoARcPSD|38673565

120. I’d like to test whether a student knows what a particular word means. Which should I ask
the students to do?
a. Give the word a tune then sing it
b. Define the word.
c. spell the word and identify its part of speech
d. Give the etymology of the word.
121. With SMART lesson objectives in the synthesis level in mind, which one does NOT belong
to the group?
a. Formulate c. Build
b. Organize d. Dissect
122. Who asserted that children must be given the opportunity to explore and work on different
materials so that they will develop the sense of initiative instead of guilt?
a. Kohlberg c. Maslow
b. Erickson d. Gardner
123. Teacher Zen uses direct instruction strategy. Which will she first do?
a. Presenting and structuring c. Independence practice
b. Guided student practice d. Reviewing the previews day’s work
124. The free public elementary and secondary education in the country is in the line with the
government effort to address educational problems of _____________.
a. Productivity c. Access and equality
b. Relevance and Quality d. Effectiveness and efficiency`
125. Here row score in the quiz 97 95 85 83, 77, 75, 50 10, 5 ,2, 1, to get a picture of the group’s
performance, which measure of central tendency is MOST reliable/
A. Median
B. Mean
C. Mode
D. non.it is best to look at the individual scores
126. Which statement about standard deviation is CORRECT?
a. The lower the standard deviation the more spread the scores are.
b. The higher the standard deviation the more spread the scores are.
c. The higher the standard deviation the less spread the scores are
d. It is a measure of central tendency.
127. which is the teachers’ professionalization act?
a. R A 7836
b. R A 4670
c. R A 7722
d. R A 9293
128. The state shall protect and promote the right of all citizens to quality education at all level.
’’Which government program is in support of this?
a. Exclusion of children with special needs from the formal system
b. Free elementary and secondary education
c. Deregulated tuition fee hike
d. Re-introduction of the NEAT and NSAT
129. The K-12 curriculum is the otherwise called as
a. Basic Education Curriculum
b. Revitalized Basic Education Curriculum
c. Enhanced Basic Education Curriculum
d. Extended Basic Education Curriculum
130. The Filipino learners envisioned by the Department of Education in the light of K-12
curriculum is
a. Technologically literate and holistically developed Filipinos
b. Functionally literate and holistically developed Filipinos
c. Scientifically advanced and values oriented Filipinos
d. Nationally oriented and internationally competitive Filipinos
131. Principle tells her teachers that training in the humanities is most important To which
education philosophy does he adhere?
a. Existentialism
b. progressivism
c. Essentialism
d. Perennialism
132. Tony exhibits fear response to freely roaming dogs but does not show fear wen a dog is on a
leash or confined in a cage. Which conditioning process is illustrated?

140

Downloaded by Noime Burtanog (noimeburtanog@gmail.com)


lOMoARcPSD|38673565

a. Generalization c. Discrimination
b. Acquisition d. Extinction
133. Behaviour followed by pleasant consequences will be strengthened and will be more likely
to occur in the future. Behaviour followed by unpleasant Consequences will be weakened and
will be less likely to be repeated in the future. Which one is explained?
a. Freud’s psychoanalytic theory
b. Thorndike’s law of effect
c. B.F. Skinner’s operant conditioning theory
d. Bandura’s social learning theory
134. Theft of school equipment like T.V., computer, etc. by people in the community itself is
becoming a common phenomenon. Which does this incident signified?
a. Prevalence of poverty in community
b. Inability of school to hire security guards
c. Deprivation of Filipino schools
d. Community’s lack of sense of co-ownership
135. The main purpose of compulsory study of Constitution is to ____________.
a. Develop students into responsibilities, thinking citizen.
b. Acquaint students with the historical development of the Phil. Constitution.
c. Make constitutional experts of the students
d. Prepare students for law-making
136. With which goals of educational institution as provided for by the constitution is the
development of work skills aligned?
a. To develop moral character c. To teach the duties of citizenship
b. To develop vocational efficiency d. To inculcate love of country
137. Parents admit that SMS has an effect on _______________.
a. Communicating through guessing c. Shortening messages effectively
b. Ability to guess words correctly d. Ability to spell words correctly
138. In computer based instruction, which tool can help you revise your short stories essays and
other written work?
a. Word processing c. Database
b. Spread sheet d. Desktop publishing
139. Which is closest to the real human digestive system for study in the classroom?
a. Model of the human digestive system.
b. Drawing of the human digestive system on the board.
c. The human digestive system projective on an OHP
d. Drawing of the human digestive system on the page of the textbook.
140. Here is a question: “Is the paragraph a good one? Evaluate.” If broken down to simplify,
which is the BEST simplification?
a. Is the paragraph a good one? Why or why not?
b. Why is the paragraph a good one? Prove.
c. If you ask to evaluate something, what do you do? Evaluate the paragraph.
d. What qualities of a good paragraph? Does the paragraph have these qualities?
141. I want my student to have mastery learning of a basic topic. Which can help?
a. Problem Solving c. Lecture Method
b. Drill d. Socratic Method
142. What is the mastery level of a school in a 100-item test with a mean of 55?
a. 42% c. 45%
b. 50% d. 55%
143. Who stressed the idea that students cannot learn if their basic needs are not first met?
a. Thorndike c. Wertheirmer
b. Maslow d. Bandura
144. A person who had painful experiences at the dentist’s office may become fearful at the mere
sight of the dentist’s office building. Which theory can explain this?
a. Attribution theory c. Social learning
b. Classical conditioning d. Operant conditioning
145. I want the fast learners in my class to do self-directed learning. What strategy will I use?
a. Collaborative learning c. Meta-cognitive
b. Social dialogue d. Problem based learning
146. For which lesson objective will you use the direct instruction method?
a. Appreciate Milton’s Paradise Lost
b. Use a microscope properly
c. Distinguish war from aggression

141

Downloaded by Noime Burtanog (noimeburtanog@gmail.com)


lOMoARcPSD|38673565

d. become aware of the pollutants in the environment


147. To encourage introspection, which teaching method is MOST appropriate?
a. Cognitive c. Process
b. Reflective d. Cooperative
148. With indirect instruction in mind, which does NOT belong to the group?
a. Experimental method c. Lecture-recitation method
b. Inductive method d. Discovery method
149. A master teacher, the resource speaker in an in-service training presented the situated
learning theory and encouraged her colleagues to apply the same in-class. Which did she NOT
encourage her colleagues to do?
a. Apprenticeship c. Learning as it normally occurs
b. Decontextualized teaching d. Authentic problem solving
150. Teachers are encouraged to make use of authentic assessment. Which goes with the
authentic assessment?
a. De-contextualized drills
b. Unrealistic performances
c. Answering multiple choice test question
d. Real word application of lesson learned
151. Teacher Jay discovered that her pupils are weak in comprehension. To further determine in
which particular skill(s) her pupils are weak; which test should Teacher Jay give?
a. Standardized Test c. Placement Test
b. Aptitude Test d. Diagnostic Test
152. Which will be the most authentic assessment tool for an instructional objective on working
with and relating with people?
a. Writing articles on working relating to people
b. Organizing a community project
c. Home visitation
d. Conducting mock election
153. Teacher Lorelle does norm-referenced interpretation of score. Which of the following does
she do?
a. She uses a specified content as frame reference.
b. She describes the class performance against a set level of mastery.
c. She compares individual student’s scores with other student’s scores.
d. She describes what should be every individual student’s performance.
154. You practice inclusive education. Which of these apply to you?
I. You accept every student as full and valued member of the class and school community.
II. Your special attention is on learners with specific learning or social needs
III. You address the needs of the class as a whole within the context of the learners with specific
learning or social needs.
a. II only c. I only
b. I and II d. I and III
155. What principle is violated by overusing the chalkboards, as though it is the only education
technology available?
a. Isolated use c. Variety
b. Flexibility d. Uniformity
156. Which statement applies CORRECTLY to Edgar Dale’s “Cone of Experience”?
a. The father you are form the base, the more direct the learning the experience becomes.
b. The father you are form the bottom, the more direct the learning experience becomes
c. The closer you are to the base, the more indirect the learning experience become.
d. The closer you are to the base, the more direct learning experience becomes.
157. I want to teach concepts, patterns and abstractions. Which method will be MOST
appropriate?
a. Discovery c. Direct instruction
b. Indirect instruction d. Problem solving
158. Teacher Ann, an experienced teacher, does daily review of past lessons in order to ____.
a. Introduce a new lesson
b. Reflect on how presented the previous lesson
c. Provide his pupils with a sense of continuity
d. Determine who among his pupils are studying
159. To nurture students’ creatively, which activity should be a AVOID?
a. Ask “what if…” questions
b. Ask divergent thinking questions

142

Downloaded by Noime Burtanog (noimeburtanog@gmail.com)


lOMoARcPSD|38673565

c. Emphasize the need to give right answer


d. Be open to “out-of-this-world” ideas
160. After reading and essay, Teacher B wants to help sharpen her student’ ability to interpret.
Which of these activities will be most appropriate?
a. Drawing conclusion c. Getting the main idea
b. Making interference d. Listing facts separately from opinion
161. Under no circumstances shall a teacher be prejudiced nor discriminatory against any learner,
says the Code of Ethics. When is a teacher prejudice against any learner?
a. When he makes a near sighted pupils set at the front
b. When he considers multiple intelligences in the choice of teaching strategies
c. When he makes a farsighted pupil sit at the back
d. When he refuses a pupil with slight physical disability in class
162. Which learning activity is most appropriate if teacher’s focus I attitudinal change?
a. Role play c. Exhibit
b. Fieldtrip d. Game
163. The mode of a score distribution is 25. This means that
a. There is no score of 25.
b. Twenty five (25) is the score that occurs most
c. Twenty five is the average of the score distribution
d. Twenty five is the core that occurs least
164. The following characterize a child-centered kindergarten EXCEPT ______.
a. Focus on the education of the whole child
b. Importance of play in development
c. Extreme orientation an academic
d. Emphasis on individual uniqueness
165. The last year of pre-service education is student teaching. Student teaching is classified as
____________.
a. Role playing c. Field trip
b. Simulation d. Demonstration
166. Which term refers helping a colleague grow professionally?
a. Technology transfer c. Facilitating
b. Peer monitoring d. Independent study
167. What primary criterion should guide a teacher in the choice of instructional devices?
a. Novelty c. Appropriateness
b. Cost d. Attractiveness
168. Which of the following is considered peripheral device?
a. Printer c. CPU
b. Keyboard d. Monitor
169. Which questioning practice will promote more class interaction?
a. Asking rhetorical question
b. Rejecting wrong question
c. Focusing on convergent question
d. Asking the question before calling on a student to answer
170. Research on Piagetian tasks indicates that thinking becomes more logical and abstract as
children reach the formal operations stage. What is an educational implication of this finding?
a. Expect hypothetical reasoning for learners between 12 to 15 years of age.
b. Learners who are not capable of logical reasoning from ages 8 to 11 behind in their cognitive
development.
c. Engage children in analogical reasoning as early as preschool to train them for higher order
thinking skills (HOTS)
d. Let children be children
171. A mathematics test was given to all Grade V pupils to determine the contestants for the
Math Quiz Bee. Which statistical measure should be used to identify the top 15?
a. Percentage score c. Quartile Score
b. Mean percentage Score d. Percentile Score
172. How can you exhibit expert power on the first day of school?
a. By making them feel you know what you are talking about
b. By telling them the importance of good grades.
c. By reminding your students your authority over them again and again
d. By giving your students a sense of belonging and acceptance
173. The following are sound specific purposes of questioning EXCEPT ___________.
a. To stimulate learners to ask questions

143

Downloaded by Noime Burtanog (noimeburtanog@gmail.com)


lOMoARcPSD|38673565

b. To call attention of an attentive student


c. To arouse interest and curiosity
d. To elicit answers from students
174. Which is NOT a sound purpose for asking questions?
a. To remind students of a procedure
b. To probe deeper after an answer is given
c. To encourage self-reflection
d. To discipline a bully in class
175. A student passes a book report poorly written but ornately presented in a folder to make up
for the poor quality of the report of the book report content. Which Filipino trait does this
practice prove?
a. Art over science c. Substance over “porma”
b. Art over academic d. “Porma” ove substance
176. Which material consists of instructional units that cater to varying mental levels pupils?
a. Plantilla c. Multi-grade materials
b. Multi-level materials d. Minimum learning competencies
177. In K-W-L technique K stands for what the pupils already knows, W for what he wants to
know and L for what he
a. Learned c. Failed to learn
b. he likes to learn d. Needs to learn
178. Self-rating and follow-up conference with teacher benefit both teacher and student. The
following are for the students EXCEPT
a. heighten student’s defensiveness
b. helps the student develop increased skill in self-assessment
c. helps the students recognize his progress toward the instructional objectives
d. helps the student diagnose his particular strengths and weaknesses
179. Which one should you do if a parent of one of your failing pupils asks you to tutor her
daughter in consideration of a certain amount of money which badly need?
a. Accept the offer but do tutoring outside office hour
b. direct the parent to another tutor and make internal arrangement with the tutor for commission
c. accept the offer with discount
d. advise the parent to look for another tutor
180. Teacher C does not personally agree with one school policy. What is the professional thing
for him to do?
a. lead a campaign against the abolition of that school policy
b. make honest effort to understand, support, and carry out the school policy even if he does not
personally agree
c. be indifferent about it sa she exerts effort to understand
d. defy the policy because in conscience he cannot agree
181. A teacher should not be a slave of his lesson plan. This means that
a. A lesson plan must be followed by a teacher no matter
b. A teacher must be ready to depart from her lesson plan if she remembers something more
interesting that what she earlier planned
c. A teacher must be willing to depart from her lesson plan if students are interested in something
other than her interested lesson.
d. Teacher is the best lesson plan designer
182. To provide for individual differences how is curriculum designed?
a. Minimum learning competencies are included
b. Realistic and meaningful experiences are provided
c. Some degree of flexibility is provided
d. Social skills are emphasized
183. Which is the TRUE foundation of the social order?
a. Strong, political leadership c. Equitable distribution of wealth
b. The reciprocation of rights and duties d. Obedient citizenry
184. A test consists of a graph showing the relationship between age and population. Following it
is a series of true-false items based on the graph. Which type of test does this illustrate?
a. Laboratory exercise c. Performance
b. Problem solving d. Interpretive
185. Which curricular move served to strengthen spiritual and ethical values?
a. Integration of creative thinking in all subject
b. Reducing the number of subject areas into the skills subject
c. Introduction of Values Education as separate subject area

144

Downloaded by Noime Burtanog (noimeburtanog@gmail.com)


lOMoARcPSD|38673565

d. Re-introducing Science as all subject in Grade 1


186. A teacher notices glaring wrong pronunciation of vowel sounds among her students
necessitating more practice. Which of the following activities would be of help?
a. Dictionary c. Assignment
b. Review d. Drill
187. Teacher A is observed to be a bit aloof from the children of the Aetas. She justifies her
action by saying “We are about human; we cannot like every pupil.” Is she acting ethically?
a. Yes, because Teacher A is not yet engaged in a destructive behaviour.
b. Yes, persons have their own peculiarities and we expect that we cannot like everybody in the
same way that not everybody can like us.
c. No, under no circumstances shall a teacher be prejudices against any learner
d. No, unless she has extraordinary reason for being aloof to the Aetas.
188. Which of the following teaching practices should be AVOIDED?
a. Using “put down” strategy c. Asking more divergent questions
b. Using multiple response strategy d. Asking more evaluative questions
189. Which of the following embodies the operation “return to the basics”?
a. National Secondary Achievement Test
b. New Secondary Curriculum
c. New Elementary School Curriculum
d. National Elementary Achievement Test
190. Which of the principle of development is manifested in the saying “as a tree is bent, so shall
it grow that of, “a person’s action reflect the training he received as a child?”
a. The early formative years are more crucial than later development
b. Development is determined by the environment or how one is nurtured
c. Development is determined by heredity or one’s inherent nature
d. It is never too late to teach a child new ways

145

Downloaded by Noime Burtanog (noimeburtanog@gmail.com)

You might also like